You are on page 1of 153

Chapter 8

Kinematics and Kinetics of Rigid


Bodies in Three-dimensional Motion

8.1 Spherical coordinates

8.2 Angular Velocity of Rigid Bodies in Three-Dimensional Mo-


tion

8.3 Angular Acceleration of Rigid Bodies in Three-Dimension Mo-


tion

8.4 General Motion Of and On Three-Dimensional Bodies

87
8.4.1
GOAL: Determine the angular velocity and acceleration of one arm of the illustrated mechanism.
GIVEN: Constant angular velocity of inner arm, and constant angular velocity of outer arm with
respect to inner arm
DRAW: The figure shows the mechanism with the original coordinate axes and some newly at-
*
tached unit vectors. Unit vectors * ı,* , k are aligned with the ground-fixed X, Y, Z axes. Unit
* * *
vectors b 1 , b 2 , b 3 are attached to the inner arm OA.

FORMULATE EQUATIONS: We’ll use the expressions for angular velocity and acceleration
on a rotating body.
SOLVE: The angular velocity of arm AB is equal to the angular velocity of arm OA plus the
*
relative angular velocity of arm AB with respect to arm OA. The angular velocity of OA is ψ̇ k ,
*
and the relative angular velocity of AB with respect to OA is −θ̇ b 2 . Thus:

* * *
ωAB = ψ̇ k − θ̇ b 2 (1)
*
This can be written is the b -frame as:

* * *
ωAB = ψ̇ b 3 − θ̇ b 2

To determine the angular acceleration of AB we can differentiate (1):

d* d  * d* d*
    
* * * *
αAB = ωAB = ψ̇ k − θ̇ b 2 = ψ̈ k + ψ̇ k − θ̈ b 2 −θ̇ b
dt dt |{z} dt |{z} dt 2
=0 | {z } =0
=0

*
 
d *
Since the angular speeds are constant, and k is fixed in space, the only term remaining is −θ̇ dt b 2 .
* *
The tip of unit vector b 2 sweeps in the − b 1 direction with speed ψ̇. So we have:

d*
 
* *
αAB = −θ̇ b 2 = −θ̇ψ̇(− b 1 )
dt

* *
αAB = θ̇ ψ̇ b 1

88
Alternatively, we could have used the expression:

d * d * * *
ω
dt AB
= ω +ω
dt S AB OA ×ωAB
* * * *
= 0 + ψ̇ k ×(ψ̇ k − θ̇ b 2 ) = θ̇ψ̇ b 1

89
8.4.2
GOAL: Determine the angular velocity of a rotating disk.
GIVEN: Inner shaft’s angular velocity and the angular velocity of the disk with respect to the
shaft.
DRAW:

FORMULATE EQUATIONS: We’ll use the expression for angular velocity on a rotating body.
SOLVE: The angular velocity of the disk D is equal to the angular velocity of inner shaft AB plus
the relative angular velocity of disk D with respect to shaft AB. The angular velocity of AB is
−ω1 *
 , and the relative angular velocity of disk to shaft is ω2 *
ı . Thus:
* * *
ωD = ω2 ı − ω1 

90
8.4.3
GOAL: Determine the angular acceleration of a rotating caster.
*
GIVEN: Angular velocity of the caster’s frame is ω1 b 3 and the angular velocity of the caster with
*
respect to the frame is −ω2 b 1 .
DRAW:

FORMULATE EQUATIONS: We’ll use the expression for acceleration on a rotating body:

d * d * * *
ω = ω +ω F ×ωC (1)
dt C dt C
N F
SOLVE: The caster C has a constant rotation rate with respect to the frame F and so (1) simplifies
to
d * * * * * *
ω =ω F ×ωC = ω1 b 3 ×(−ω2 b 1 ) = −ω1 ω2 b 2
dt C
N

* *
αC = −ω1 ω2 b 2

91
8.4.4
GOAL: Determine the angular velocity and acceleration of one wheel of the illustrated mechanism.
* *
GIVEN: Angular velocity of the three-armed body is given by ω T = 2  rad/s. h = 0.1 m and
r = 0.004 m. * * * *
DRAW: The figure shows the mechanism with ground-fixed * ı,* , k unit vectors as well b 1 , b 2 , b 3
(fixed to the rotating body T ).

ASSUME: We’ll assume that the wheels roll without slip on the bottom of the microwave oven.
FORMULATE EQUATIONS: We’ll use the expressions for angular velocity and acceleration
on a rotating body.
SOLVE: The angular velocity of the wheel A is equal to the angular velocity of arm three-arm
body (T ) plus the relative angular velocity of wheel A with respect to the body. The angular
* *
velocity of the three arm body is 2 *
 rad/s = 2 b 2 rad/s ( *
 and b 2 are identical for this problem).
The wheel rolls without slip and can find the wheel’s angular velocity by determining the speed of
the wheel A’s center (GA ) and then using
v*C = v*GA + ω
* *
A × r C/
GA

Applying the no-slip constraint that v*C = 0 will get us the angular velocity information we need.
We’ll assume that the wheel A is rotating with respect to T with angular speed φ̇.
* * *
v*A = (2 b 2 rad/s)×(0.1 b 1 m) = −0.2 b 3 m/s
v*C = v*A + v*C/
A
* * *
= −0.2 b 3 m/s + ω A ×(−r b 2 )
* * * *
= −0.2 b 3 m/s + (φ̇ b 1 + 2 b 2 )×(−r b 2 )
* *
= −0.2 b 3 m/s − rφ̇ b 3
Applying the no-slip constraint that v*C = 0 gives us
0.2 m/s 0.2 m/s
φ̇ = − =− = −50 rad/s
r 0.004 m
Thus we have
* * *
ω A = (−50 b 1 + 2 b 2 ) rad/s

d * d* d*
   
* *
αA = (ωA ) = 50 b 1 + 2 b 2 rad/s = 50(−2 b 3 rad/s) rad/s
dt dt dt
* * 2
αA = −100 b 3 rad/s

92
8.4.5
GOAL: Compute the angular acceleration of a bicycle wheel as a bicyclist travels a circular path.
GIVEN: Radius of wheel, its inclination from vertical, time for the bicyclist to complete one full
circle, and forward speed of bicyclist.
DRAW:

* * *
The x, y, z coordinate frame, with corresponding unit vectors b 1 , b 2 , b 3 is fixed to the frame of the
*
bicycle. The *ı,* , k unit vectors are fixed to the inertial ground frame. The relationship between
these two sets of unit vectors is:
* * *
ı  k
*
b1 cos φ 0 sin φ
*
b2 0 1 0
*
b3 − sin φ 0 cos φ

ASSUME: We assume that the given 25 mph speed is that of the contact between the wheel and
ground as it travels a circular path along the ground. This neglects the small variations in forward
velocity that result from the inclination angle of the bike and the bicyle/rider’s dimensions. (e.g.
Since the bicyclist’s center of mass travels a smaller circle than the center of the wheel, the forward
velocities of each would be slightly different.)
FORMULATE EQUATIONS: We’ll use the expressions for angular velocity and acceleration
of a rotating body.
SOLVE: First we need to determine the two angular rates; that of the bicyclist about the center
of the circular path, and that of the wheel about its center. Since the bicyclist completes one full
circle in 6 s, we have:
2π rad π
θ̇ = = rad/s
6s 3

The angular speed of the wheel about its center will be the forward speed of the wheel divided by
its radius:
25 mph 440 in/s
ψ̇ = = = 33.8 rad/s
13 in 13 in

These angular rates are constant.


The angular velocity of the bicycle about the center of the path is −θ̇ *
ı , and the angular velocity
*
of the wheel with respect to the frame of the bicycle is −ψ̇ b 3 . The total angular velocity of the
wheel is the sum of these:
* *
ω = −θ̇ *
ı − ψ̇ b 3

93
The wheel’s angular acceleration may be computed from:

*
d * d * * *
α = ω = ω +ω S ×ωBody
dt N Body dt S Body

d * *
= (−θ̇ *
ı − ψ̇ b 3 ) + (−θ̇ *
ı )×(−θ̇ *
ı − ψ̇ b 3 )
dt xyz
*
= 0 + θ̇ψ̇( *
ı × b 3)

From the transformation matrix, the unit vector *


ı is
* * *
ı = cos θ b 1 − sin θ b 3

Thus
* * * * *
α = θ̇ψ̇(cos θ b 1 − sin θ b 3 )× b 3 = θ̇ψ̇ cos θ b 2
π
 
*
*
α = − rad/s (33.8 rad/s) cos 30◦ b 2
3
*
*
α = −30.7 b 2 rad/s2

94
8.4.6
GOAL: Determine the angular acceleration of a person’s forearm.
GIVEN: Time for the arm segments to reach given configurations. Body geometry.
DRAW:

FORMULATE EQUATIONS: We’ll use the expression for acceleration on a rotating body:

d * d * * *
ω = ωBC + ω F ×ωBC (1)
dt BC dt
N F

where F indicates a rotating frame of reference that is rotating at the same speed as AB.
SOLVE:
* * *
ωBC = ω1 b 1 + ω2 b 2

We’re given that the two arm segments change orientation by π 2 rad in 1 s and so have
π π
ω1 = − rad/s, ω2 = rad/s
2 2
The rotation rates are constant so our angular acceleration expression simplifies to


* d ω
* * *
αBC = dt
BC
=ωF ×ωBC
*
N * *
= ω1 b 1 ×(ω1 b 1 + ω2 b 2 )
*
= ω1 ω2 b 3

* π π * π2 * 2
αBC = (− 2 rad/s)( 2 rad/s) b 3 = − 4 b 3 rad/s

95
8.4.7
GOAL: Determine a forearm’s angular acceleration for 0 ≤ t ≤ 1 s.
GIVEN: Rotational information with regard to the arm segments and body geometry.
DRAW:

FORMULATE EQUATIONS: We’ll use the expression for acceleration on a rotating body:

d * d * * *
ω = ωBC + ω F ×ωBC (1)
dt BC dt
N F

where F indicates a rotating frame of reference that is rotating at the same speed as AB.
SOLVE: The arm segment AB alters orientation by π 2 rad in 1 s and so we have
* * π*
ω AB = ω1 b 1 = − 2 b 1 rad/s

BC rotates with a constant angular acceleration relative to AB such that after starting from rest
it has moved π
2 rad after 1 s. Denoting its relative rotation angle with φ, we have
π 1 2(π/2)
∆φ = rad = φ̈t2 ⇒ φ̈ = ⇒ φ̈ = π rad/s2
2 2 (1 s)2
We can now use our formula for the derivative of a vector in a rotating body:


* d ω
* * *
αBC = dt
BC
+ωF ×ωBC
*
F * * *
= φ̈ b 2 + ω1 b 1 ×(ω1 b 1 + (φ̈t) b 2 )
* *
= φ̈ b 2 + ω1 φ̈t b 3
 
* *
αBC = π b 2 rad/s − 2 π2 * 3
2 b 3 rad/ s t

96
8.4.8
GOAL: Determine the angular velocity and angular acceleration of the forearm segment during a
given motion of the arm.
GIVEN: Position of the forearm and upper arm at times t = 0 and t = 1 s, and the fact that the
angular speeds of the segments are constant.
* * *
DRAW: The figures show the arm at times t = 0 and t = 1 s. The unit vectors b 1 , b 2 , b 3 , fixed to
the upper arm, are shown along with the inertial x, y, z axes. At time t = 1 s, φ = 90◦ and ψ = 45◦ .

FORMULATE EQUATIONS: We’ll use the formulas for the angular velocity and acceleration
of a rotating body.
SOLVE: Since the angular speeds are constant, we can determine the values by dividing the change
in angle by the change in time:

∆ψ π/4 rad π
ψ̇ = = = rad/s
∆t 1s 4

∆φ π/2 rad π
φ̇ = = = rad/s
∆t 1s 2
The angular velocity of the forearm is the angular velocity of the upper arm plus the angular
velocity of the forearm with respect to the upper arm:
* * *
ωBC = ωAB + ωBC /AB
* *
= −φ̇ b 1 + ψ̇ b 3

* π * π *
ωBC = − 2 b 1 + 4 b 3 rad/s

The angular acceleration is thus:

d π* π* π*˙ π*˙ π π*
   
* *˙
αBC = ωBC = − b1 + b3 = − b 1 + b 3 = 0 + b
dt 2 4 2 4 4 2 2

π2 *
*
αBC = 8 b 2 rad/s2

97
Alternatively,

*
d * * *
αBC = ω +ω
dt AB BC AB ×ωBC

d π* π* π* π* π*
    
= − b1 + b3 + − b1 × − b1 + b3
dt AB 2 4 2 2 4
| {z }
=0
π2 *
= b rad/s2
8 2

98
8.4.9
GOAL: Find the angular velocity of the central box C in a differential.
* *
GIVEN: v*A = −2k m/s, v*B = −3k m/s. r1 = 0.2 m, r2 = 0.3 m, d = 0.5 m.
DRAW:

FORMULATE EQUATIONS: We’ll use the equations for finding velocities on rotating bodies.
SOLVE: All bodies in the differential have two rotational degrees of freedom, which are in the * ı
*
and * directions. No rotation is allowed in the k direction, as both wheels remain in contact with
the ground. Let
*
ω A = ωA1 *ı + ωA2 * 
* * *
ω B = ω B1 ı + ω B 2 
*
ωC = ω C1 *
ı + ω C2 *
*
ωG3 = ωG31 *ı + ωG32 * 
* * *
ωG4 = ωG41 ı + ωG42 
By the geometric constraints, ωA2 = ωB2 = ωC2 , and ωG31 = ωG41 = ωC1 .
We begin by computing the angular velocities of wheels A and B from the given linear velocities
and the rolling constraint. v*A , v*B refer to the velocity of the centerpoints of the two wheels,
* *
respectively, and ωA , ωB refer to the angular velocity of the wheels themselves. The wheels have
two angular velocity components, one in the * ı direction and another in the *
 direction (due to the
lefthand turn).
v*A *
= ω *
A × r A/ v*B *
= ω *
B × r B/
G H
* *
vA k = (ωA1 *
ı + ωA2 *
 )×r2 *
 vB k = (ωB1 *
ı + ω B2 *
 )×r2 *

⇒ ωA1 = vA /r2 ⇒ ωB1 = vB /r2
The above yields:
−2 m/s
ωA1 = = −6.6̄ rad/s
0.3 m
−3 m/s
ω B1 = = −10 rad/s
0.3 m
To determine the * components of the wheel angular velocities we’ll use the relative motion of B
with respect to A (in this case point A can be considered an extended point of wheel B):
v*B = v*A + ω
* *
B × r B/
A
* *
vB k = vA k + (ωB1 ı + ωB2 *
*
 )×2d *
ı
−vB + vA 3 m/s − 2 m/s
⇒ ω B2 = = = 1.0 rad/s
2d 2(0.5 m)

99
Thus we have
ωC2 = ωB2 = ωA2 = 1.0 rad/s (1)
To find ωC1 we need to determine the rate at which gears G3 and G4 are rotating about axle AB.
Gears G3 and G4 roll against gears G1 and G2 , and it is this rolling, in addition to the rolling of
the wheels, that determines the *
ı component of ω*
C . Examining the relative velocities of the gears
with respect to each other and with respect to point O will allow us to find ωC1 .

v*F/ *
= ω *
C × r F/
O O

 )×r1 *
ı + ω C2 *
= (ωC1 * 
*
= ω C 1 r1 k
*
v*F/ = ωC1 r1 k (2)
O

Taking a different path to F from O:

v*F/ = v*E/ + v*F/


O O E

= [(ωA1 *
ı + ωA2 *
 )×(−r1 *
ı + r1 *
 )] + [(ωG31 *
ı + ωG32 *
 )×r1 *
ı]
*
= (ωA1 r1 + ωA2 r1 − ωG32 r1 )k
*
v*F/ = (ωA1 r1 + ωA2 r1 − ωG32 r1 )k (3)
O

Combining (2) and (3), we see that

ωC1 = ωA1 + ωA2 − ωG32 (4)

Taking yet a different path:

v*F/ = v*J/ + v*F/


O O J

= [(ωB1 *
ı + ω B2 *
 )×(r1 *
ı + r1 *
 )] + [(ωG31 *
ı + ωG32 *
 )×−r1 *
ı]
*
= (ωB1 r1 − ωB2 r1 + ωG32 r1 )k
*
v*F/ = (ωB1 r1 − ωB2 r1 + ωG32 r1 )k (5)
O

Combining (2) and (5), we see that

ωC1 = ωB1 − ωB2 + ωG32 (6)

Combining (4) and (6), with ωA2 = ωB2 , yields:


ωA1 + ωB1
ω C1 = (7)
2
Thus,
* * *
−6.6̄ rad/s − 10 rad/s *
ωC = ω C1 ı + ω C2  = ı + 1.0 *
 rad/s
2
* * *
ωC = −8.33 ı + 1.0  rad/s

100
8.4.10
GOAL: Find the angular velocity of gear G2 .
* *
GIVEN: v*A = 10k m/s, v*B = −10k m/s. r1 = 0.2 m, r2 = 0.3 m, d = 0.5 m.
DRAW:

FORMULATE EQUATIONS: We’ll use the equations for finding velocities on rotating bodies.
SOLVE: All bodies in the differential have two rotational degrees of freedom, which are in the * ı
*
and * directions. No rotation is allowed in the k direction, as both wheels remain in contact with
the ground. Let
*
ω A = ωA1 *ı + ωA2 * 
* * *
ω B = ω B1 ı + ω B 2 
*
ωC = ω C1 *
ı + ω C2 *
*
ωG3 = *
ωG31 ı + ωG32 * 
*
ωG4 = ωG41 *ı + ωG42 * 
By the geometric constraints, ωA2 = ωB2 = ωC2 , and ωG31 = ωG41 = ωC1 .
We begin by computing the angular velocities of wheels A and B from the given linear velocities
and the rolling constraint. v*A , v*B refer to the velocity of the centerpoints of the two wheels,
* *
respectively, and ωA , ωB refer to the angular velocity of the wheels themselves. The wheels have
two angular velocity components, one in the * ı direction and another in the *
 direction (due to the
lefthand turn).
v*A *
= ω *
A × r A/ v*B *
= ω *
B × r B/
G H
* *
vA k = (ωA1 *
ı + ωA2 *
 )×r2 *
 vB k = (ωB1 *
ı + ω B2 *
 )×r2 *

⇒ ωA1 = vA /r2 ⇒ ωB1 = vB /r2
The above yields:
10 m/s
ωA1 = = 33.3̄ rad/s
0.3 m
−10 m/s
ω B1 = = −33.3̄ rad/s
0.3 m
To determine the * components of the wheel angular velocities we’ll use the relative motion of B
with respect to A (in this case point A can be considered an extended point of wheel B):
v*B = v*A + ω
* *
B × r B/
A
* *
vB k = vA k + (ωB1 ı + ωB2 *
*
 )×2d *ı
−vB + vA 10 m/s + 10 m/s
⇒ ωB2 = = = 20.0 rad/s
2d 2(0.5 m)

101
Having both ωB1 and ωB2 , and knowing that wheel B is rigidly attached to gear G2 , we have
* * * * *
ωG2 = ω1 ı + ω2  = (−33.3 ı + 20.0  ) rad/s

102
8.4.11
GOAL: Find the angular velocity of Gear G3 .
* *
GIVEN: v*A = 10k m/s, v*B = 10k m/s. r1 = 0.2 m, r2 = 0.3 m, d = 0.5 m.
DRAW:

FORMULATE EQUATIONS: We’ll use the equations for finding velocities on rotating bodies.
SOLVE: All bodies in the differential have two rotational degrees of freedom, which are in the * ı
*
and * directions. No rotation is allowed in the k direction, as both wheels remain in contact with
the ground. Let
*
ω A = ωA1 *ı + ωA2 * 
* * *
ω B = ω B1 ı + ω B 2 
*
ωC = ω C1 *
ı + ω C2 *
*
ωG3 = ωG31 *ı + ωG32 * 
* * *
ωG4 = ωG41 ı + ωG42 
By the geometric constraints, ωA2 = ωB2 = ωC2 , and ωG31 = ωG41 = ωC1 .
We begin by computing the angular velocities of wheels A and B from the given linear velocities
and the rolling constraint. v*A , v*B refer to the velocity of the centerpoints of the two wheels,
* *
respectively, and ωA , ωB refer to the angular velocity of the wheels themselves. The wheels have
two angular velocity components, one in the * ı direction and another in the *
 direction (due to the
lefthand turn).
v*A *
= ω *
A × r A/ v*B *
= ω *
B × r B/
G H
* *
vA k = (ωA1 *
ı + ωA2 *
 )×r2 *
 vB k = (ωB1 *
ı + ω B2 *
 )×r2 *

⇒ ωA1 = vA /r2 ⇒ ωB1 = vB /r2
The above yields:
10 m/s
ωA1 = = 33.3̄ rad/s
0.3 m
10 m/s
ω B1 = = 33.3̄ rad/s
0.3 m
To determine the * components of the wheel angular velocities we’ll use the relative motion of B
with respect to A (in this case point A can be considered an extended point of wheel B):
v*B = v*A + ω
* *
B × r B/
A
* *
vB k = vA k + (ωB1 ı + ωB2 *
*
 )×2d *ı
−vB + vA 10 m/s − 10 m/s
⇒ ω B2 = = = 0
2d 2(0.5 m)

103
Thus we have
ωC2 = ωB2 = ωA2 = 0 (1)
From the figure it is clear that gear G3 will have the same angular velocity as the carrier C as well
as an angular velocity with respect to C. Gears G3 and G4 roll against gears G1 and G2 , and it
is this rolling, in addition to the rolling of the wheels, that determines the *ı component of ω *
C.
Examining the relative velocities of the gears with respect to each other and with respect to point
O will allow us to find ωC1 .

v*F/ *
= ω *
C × r F/
O O

 )×r1 *
ı + ω C2 *
= (ωC1 * 
*
= ω C 1 r1 k
*
v*F/ = ωC1 r1 k (2)
O

Taking a different path to F from O:

v*F/ = v*E/ + v*F/


O O E

= [(ωA1 *
ı + ωA2 *
 )×(−r1 *
ı + r1 *
 )] + [(ωG31 *
ı + ωG32 *
 )×r1 *
ı]
*
= (ωA1 r1 + ωA2 r1 − ωG32 r1 )k
*
v*F/ = (ωA1 r1 + ωA2 r1 − ωG32 r1 )k (3)
O

Combining (2) and (3), we see that

ωC1 = ωA1 + ωA2 − ωG32 (4)

Taking yet a different path:

v*F/ = v*J/ + v*F/


O O J

= [(ωB1 *
ı + ω B2 *
 )×(r1 *
ı + r1 *
 )] + [(ωG31 *
ı + ωG32 *
 )×−r1 *
ı]
*
= (ωB1 r1 − ωB2 r1 + ωG32 r1 )k
*
v*F/ = (ωB1 r1 − ωB2 r1 + ωG32 r1 )k (5)
O

Combining (2) and (5), we see that

ωC1 = ωB1 − ωB2 + ωG32 (6)

Combining (4) and (6), with ωA2 = ωB2 , yields:

ωA1 + ωB1 33.3 + 33.3


ω C1 = = rad/s = 33.3 rad/s (7)
2 2
Thus,
* * * *
ωC = ωC1 ı + ωC2  = 33.3 ı rad/s
We now have
* *
ωA = 33.3 ı rad/s (8)
* *
ωB = 33.3 ı rad/s (9)
* * *
ωG3 = 33.3 ı rad/s + φ̇  (10)

104
where φ̇ is the rotational speed of Gear G3 with respect to the carrier C.
We can now apply the formula relating the velocity of two points on a rigid body, the body in
question being gear G3 .
v*E = ω* *
A × r E/
G
= 33.3 *
ı rad/s×[(d − r1 ) *
ı + (r2 + r1 ) *
] (11)
*
= 33.3(r2 + r1 )k rad/s

v*J *
=ω *
B × r J/
H
= 33.3 *
ı rad/s×[−(d − r1 ) *
ı + (r2 + r1 ) *
] (12)
*
= 33.3(r2 + r1 )k rad/s
We’ll now apply
v*J = v*E + ω
* *
G3 × r J/ (13)
E

(11), (12), (13) ⇒


* *
33.3(r2 + r1 )k rad/s = 33.3(r2 + r1 )k rad/s + (33.3 *
ı rad/s + φ̇ *
 )×(2r1 *
ı)

φ̇ = 0 (14)
* *
(10), (14) ⇒ ωG3 = 33.3 ı rad/s

105
8.4.12
GOAL: Find the angular velocity of Gear G4 .
*
GIVEN: v*A = 0, v*B = −6k m/s. r1 = 0.2 m, r2 = 0.3 m, r3 = 0.3 m, d = 0.5 m.
DRAW:

FORMULATE EQUATIONS: We’ll use the equations for finding velocities on rotating bodies.
SOLVE: All bodies in the differential have two rotational degrees of freedom, which are in the * ı
*
and * directions. No rotation is allowed in the k direction, as both wheels remain in contact with
the ground. Let
*
ω A = ωA1 *ı + ωA2 * 
* * *
ω B = ω B1 ı + ω B 2 
*
ωC = ω C1 *
ı + ω C2 *
*
ωG3 = ωG31 *ı + ωG32 * 
* * *
ωG4 = ωG41 ı + ωG42 
By the geometric constraints, ωA2 = ωB2 = ωC2 , and ωG31 = ωG41 = ωC1 .
We begin by computing the angular velocities of wheels A and B from the given linear velocities
and the rolling constraint. v*A , v*B refer to the velocity of the centerpoints of the two wheels,
* *
respectively, and ωA , ωB refer to the angular velocity of the wheels themselves. The wheels have
two angular velocity components, one in the * ı direction and another in the *
 direction (due to the
lefthand turn).
v*A *
= ω *
A × r A/ v*B *
= ω *
B × r B/
G H
* *
vA k = (ωA1 *
ı + ωA2 *
 )×r2 *
 vB k = (ωB1 *
ı + ω B2 *
 )×r2 *

⇒ ωA1 = vA /r2 ⇒ ωB1 = vB /r2
The above yields:
0 m/s
ωA1 = =0
0.3 m
−6 m/s
ω B1 = = −20 rad/s
0.3 m
To determine the * components of the wheel angular velocities we’ll use the relative motion of B
with respect to A (in this case point A can be considered an extended point of wheel B):
v*B = v*A + ω
* *
B × r B/
A
* *
vB k = vA k + (ωB1 ı + ωB2 *
*
 )×2d *
ı
−vB + vA 6 m/s − 0
⇒ ω B2 = = = 6 rad/s
2d 2(0.5 m)

106
Thus we have
ωC2 = ωB2 = ωA2 = 6 rad/s (1)
From the figure it is clear that gear G4 will have the same angular velocity as the carrier C as well
as an angular velocity with respect to C. Gears G3 and G4 roll against gears G1 and G2 , and it
is this rolling, in addition to the rolling of the wheels, that determines the *ı component of ω *
C.
Examining the relative velocities of the gears with respect to each other and with respect to point
O will allow us to find ωC1 .

v*F/ *
= ω *
C × r F/
O O

 )×r1 *
ı + ω C2 *
= (ωC1 * 
*
= ω C 1 r1 k
*
v*F/ = ωC1 r1 k (2)
O

Taking a different path to F from O:

v*F/ = v*E/ + v*F/


O O E

= [(ωA1 *
ı + ωA2 *
 )×(−r1 *
ı + r1 *
 )] + [(ωG31 *
ı + ωG32 *
 )×r1 *
ı]
*
= (ωA1 r1 + ωA2 r1 − ωG32 r1 )k
*
v*F/ = (ωA1 r1 + ωA2 r1 − ωG32 r1 )k (3)
O

Combining (2) and (3), we see that

ωC1 = ωA1 + ωA2 − ωG32 (4)

Taking yet a different path:

v*F/ = v*J/ + v*F/


O O J

= [(ωB1 *
ı + ω B2 *
 )×(r1 *
ı + r1 *
 )] + [(ωG31 *
ı + ωG32 *
 )×−r1 *
ı]
*
= (ωB1 r1 − ωB2 r1 + ωG32 r1 )k
*
v*F/ = (ωB1 r1 − ωB2 r1 + ωG32 r1 )k (5)
O

Combining (2) and (5), we see that

ωC1 = ωB1 − ωB2 + ωG32 (6)

Combining (4) and (6), with ωA2 = ωB2 , yields:


ωA1 + ωB1 0 − 20
ω C1 = = rad/s = −10 rad/s
2 2
Thus,
* * * * *
ωC = ωC1 ı + ωC2  = (−10 ı + 6  ) rad/s
We now have
* *
ωA = 6  rad/s (7)
* * *
ωB = (−20 ı + 6  ) rad/s (8)
* * *
ωG4 = −10 ı rad/s + (6 rad/s + φ̇)  (9)

107
where φ̇ is the rotational speed of Gear G4 with respect to the carrier C.
We can now apply the formula relating the velocity of two points on a rigid body, the body in
question being gear G4 .
v*K = ω* *
A × r K/
G
= 6*
 rad/s×[(d − r1 ) *
ı + (r2 − r1 ) *
] (10)
*
= −6(d − r1 )k rad/s

v*L = ω
* *
B × r L/
H
= (−20 *
ı + 6*
 ) rad/s×[−(d − r1 ) *ı + (r2 − r1 ) *
] (11)
*
= [−20(r2 − r1 ) + 6(d − r1 )]k rad/s
We’ll now apply
v*L = v*K + ω
* *
G4 × r L/ (12)
K

(10), (11), (12) ⇒

* *
[−20(r2 − r1 ) + 6(d − r1 )]k rad/s = −6(d − r1 )k rad/s + [−10 rad/s *
ı + (6 rad/s + φ̇) *
 ]×(2r1 *
ı)

−20(0.1 m/s) + 6(0.3 m/s) = −6(0.3 m/s) − (0.4 m)(6 rad/s + φ̇)

4 rad/s = −0.4φ̇

φ̇ = −10 rad/s (13)


* * *
(9), (13) ⇒ ωG4 = (−10 ı − 4  ) rad/s

108
8.4.13
GOAL: Determine the highest velocity point on a rolling ice cream cone.
GIVEN: Cone dimensions. Time to make a full circle on the floor is 2 s.
DRAW:

FORMULATE EQUATIONS: We’ll use the equations for finding velocities on rotating bodies.
SOLVE: Example 8.4 shows how to determine the angular velocities of a rotating disk on the
end of a bent shaft. The current homework problem can be thought of as precisely the same
problem. The motion of OA will be the same as that of Example 8.4’s shaft. From observation
we can deduce that the maximal speed will be found at the point B, the topmost point of the
the pictured instant. From geometry we have β = sin−1 (0.25) = 14.48◦ . |AB| = 1 in and
cone at √
|OA| = 42 − 12 in = 3.873 in
The cone takes 2 s to complete one full rotation around the floor and thus we have

ω = = π rad/s
OA 2s
The correspondence between our problem and that of Example 8.4 is
ωcone → ωW
ω → ωS
AO
3.873 in → L1
1 in → L2
Substituting these values into the expression for ωW gives us
 
ωcone = (π rad/s) sin β − 3.873 cos1β + sin β * c 1 + (π rad/s) cos β *
c3
= (−11.78 *c 1 + 3.042 *
c 3 ) rad/s

v*B *
=ω *
cone × r B/
O
= (−11.78 *
c 1 + 3.042 *
c 3 ) rad/s×(3.873 *
c1 + *
c 3 ) in

v*B = 23.56 *
c 2 m/s

109
8.4.14
GOAL: Show that the angular velocity of a rolling wheel is oriented horizontally.
GIVEN: Wheel’s angular velocity in terms of unit vectors attached to the wheel.
DRAW:

* * *
b1 b2 b3
*
c1 cos β 0 sin β
*
c2 0 1 0
*
c3 − sin β 0 cos β
FORMULATE EQUATIONS:
" We’ll use the angular velocity
# found in the example:
*
L1
cos β + L2
sin β *
ωW = ωS sin β − c 1 + ωS cos β *
c3
L2
SOLVE: * *
Re-expressing the *
c 1, *
c 3 unit vectors in terms of b 1 , b 3 gives us
L cos β + L sin β
 
* 1 2 *
c 1 + ωS cos β *
ωW = ωS sin β − L2 c3
L
 
= ωS sin β − L1 cos β − sin β * c 1 + ωS cos β *
c3
2
L
 
= − L1 cos β * c 1 + cos β *
c 3 ωS
2 
L1 * L2 *

= LL cos β − L c 1 + L c 3 ωS
2
= − L cos β * *
L2 [cos β c 1 − sin β c 3 ] ωS
ω *
= − tanSβ b 1

110
8.4.15
GOAL: Find the acceleration of point D in the illustrated mechanism.
GIVEN: System geometry.
DRAW:

* * *
b1 b2 b3
*
c1 cos β 0 sin β
*
c2 0 1 0
*
c3 − sin β 0 cos β
ASSUME: The wheel rolls without slip.
FORMULATE EQUATIONS: D is attached to the bent shaft and thus we can view it as a
point on a rotating rigid body. We’ll use
v*D = ω
* *
S × r D/
O

where S indicates the shaft and then differentiate to find the acceleration.
SOLVE:
* * * *
v*D = ωS b 3 ×(L1 cos β b 1 + L1 sin β b 3 ) = ωS L1 cos β b 2
* d *
a*D = αS L1 cos β b 2 + ωS L1 cos β dt b2
* *
= αS L1 cos β b 2 + ωS L1 cos β(−ωS b 1 )
* *
= L1 cos β(αS b 2 − ωS2 b 1 )
= L1 cos β[αS *c 2 − ωS2 (cos β *
c 1 − sin β *
c 3 )])

a*D = L1 cos β(−ωS2 cos β * c 2 + ωS2 sin β *


c 1 + αS * c 3)

111
8.4.16
GOAL: Explain what the physical root is for the angular acceleration components of the disk in
Example 8.4.
GIVEN: Angular velocity and acceleration of the disk.
DRAW:

* * *
b1 b2 b3
*
c1 cos β 0 sin β
*
c2 0 1 0
*
c3 − sin β 0 cos β

* *
FORMULATE EQUATIONS: The problem statement involves b 1 and b 3 . We’ll start with the
angular velocity found in the example and re-express it in terms of these unit vectors. Then we’ll
differentiate to find the angular acceleration.
SOLVE:
*
ωW is given by " #
*
L1 cos β + L2 sin β *
ωW = ωS sin β − c 1 + ωS cos β *
c3
L2
* *
Re-expressing the *
c 1, *
c 3 unit vectors in terms of b 1 , b 3 gives us
L cos β + L sin β
 
* 1 2 *
c 1 + ωS cos β *
ωW = ωS sin β − L2 c3
L
 
= ωS sin β − L1 cos β − sin β * c 1 + ωS cos β *
c3
2
L
 
= − L1 cos β * c 1 + cos β *
c 3 ωS
2 
L1 * L2 *

= LL cos β − L c 1 + L c 3 ωS
2
= − L cos β * *
L2 [cos β c 1 − sin β c 3 ] ωS
ω *
= − tanSβ b 1

We can differentiate by viewing this as a vector in a rotating reference frame, the frame defined by
the bent shaft. Let S indicate a rotating frame of reference that rotates at the same rate as the
shaft.

112

*
d * d * * *
αW = ω W = ω +ωS ×ωW
dt dt W
N  S
d ω * ω *
  
*
= − tanSβ b 1 + ωS b 3 × − tanSβ b 1
dt
S
α * ω2 *
= − tanSβ b 1 − tanSβ b 2

Both terms depend upon the angular speed and acceleration of the shaft because the wheel’s
rotation is determined (through a rolling constraint) on the velocity of its center (which is driven
by one end of the rotating shaft).
α *
The first term, − tanSβ b 1 , arises when differentiating the angular velocity with respect to the rotat-
ing frame. Physically it corresponds to the “usual” way we’d expect to see an angular acceleration
arise, namely by the shaft’s rotation rate increasing (or decreasing) so as to create a non-zero
acceleration. If the shaft were to rotate at a constant rate this term would disappear.
ω2 * *
The second term, tanSβ b 2 , comes about because the angular velocity (which points in the − b 1
*
direction) is swept around in a circle as the shaft rotates. The rotation is positive in the b 3
*
direction (counter-clockwise when looking down at the system along the − b 3 direction) and this
*
rotation causes the angular velocity vector to rotate as well, its tip moving in the − b 2 direction.

113
8.4.17
GOAL: Find the acceleration of point C in the illustrated mechanism. L1 = 10 cm, β = 15◦ .
* *
ω shaft = 3 b 3 rad/s.
GIVEN: System geometry.
DRAW:

* * *
b1 b2 b3
*
c1 cos β 0 sin β
*
c2 0 1 0
*
c3 − sin β 0 cos β
ASSUME: The wheel rolls without slip.
FORMULATE EQUATIONS: We’ll start with the angular velocity of the wheel, as found in
* * * *
the example. After re-expressing this in terms of b 1 , b 2 , b 3 we’ll differentiate to find αW . Then, by
viewing the body as rotating about the fixed point O, we’ll determine the acceleration using our
rigid body acceleration formula  
a*C = α
* * * * *
W × r C/ + ωW × ωW × r C/ (1)
O O

SOLVE: " #
L cos β + L2 sin β *
*
ωW = ωS sin β − 1 c 1 + ωS cos β *
c3
L2
* *
Re-expressing the *
c 1, *
c 3 unit vectors in terms of b 1 , b 3 gives us
L cos β + L sin β
 
* 1 2 *
c 1 + ωS cos β *
ωW = ωS sin β − L2 c3
L
 
= ωS sin β − L1 cos β − sin β * c 1 + ωS cos β *
c3
2
L
 
= − L1 cos β * c 1 + cos β *
c 3 ωS
2 
L1 * L2 *

= LL cos β − L c 1 + L c 3 ωS
2
= − L cos β * *
L2 [cos β c 1 − sin β c 3 ] ωS

ω *
= − tanSβ b 1

αS * ω d* 1  * *

*
αW =− b1 − S b1 = − αS b 1 + ωS2 b 2
tan β tan β dt tan β
Using these expressions in (1) gives us

114
1  * ω * ω *
  
* * *
a*C = − αS b 1 + ωS2 b 2 ×L b 1 − S b 1 × − S b 1 ×L b 1
tan β tan β tan β
* *
Interestingly, the second term drops out ( b 1 × b 1 = 0), leaving us with
LωS2 *
a*C = b
tan β 3
From geometry we have
L = 10.35 cm
(0.1035 m)(3 rad/s)2 * *
a*C = ◦ b 3 = 3.48 b 3 m/s2
tan 15

115
8.4.18
GOAL: Find the velocity and acceleration of point A.
* * * * * *
GIVEN: System geometry. ω P = 2 b 3 rad/s, αP = 4 b 3 rad/s, ωgun/ = 3 b 1 rad/s.
plate
DRAW:

FORMULATE EQUATIONS:
v*A = ω
* *
gun × r A/ (1)
O
 
a*A = α
* * * * *
gun × r A/ + ωgun × ωgun × r A/ (2)
O O

SOLVE: The angular velocity of the gun is given by


* * *
ωgun = (2 b 3 + 3 b 1 ) rad/s (3)
* * * *
(1), (3) ⇒ v*A = (2 b 3 + 3 b 1 ) rad/s×(2 m)(cos 30◦ b 2 + sin 30◦ b 3 )
* * *
v*A = (−3.464 b 1 − 3 b 2 + 5.196 b 3 ) m/s

* * 2 * * *
αgun = 4 b 3 rad/s + (2 b 3 rad/s)×(2 b 3 + 3 b 1 ) rad/s

* * * 2
αgun = (4 b 3 + 6 b 2 ) rad/s (4)
* * * *
a*A = (6 b 2 + 4 b 3 ) rad/s2 ×(2 m)(cos 30◦ b 2 + sin 30◦ b 3 )
(2)–(4) ⇒ * * * * *
+(2 b 3 + 3 b 1 ) rad/s×(−3.464 b 1 − 3 b 2 + 5.196 b 3 )
* * *
a*A = (5.07 b 1 − 22.5 b 2 − 9.0 b 3 ) m/s2

116
8.4.19
GOAL: Find the rate β̇ at which the water cannon is pivoting up at a given instant (β = 10◦ ).
GIVEN: Base rotates about Z-axis at 4 rad/s, the magnitude of the tip B’s velocity is 6 m/s at
the instant when β = 10◦ .
DRAW: The figure below depicts the canon with the original X, Y, Z axes and a set of unit vectors
*
b i that are fixed to the rotating base.

*
ASSUME: We assume that the given rotation rate of the base is in the positive k direction. It
*
could just as easily be assumed that the rotation is in the negative k direction, without affecting
the magnitude of the tip’s velocity or the calculated value of β̇.
FORMULATE EQUATIONS: We’ll use the equation for velocity on a rotating body.
*
SOLVE: The rotation rate of the base is 4k rad/s, and that of the barrel with respect to the base
*
is β̇ b 1 . The total angular velocity of the barrel is thus:
* * * *
ω = 4k + β̇ b 1 = 4 b 3 + β̇ b 1
* *
as k = b 3 .
The velocity of the tip B is:

v*B *
= ω× r*B
/O
 * *
  * *

= 4 b 3 + β̇ b 1 × 1.2 cos 10◦ b 2 + 1.2 sin 10◦ b 3
* * *
= −4(1.2) cos 10◦ b 1 − 1.2β̇ sin 10◦ b 2 + 1.2β̇ cos 10◦ b 3 m/s

The magnitude of the velocity is (v*B · v*B )1/2 and is given as 6 m/s:
r 2  2  2
−4.8 cos 10◦ m/s + 1.2β̇ cos 10◦ m/s + 1.2β̇ sin 10◦ m/s
*
vB =
r 2  2
6 m/s = −4.8 cos 10◦ m/s + 1.2β̇ m/s

β̇ = 3.08 rad/s

117
8.4.20
GOAL: Determine the total angular acceleration of a model airplane propeller using two meth-
ods: a) differentiation of the components of the angular velocity vector, and b) the formula for
differentiating a vector in a rotating frame:

d * d
p = p* + ω×
*
p*
dt N
dt S
GIVEN: The airplane is attached to a tether and circles its attachment point O with a current
angular velocity of ω3 rad/s and an angular acceleration of α3 rad/s2 . The propeller turns with
angular velocity ω2 rad/s and angular acceleration α2 rad/s2 , both with respect to the model.
* * *
DRAW: The b 1 , b 2 , b 3 frame is attached to the plane.

FORMULATE EQUATIONS: We’ll use the equation for angular velocity and angular acceler-
ation of a rotating body.
SOLVE: The angular velocity of the propeller is the sum of the angular velocity of the plane and
the angular velocity of the propeller with respect to the plane:
* * *
ω = ω3 b 3 + ω2 b 2

a) Differentiating:
*
α = ˙ = ω̇ *
*
ω
*˙ * *˙
b + ω3b 3 + ω̇2 b 2 + ω2b 2
3 3
* * *
= α3 b 3 + ω3 (0) + α2 b 2 + ω2 (−ω3 b 1 )

* * * *
α = −ω2 ω3 b 1 + α2 b 2 + α3 b 3
*
b) Using formula for differentiation in the rotating frame B (with b i attached):

*
d * d * * *
α = ω = ω + ω ×ω
dt N dt B B

d * *
  *  * *

= ω3 b 3 + ω2 b 2 + ω3 b 3 × ω3 b 3 + ω2 b 2
dt B

* * *
= α3 b 3 + α2 b 2 − ω2 ω3 b 1

* * * *
α = −ω2 ω3 b 1 + α2 b 2 + α3 b 3

118
8.4.21
GOAL: Find the velocity and acceleration of point B.
* * * * * * 2 * *
GIVEN: System geometry. ω OA ≡ ω1 = 0.8 b 3 rad/s, αOA ≡ α1 = 2 b 3 rad/s , ωAB/ torso
≡ω2 =
* * * * 2
−1.6 b 1 rad/s, αAB/ ≡α2 = −0.5 b 1 rad/s ,
torso
DRAW:

FORMULATE EQUATIONS: We’ll use the general rigid body equations


v*B = v*A + ω×
*
r*B/ (1)
A
 
a*B = a*A + α×
*
r*B/ + ω×
* *
ω× r*B/ (2)
A A

SOLVE:
* * *
(1) ⇒ v*A = (0.8 b 3 ) rad/s×(0.24 b 1 m) = 0.192 b 2 m/s (3)
* * * *
(2) ⇒ a*A = 2 b 3 rad/s2 ×(0.24 b 1 m) + 0.8 b 3 rad/s×(0.192 b 2 m/s)
* * (4)
= (0.48 b 2 − 0.1536 b 1 ) m/s2
* * * *
(1), (3) ⇒ v*B = 0.192 b 2 m/s + (0.8 b 3 − 1.6 b 1 ) rad/s×(−0.24 b 3 m)
*
= (0.192 − 0.384) b 2 m/s
*
v*B = −0.192 b 2 m/s
* * * * * *
αAB =α 1 + α2 + ω1 ×(ω1 + ω2 )
* * 2 * * *
= (2 b 3 − 0.5 b 1 ) rad/s + (0.8 b 3 rad/s)×(0.8 b 3 rad/s − 1.6 b 1 rad/s)
* * *
= (−0.5 b 1 − 1.28 b 2 + 2.0 b 3 ) rad/s2
Using (1),(2) and (4) gives us
* * * *
a*B = a*A + (−0.5 b 1 − 1.28
h b 2 + 2.0 b 3 ) rad/s2 ×(−0.24ib 3 m)
* * * * *
+(0.8 b 3 − 1.6 b 1 )× (0.8 b 3 − 1.6 b 1 )×(−0.24 b 3 m)
* *
= (0.48 b 2 − 0.1536 b 1 ) m/s2
* *
+(−0.12 b 2 + 0.3072 b 1 ) m/s2
* * *
+(0.8 b 3 − 1.6 b 1 ) rad/s×(−0.384 b 2 m/s)
* * *
a*B = (0.461 b 1 + 0.36 b 2 + 0.614 b 3 ) m/s2

119
8.4.22
GOAL: Determine the velocity of a point T on the tip of a fan blade as a function of time.
GIVEN: Dimensions of fan and blade, tilt angle of fan, spin rate of blades, and the side to side
oscillation. *
DRAW: Let b i be the set of unit vectors attached to the rotating base of the fan; let unit vectors
* *
c i be the set attached to the tilt arm OA; and let di be a set attached to the blade itself.

* *
The transformations from b i to *
c i and from *
c i to di are:
* * * * * *
b1 b2 b3 c1 c2 c3
* *
c1 cos β sin β 0 d1 1 0 0
* *
c2 − sin β cos β 0 d2 0 cos ψ sin ψ
* *
c3 0 0 1 d3 0 − sin ψ cos ψ
ASSUME: We assume that the blades are spinning in the positive *
c 1 direction.
FORMULATE EQUATIONS: The velocity of a point T at the blade tip may be written as:
v*T = v*A + v*T = v*A + ω
*
× r*T + v*rel (1)
/A AT /A

SOLVE: Since the rotating frame used in the construction of (1) is that of the blade itself, v*rel = 0.
We have:
v*T = v*A + ω
*
× r*T (2)
AT /A
*
= ω × r*A + ω × r*T
*
(3)
OA /O AT /A

Let the length of arm OA be L1 = 8 in. and the length of a blade AT be L2 = 4 in. The angular
velocity of the blade is sum of the angular velocity of the arm and the angular velocity of the blade
with respect to the arm. From (3):
*
 *
 *
v*T = θ̇ b 2 ×L1 *
c 1 + θ̇ b 2 + ψ̇ *
c 1 ×L2 d2
= θ̇ (sin β *
c 1 + cos β *
c 2 )×L1 *
c1 +
h i
+ θ̇ (sin β *
c 1 + cos β *
c 2 ) + ψ̇ *
c 1 ×L2 (cos ψ *
c 2 + sin ψ *
c 3)
   
= −θ̇L1 cos β *
c 3 + θ̇ sin β + ψ̇ L2 cos ψ *
c 3 − sin ψ *
c 2 + θ̇L2 cos β sin ψ *
c1

120
Given θ = θ0 sin (ωt), taking the time derivative yields θ̇ = θ0 ω cos (ωt). Letting ψ0 = 0 such that
ψ = ψ̇t = 26t gives:
 
v*T = 4 θ0 ω cos (ωt) cos β sin (26t) *
c 1 − 4 θ0 ω cos (ωt) sin β + 104 sin (26t) *
c 2+
h  i
+ 4 θ0 ω cos (ωt) sin β + 104 cos (26t) − 8 θ0 ω cos (ωt) cos β *
c 3 in/s

121
8.4.23
GOAL: Find the acceleration of collar D.
GIVEN: Angular velocity of link OB, angular velocity of link BC with respect to OB, and velocity
at which collar D slides along link BC.
DRAW:

*
Unit vectors b i are fixed to link OB, while unit vectors *
c i are fixed to link BC. At the illustrated
instant, both sets of unit vectors and the X, Y, Z axes are aligned.
FORMULATE EQUATIONS: We’ll use theequationfor motion on a three-dimensional body:
a*D = a*B + α×
*
r*D + ω×
* *
ω× r*D + a*rel + 2ω×
*
v*rel (1)
/B /B

SOLVE: The vectors quantities



appearing

in Equation (1) are:
*
 * *
 *
a*B *
=ω *
× ω × r*B = θ̇ b 1 × θ̇ b 1 ×−0.5 b 3 = 0.5 θ̇2 b 3
OB OB /O
* * * *
ω =ω = θ̇ b 1 + φ̇ b 3
BC
* *
* db * db * *
α *
=ω*˙
= θ̈ b 1 + θ̇ dt1 + φ̈ b 3 + φ̇ dt3 = 0 + 0 + 0 − φ̇θ̇ b 2 = −φ̇θ̇ b 2
BC
v*rel =5* c 2 m/s
a*rel =0
r*D = 0.2 * c2 m
/B

The individual terms in (1), with θ̇ = 4 rad/s and φ̇ = 5 rad/s, are now:

122
* *
a*B = (0.5 m) (4 rad/s)2 b 3 = 8 b 3 m/s2

* *
α× r*D = −φ̇θ̇ b 2 ×0.2 *
c2 = 0
/B

    h  i  *
* * * *
*
ω× *
ω× r*D = θ̇ b 1 + φ̇ b 3 × c 2 = −0.2 θ̇2 + φ̇2 b 2
θ̇ b 1 + φ̇ b 3 ×0.2 *
/B
 * *
= − (0.2 m) (4 rad/s)2 + (5 rad/s)2 b 2 = −8.2 b 2 m/s2
 * *
 * *
*
2ω× v*rel = 2 θ̇ b 1 + φ̇ b 3 ×5 *
c 2 = 10θ̇ b 3 − 10φ̇ b 1
* *
 * *

= (10 m/s) (4 rad/s) b 3 − (10 m/s) (5 rad/s) b 1 = 40 b 3 − 50 b 1 m/s2

Substituting into (1):


* * * *
a*D = 8 b 3 + 0 − 8.2 b 2 + 0 + 40 b 3 − 50 b 1 m/s2
 * * *

a*D = −50 b 1 − 8.2 b 2 + 48 b 3 m/s2

123
8.5 Moments and Products of Inertia for a Three-Dimensional
Body

8.6 Parallel Axis Expressions for Inertias

124
8.6.1
GOAL:Determine the mass moments of inertia and products of inertia along the x, y, z axes for
the the illustrated body. Then calculate the inertias about axes parallel to x, y, z that go through
the body’s mass center. Express results in terms of the body’s mass m.
GIVEN: The dimensions of the body are illustrated in the figure; we assume a linear density ρ.
DRAW:

FORMULATE EQUATIONS:
First, we note the relationship between the mass and linear density of the body:
m
ρ= (1)
L1 + L2

Since moments and products of inertia are additive for composite bodies, we will treat the illustrated
body as two narrow rod segments and add the computed inertias of each segment to arrive at the
resultant value for the entire body.
SOLVE:

L L
1 2 L L
y 3 1 z 3 2
Z   Z Z
2 2 2 2
Ixx = y +z dm = y ρdy + z ρdz = ρ + ρ
3 0 3 0
Body 0 0
ρ 3 
= L1 + L32
3
L3 +L3
 
m 1 2
= 3 L +L
1 2

L L
1 2 L
z 3 2
Z   Z Z
2 2 2
Iyy = x +z dm = (0 + 0) dm + z ρdz = ρ
3 0
Body 0 0

L3
 
m 2
= 3 L +L
1 2

L L
1 2 L
y 3 1
Z   Z Z
2 2 2
Izz = x +y dm = y ρdy + (0 + 0) dm = ρ
3 0
Body 0 0

L3
 
m 1
= 3 L +L
1 2

The segment of length L1 has two planes of symmetry, the x−y plane and the y−z plane. Likewise,
the segment of length L2 has two planes of symmetry, the x−z plane and the y −z plane. Since
each segment has two planes of symmetry all of the products of inertia are zero. (Also, note that

125
since each segment is one dimensional, the product of dimensions appearing in the integral terms
must be zero.)
Ixy = Iyx = Ixz = Izx = Iyz = Izy = 0
Denote the segment of length L1 as mass m1 and the segment length L2 as mass m2 . The center
of mass of the body is located at:
     
L L L L *
m 1 − 1 * 2 2k
m1 r*G−1 + m2 r*G−2 L +L
1 2 2  +m L +L
1 2 2
r*G = =
m1 + m2 m
L1 2 L2 * 2
1 *
1
= −  + k
2 L1 + L2 2 L1 + L2
Let x0 , y 0 , z 0 be a set of coordinate axes through the center of mass G that are parallel to the
x, y, z axes, respectively. Using the parallel axis expressions for moments and products of inertia
we obtain:
 
I¯x0 x0 = Ixx − m r22 + r32
 !2 !2 
L13 + L32 2 2
!
m 1 L1 1 L2
= − m − + 
3 L1 + L2 2 L1 + L2 2 L1 + L2
h    i
m 1 1
= (L +L )2 12 L14 + L42 + 3 L1 L32 + L2 L13
1 2
 
I¯y0 y0 = Iyy − m r12 + r32
 !2 
L32 2
!
m 1 L2
= − m 0 + 
3 L1 + L2 2 L1 + L2
mL3  
1
= 2
(L +L )2 12 L2 + 13 L1
1 2
 
I¯z 0 z 0 = Izz − m r12 + r22
 !2 
L31 L21
!
m 1
= − m 0 + − 
3 L1 + L2 2 L1 + L2
mL3  
1
= 1
(L +L )2 12 L1 + 13 L2
1 2

I¯y0 z 0 = I¯z 0 y0 = Iyz − mr2 r3


2 2
! !
1 L1 1 L2
= 0−m −
2 L1 + L2 2 L1 + L2
 2
m L L
= 1 2
4 L +L
1 2

The center of mass of the body lies on the plane x = 0, and thus the y 0 , z 0 plane is also a plane of
symmetry. The coordinate axis normal to this plane is the x0 axis. Therefore all products of inertia
involving the x0 axis are zero:

I¯x0 y0 = I¯y0 x0 = I¯x0 z 0 = I¯z 0 x0 = 0

126
Alternatively, note that since the center of mass lies on the plane x = 0, r1 = 0.

127
8.6.2
GOAL:Determine the mass moments of inertia and products of inertia along the x, y, z axes for
the the illustrated body. Express results in terms of the body’s mass m.
GIVEN: The dimensions of the body are illustrated in the figure. We assume a linear density ρ.
DRAW:

FORMULATE EQUATIONS:
First, we note the relationship between the mass and linear density of the body:
m = 3aρ
Because moments and products of inertia are additive for composite bodies, we will treat the
illustrated body as three narrow rod segments and add the computed inertias of each segment to
arrive at the resultant value for the entire body.
SOLVE:
a2 7ρa3
 2
a
Ixx = (ρa) + 2(ρa) =
12 2 12
(ρa)a2 2ρa3
Iyy = 2 =
3 3
"  2 !#
(ρa)a2 (ρa)a2 5ρa3
 2
a a
Izz = +2 + (ρa) + =
12 12 2 2 4

a a ρa3
  
Ixy = 2(ρa) = , Iyz = Izx = 0
2 2 2
Expressing these in terms of the overall mass m gives us
2
Ixx = 7ma 2ma2 5ma2 ma2
36 , Iyy = 9 , Izz = 12 , Ixy = 6 , Iyz = Izx = 0

128
8.6.3
GOAL: Determine the rotational inertias of the illustrated flat plate.
GIVEN: Body’s orientation and dimensions.
DRAW:

FORMULATE EQUATIONS: From Appendix B we have


ma2 mb2 m(a2 + b2 )
Ix0 x0 = , Iy0 y0 = , Iz 0 z 0 =
12 12 12
Ix0 y0 = Iy0 z 0 = Iz 0 x0 = 0
SOLVE:
To find the mass moments of inertia and products of inertia we use (8.24)-(8.29) with r1 = b/2,
r2 = a/2 and r3 = 0.
ma2
Ixx = Ix0 x0 + m(r22 + r32 ) =
3
mb2
Iyy = Iy0 y0 + m(r32 + r12 ) =
3
Izz = Iz 0 z 0 + m(r12 + r22 )

m(a2 + b2 )
     2 
2
+ b2
= m a 12 +m b 2
+ a =
2 2 3
ab
 
Ixy = Ix0 y0 + mr1 r2 = m
4

Iyz = Izx = 0

2 2 2
Ixx = ma , I = mb2 , I = m(a + b ) , I = mab , I = I = 0 ,
3 yy 3 xx 3 xy 4 yz zx

129
8.6.4
GOAL: Determine the moments and products of inertia along the x, y, z axes for the the illustrated
triangular body. Then calculate the inertias about axes parallel to x, y, z that go through the body’s
mass center. Express results in terms of the areal density ρ.
GIVEN: The dimensions of the body are illustrated in the figure.
DRAW:

SOLVE: The borders of the triangular region are defined by the lines: x = 0, y = 0, and x =
a(1 − y/b). The moments and products of inertia are computed by integrating over this region:

Z   Z bZ a(1−y/b)
 
2 2
Ixx = y +z dm = y 2 + 0 ρ dx dy
Body 0 0
b
y
Z  
= ρ a 1− y 2 dy
b
0
!
b3 b4
= ρa −
3 4b
1 3
= 12 ρab

Iyy may be found similarly, by reversing the order of integration in x and y and using integration
1
limits of 0 and b(1 − x/a) on y. By the symmetry of the problem we should find that Iyy = 12 ρa3 b.
However, let’s proceed by maintaining the order above and performing the integration:

Z   Z bZ a(1−y/b)
 
2 2
Iyy = x +z dm = x2 + 0 ρ dx dy
Body 0 0
b
y 3
a3 1 −
Z
b
= ρ dy
3
0
4 b
ρa3 b y

= − 1−


12 b
0
1 3
= 12 ρa b

For Izz :
Z   Z bZ a(1−y/b)
 
2 2
Izz = x +y dm = x2 + y 2 ρ dx dy
Body 0 0

130
Notice that this is just the sum of Ixx and Iyy (this is true for all planar figures). Thus,
1
ρ ab3 + a3 b

Izz = Ixx + Iyy = 12

Since the x − y plane is a plane of symmetry for the body, all products of inertia involving the
normal z-axis are zero:
Ixz = Izx = Iyz = Izy = 0
The remaining products of inertia are:
Z Z bZ a(1−y/b)
Ixy = Iyx = xy dm = xy ρ dx dy
Body 0 0
b
y 2
a2 1 −
Z
b
= ρ y dy
2
0
!
a2 b2 2b2 b2
= ρ − +
2 2 3 4
1 2 2
= 24 ρa b

To determine the moments and products of inertia about the center of mass, we first need to
determine its location.
Z bZ a(1−y/b)
1 2
Z
r*G = r* dm = (x *
ı + y*
 ) ρ dx dy
m ρab
Body 0 0
b 2 !
2 a2 y y
Z   
*
= 1− ı +a 1− y*
 dy
ab 2 b b
0
 !3 ! b
2  a2 b y *
y 2 y 3 *
= − 1− ı +a − 
ab 6 b 2 3b
0
1 * 
= a ı + b*
3
Using our parallel axis expressions for inertias, with r1 = a/3, r2 = b/3, and r3 = 0 we have:
!
  1 ρab b2
I¯x0 x0 = Ixx − m r22 + r32 = ρab3 − = 1
36 ρab3
12 2 9
!
  1 3 ρab a2
I¯y0 y0 = Iyy − m r12 + r32 = ρa b − = 1
36 ρa3 b
12 2 9
 
I¯z 0 z 0 = Izz − m r12 + r22 I¯x0 x0 + I¯y0 y0 1
ρ ab3 + a3 b

= = 36
! !
1 2 2 ρab a b
I¯y0 x0 = I¯x0 y0 = Ixy − mr1 r2 = ρa b − = 1
− 72 ρa2 b2
24 2 3 3

Due to the fact that r3 = 0, or the fact that the z 0 -axis is normal to the plane of symmetry through
the mass center, all other products of inertia are zero.

I¯x0 z 0 = I¯z 0 x0 = I¯y0 z 0 = I¯z 0 y0 = 0

131
8.6.5
GOAL: Determine the rotational inertias of the illustrated flat plate.
GIVEN: Body’s orientation and dimensions.
DRAW:

FORMULATE EQUATIONS: We’ll find the solution by taking the difference of the rotational
inertias corresponding to a plate with dimensions a2 , b2 and one corresponding to the “missing
chunk” having dimensions a1 , b1 . Each plate will have the same density ρ. From Appendix B we
have, for a plate with dimensions a, b:
mb2 ma2 m(a2 + b2 )
Ix0 x0 = , Iy0 y0 = , Iz 0 z 0 =
12 12 12
Ix0 y0 = Iy0 z 0 = Iz 0 x0 = 0

SOLVE: We’ll start with a complete plate (a2 , b2 ) and put a “2” to the upper left of the I as an
identifier. To find the mass moments of inertia and products of inertia we use (8.24)-(8.29) with
r1 = a2 /2, r2 = b2 /2 and r3 = 0.
2 2 2 2
m2 b22
Ixx = Ix0 x0 + m2 (r2 + r3 ) =
3

2 2
m2 a22
Iyy = Iy0 y0 + m2 (r32 + r12 ) =
3
2I = 2I + m2 (r12 + r22 )
zz z0 z0

a22 + b22 m2 (a22 + b22 )


! " 2 #
b2  a 2
2
= m2 12 + m2 2 + 2 = 3

a2 b2
 
2 2
Ixy = Ix0 y0 + m2 r1 r2 = m2
4
2 2
Iyz = Izx = 0
Next we’ll consider the a plate corresponding to the “missing chunk” (a1 , b1 ) and put a “1” to the
upper left of the I as an identifier. To find the mass moments of inertia and products of inertia we
use (8.24)-(8.29) with r1 = a1 /2, r2 = b1 /2 and r3 = 0.
1 1 2 2
m1 b21
Ixx = Ix0 x0 + m1 (r2 + r3 ) =
3

1 1
m1 a21
Iyy = Iy0 y0 + m1 (r32 + r12 ) =
3

132
1I = 1I + m1 (r12 + r22 )
zz z0 z0

a21 + b21 m1 (a21 + b21 )


! " 2 #
b1  a 2
1
= m1 12 + m1 2 + 2 = 3

a1 b1
 
1 1
Ixy = Ix0 y0 + m1 r1 r2 = m1
4
1 1
Iyz = Izx = 0
We can now take the difference of the two sets of rotational inertias, using m1 = ρa1 b1 and
m2 = ρb1 b2 to obtain
ρa2 b32 ρa1 b31
Ixx = −
3 3
ρb2 a32 ρb1 a31
Iyy = −
3 3
ρ(a32 b2 + a2 b32 ) ρ(a31 b1 + a1 b31 )
Izz = −
3 3
ρa22 b22 ρa21 b21
Ixy = −
4 4
Finally, expressing these in terms of the system mass m = ρ(a2 b2 − a1 b1 ) gives us
h i
m(a2 b32 − a1 b31 ) m(b2 a32 − b1 a31 ) m a2 b2 (a22 + b22 ) − a1 b1 (a21 + b21 )
Ixx = Iyy = Izz =
3(a2 b2 − a1 b1 ) 3(a2 b2 − a1 b1 ) 3(a2 b2 − a1 b1 )

m(a2 b2 + a1 b1 )
Ixy = 4 Iyz = Izx = 0

133
8.6.6
GOAL: Determine the mass moments of inertia and products of inertia along the x, y, z axes for
the illustrated body. Express answers in terms of the body’s areal density ρ.
GIVEN: Dimension of the body are given in the figure.
DRAW:

FORMULATE EQUATIONS: To determine the inertia values of the composite body, we will
model it as a triangular region with a quarter-circular region removed. Since inertias are additive,
we may obtain the overall inertia values by subtracting those of the quarter-circular region from
those of the triangular region.
* * *
I = I triangle − I q.circle
SOLVE: The triangular region is bounded by the lines x = 0, y = 0, and x = b − y. The moments
and products of inertia are determined by integrating over this area:

Z   Z Zb b−y 
2 2
4
Ixx = y +z dm = y 2 + 0 ρ dx dy
Body 0 0

Z b

= ρ (b − y) y 2 dy
0
!
b4 b4
= ρ −
3 4
1 4
= ρb
12
By symmetry,
4 4 1 4
Iyy = Ixx = ρb
12
4
Computing Izz , noting that z 2 = 0 over the region,
Z  
4
Izz = x2 + y 2 dm = Iyy
4 4
+ Ixx
Body

1 4
= ρb
6
Since the x − y plane is a plane of symmetry for the body, all products of inertia involving the
normal z-axis are zero:
4 4 4 4
Ixz = Izx = Iyz = Izy = 0

134
4 4
Computing Ixy = Iyx for the triangular region,

Z Z Zb b−y
4 4
Ixy = Iyx = xy dm = xyρ dx dy
Body 0 0
b
(b − y)2
Z
= ρ y dy
2
0
!
b4 b 4 b 4
= ρ − +
4 3 8
1 4
= ρb
24

The quarter-circular area may be defined using polar r, θ coordinates, with x = r cos θ, y = r sin θ,
and r2 = x2 + y 2 . The bounds on the region are 0 ≤ r ≤ a and 0 ≤ θ ≤ π/2. In r, θ coordinates,
the differential area may be written as dA = dr(rdθ), thus the differential mass unit becomes
dm = ρ r dr dθ.

π
Z  Z 2Z a


Ixx = 2
y +z 2
dm = (r sin θ)2 ρ r dr dθ
Body 0 0
π
2
a4
Z
= ρ sin2 θ dθ
4
0
π
a4 2
= ρ (θ − sin θ cos θ)
8 0
1 4
= ρπa
16

By symmetry,
◦ ◦ 1
Iyy = Ixx = ρπa4
16
◦ ,
Now for Izz

π
Z  Z 2Z a

◦ 2 2
Izz = x +y dm = r2 ρ r dr dθ
Body 0 0
π
2
a4
Z
= ρ dθ
4
0

1
= ρπa4
8

The x−y plane is a plane of symmetry and thus all moments of inertia involving the normal z-axis
are zero:
◦ ◦ ◦ ◦
Ixz = Izx = Iyz = Izy = 0

135
The remaining products of inertia are:
π
Z Z 2Z a
◦ ◦
Ixy = Iyx = xy dm = r2 sin θ cos θρ r dr dθ
Body 0 0
π
2
a4
Z
= ρ sin θ cos θ dθ
4
0
π
a4 2
= ρ sin2 θ
8 0
1 4
= ρa
8
The moments and products of inertia of the composite body are obtained by subtracting the inertias
of the quarter-circular region from the respective inertias of the triangular region:

4 ◦ 1 4 1 4
Ixx = Ixx − Ixx = 12 ρb − 16 ρπa
4 ◦ 1 4 1 4
Iyy = Iyy − Iyy = 12 ρb − 16 ρπa
◦ 1 4
4
Izz = Izz − Izz = 6 ρb − 18 ρπa4
◦ 1
4
Ixy = Ixy − Ixy = 24 ρb
4 − 18 ρa4
1
Iyx = Ixy = 24 ρb
4 − 18 ρa4

By symmetry, all products of inertia involving the z-axis are zero:

Ixz = Izx = Iyz = Izy = 0

136
8.6.7
GOAL: Determine the rotational inertias of the illustrated body.
GIVEN: Body’s orientation. a = 0.1 m, b = 0.25 m, c = 0.15 m, d = 0.1 m, e = 0.1 m, f = 0.2 m,
g = 0.15 m. Body’s density is ρ and mass is m.
DRAW:

FORMULATE EQUATIONS: We’ll solve this problem by breaking it up into two rectangular
bodies, one with dimensions a, b, c and the other with dimensions c, e, g. From Appendix B we
have: ! ! !
c2 + b2 a2 + c2 a2 + b2
Ix0 x0 = m , Iy0 y0 = m , Iz 0 z 0 = m
12 12 12
To find the mass moments of inertia and products of inertia we use (8.24)-(8.29) with appropriate
values of r1 , r2 and r3 .
SOLVE: Let’s first consider the body with dimensions a, b, c. We’ll put a “1” to the upper left of
the I as an identifier.
For this body r1 = a/2, r2 = b/2 and r3 = −c/2.
1I 1 2 2
xx = Ix0 x0 + m1 (r2 + r3 )

     
= m1 c2 + b2 + m1 c2 + b2 = m1 c2 + b2
12 4 3
1I = 1I + m1 (r32 + r12 )
yy y0 y0
     
= m1 a2 + c2 + m1 a2 + c2 = m1 a2 + c2
12 4 3
1I = 1I + m1 (r12 + r22 )
zz z0 z0
     
= m1 a2 + b2 + m1 a2 + b2 = m1 a2 + b2
12 4 3
ab
 
1 1
Ixy = Ix0 y0 + m1 r1 r2 = m1
4
cb
 
1 1
Iyz = Iy0 z 0 + m1 r2 r3 = −m1
4
ac
 
1 1
Izx = Iz 0 x0 + m1 r3 r1 = −m1
4
Now we’ll consider the body with dimensions c, e, g. We’ll put a “2” to the upper left of the I as

137
an identifier.
For this body r1 = a + e/2, r2 = d + g/2 and r3 = −c/2.
2I 2 2 2
xx = Ix0 x0 + m2 (r2 + r3 )

c2 + g 2
    
2
= m2 + m2 c + d+ g 2
12 2 2

2I = 2I + m2 (r32 + r12 )
yy y0 y0
    
= m2 e2 + c2 + m2
2
c + a+ e 2

12 2 2

2I = 2I + m2 (r12 + r22 )
zz z0 z0

e2 + g 2
   2 
d + g2 e 2

= m2 12 + m2 + a+ 2

e g
  
2 2
Ixy = Ix0 y0 + m2 r1 r2 = m2 a + d+
2 2
g c
  
2 2
Iyz = Iy0 z 0 + m2 r2 r3 = m2 d + −
2 2
c e
  
2 2
Izx = Iz 0 x0 + m2 r3 r1 = m2 − a+
2 2
The individual masses are given by
m1 = 3.75×10−3 ρ, m2 = 2.25×10−3 ρ
Adding the rotational inertias together and evaluating for the given dimensional values gives us
Ixx = 1
Ixx + 2 Ixx = (1.96×10−4 m5 )ρ

Iyy = 1
Iyy + 2 Iyy = (1.1×10−4 m5 )ρ

Izz = 1
Izz + 2 Izz = (2.16×10−4 m5 )ρ

Ixy = 1
Ixy + 2 Ixy = (8.25×10−5 m5 )ρ

Iyz = 1
Iyz + 2 Iyz = (−6.469×10−5 m5 )ρ

Izx = 1
Izx + 2 Izx = (−3.94×10−5 m5 )ρ
Finally, evaluating in terms of the object’s mass, m = (abc + ceg)ρ = (6.0×10−3 m3 )ρ, gives us our
final results:
Ixx = (3.27×10−2 m2 )m, Iyy = (1.83×10−2 m2 )m, Izz = (3.60×10−2 m2 )m

Ixy = (1.375×10−2 m2 )m, Iyz = (−1.078×10−2 m2 )m, Izx = (−6.56×10−3 m2 )m

138
8.6.8
GOAL: Determine the mass moments of inertia and products of inertia for the illustrated washer
along the x, y, z axes (which go through the body’s mass center). Express the results in terms of ρ.
GIVEN: The outer radius of the washer is ro = 0.03 m, the inner radius is ri = 0.02 m, and the
thickness is h = 0.006 m.
DRAW:

SOLVE: We may compute the inertias in cylindrical r, θ, z coordinates, with x = r cos θ, y = r sin θ,
and x2 + y 2 = r2 . First, we’ll compute Izz .
h
R 2 R2π Rro
x2 y2 r2 ρ r dr dθ dz
R 
Izz = + dm =
Body −h
2
0 r
i
h
r4
 
R 2 R2π ro4 i
=ρ 4 − 4 dθ dz
−h
2
0

r4
 
ro4 i
= 2πρh 4 − 4
(0.03 m)4 (0.02 m)4
 
= 2πρ (0.006 m) 4 − 4

Izz = 6.13×10−9 m5 ρ


By symmetry, Ixx = Iyy , so we will only need to calculate Ixx :

h
Z   Z roZ 2πZ 2 
Ixx = y 2 + z 2 dm = r2 sin2 θ + z 2 ρ r dz dθ dr
Body r 0 −h
i 2
ro 2π !
h2
Z Z
3 2
= ρh r sin θ + r dθ dr
12
r 0
i
ro
2π !
r3 h2
Z 
= ρh (θ − sin θ cos θ) + 2πr dr
2 0 12
r
i
Z ro !
3 h2
= πρh r +r dr
6
r
i
1   1  
= πρh ro4 − ri4 + πρh3 ro2 − ri2
4 12
1  
= πρ (0.006 m) (0.03 m)4 − (0.02 m)4 +
4
1  
+ πρ (0.006 m)3 (0.03 m)2 − (0.02 m)2
12

139
Iyy = Ixx = 3.09×10−9 m5 ρ


Note that when perfoming the above integrations we could have substituted the lower limit ri with
0, to find the moments of inertia of a solid disk. Subtracting the inertia values of a solid disk of
radius ri from those of a solid disk of radius ro yields identical results.
Since all three planes, x−y, x−z, and y−z, are planes of symmetry for the body, all of the products
of inertia are zero:
Ixy = Iyx = Ixz = Izx = Iyz = Izy = 0

140
8.6.9
GOAL: Determine the mass moments and products of inertia for the illustrated triangular prism
GIVEN: Body’s orientation and dimensions.
DRAW:

FORMULATE EQUATIONS: From Appendix B we have the rotational inertias about the
body’s mass center G of a flat right-triangle:
mb2 ma2 m(a2 + b2 )
Ix0 x0 = , Iy0 y0 = , Iz 0 z 0 =
18 18 18
mab
Ix0 y0 = , Iy0 z 0 = Iz 0 x0 = 0
36
To find the mass moments of inertia and products of inertia we’ll use (8.24)-(8.29) with r1 = a 3,
r2 = 3b and r3 = 2c after determining the rotational inertias of a solid prism from those of the flat
right-triangle.
SOLVE:
The mass of a thin slice having thickness dz is ρab2 dz .
Rc ρab3 Rc b2
   2   
Ixx = ρab 2 b dz = ρab 2
0
36 + 2 z + 3 2 0 6 + z dz
b2 z + z 3 c
 
= ρab
2 6 3 0
   
b2 + c2
Ixx = ρabc 12
2 2
= m b6 + c3
6

From symmetry we have


   
2
Iyy c2
= ρabc b6 + 12
2 2
= m b3 + c6
! ! !
a2 b2 a2 b2 a2 b2
Izz =m + +m + =m +
18 18 9 9 6 6
   
2 2
Izz = m a6 + b6 a2 + b2
= ρabc 12 12

a b mab mab mab


  
Ixy = Ix0 y0 + m =− + =
3 3 36 9 12

Ixy = mab ρa2 b2 c


12 = 24

141
b c
  
Iyz = m
3 2

Iyz = mbc = ρab2 c2


6 12
From symmetry we have

Izx = mac = ρa2 bc2


6 12

142
8.6.10
GOAL: Determine the mass moments and products of inertia for the illustrated cylindrical body.
GIVEN: Body’s orientation and dimensions.
DRAW:

FORMULATE EQUATIONS: From Appendix B we have the rotational inertias about the
body’s mass center G:
mr2 m(3r2 + h2 )
Iy0 y0 = , Ix0 x0 =
2 12
Ix0 y0 = Iy0 z 0 = Iz 0 x0 = 0
SOLVE:
To find the mass moments of inertia and products of inertia we use (8.24)-(8.29) with r1 = 0,
r2 = h
2 and r3 = r.
2
Iyy = Iy0 y0 + mr2 = 3mr
2
  2  2
Ixx = Ix0 x0 + m r2 + h = mh 5mr2
2 3 + 4

From symmetry we see that Ixy = 0 .


The mass of the body is given by
m = ρπr2 h
giving us
2 3 4
Ixx = ρπr3 h + 5ρπr h , I = 3ρπr4 h
4 yy 2

Ixy = 0

143
8.6.11
GOAL: Determine Ixx , Ixy , and Izz , for the illustrated body. Express the results in terms of both
the areal density ρ and the body’s mass m. Denote the mass of each half-circular piece by mhc and
the mass of the rectangular piece by mr .
GIVEN: The body’s dimensions are given in the figure.
DRAW:

FORMULATE EQUATIONS: We’ll use the rotational inertia formulas for a half-circle and flat
rectangular body from Appendix B and then apply the parallel axis theorem.
SOLVE: We’ll first look at Body A (the left half-circle) then B (the right half-circle) and finally
the rectangular piece C that joins the two half-circles.
Body A:
 2
A 1 b 1
Ix00 x00 = mhc = mhc b2
2 2 8
d is the distance from the y axis to the body’s mass center.
Using the parallel axis theorem gives us
A A
Ix0 x0 = Ix00 x00 − mhc d2
and finally
 2 !  2
A A 2 b 1 b 3
Ixx = Ix0 x0 + mhc d + = mhc b2 + mhc = mhc b2
2 8 2 8
 2  2  2
A A b 1 b b 5
Izz = Iz 0 z 0 + mhc = mhc + mhc = m b2
2 4 2 2 16 hc
Because Body A lies in the y,z plane Ixy = 0.
Body B :
We’ll again use x0 , y 0 , z 0 to refer to axes through that body’s mass center. From observation we
have the same value for B Ixx as for Body A:
B 3
Ixx = mhc b2
8

144
  
2
BI = BI + mhc b + a2
zz z0 z0 2
  
2
= 1 2 b a2
16 mhc b + mhc 2 +

5 2
= 16 mhc b + mhc a2
b mhc ab
 
B
Ixy = mhc a =
2 2
Body C:
C
Ixx = mr b2
C 1
Izz = mr (a2 + b2 )
3
a b m ab
  
C
Ixy = mr = r
2 2 4
The masses of the individual pieces are given by
mhc = (0.1 m)2 π/2 = (1.57×10−2 m2 )ρ
mr = (0.4 m)(0.1 m)ρ = (8.00×10−2 m2 )ρ

Ixx = A Ixx + B Ixx + C Ixx


= (1.57×10−2 m2 )ρ(2) 83 (0.2 m)2 + (8.00×10−2 m2 )ρ(0.2 m)2
 

3.67×10−3 m5 ρ

=

Izz = A Izz + B Izz + ChIzz  


5 (0.2 m)2 + (0.4 m)2 + 1 (8.00×10−2 m2 )ρ (0.4 m)2 + (0.2 m)2 
i
= (1.57×10−2 m2 )ρ (2) 16 3
8.24×10−3 m5 ρ

=

Ixy = A Ixy + B Ixy + C Ixy


= (1.57×10−2 m2 )ρ (0.2 m)(0.4
2
m) + (8.00×10−2 m2 )ρ (0.2 m)(0.4 m)
4
2.23×10−3 m5 ρ

=
The total mass of the body is given by
mt = mr + 2mhc = (0.111 m2 )ρ

Ixx = 3.30×10−2 mt Izz = 7.40×10−2 mt Ixy = 2.00×10−2 mt

145
8.6.12
GOAL: Determine Ixx , Iyy , Izz , and Ixy for the illustrated body. Express the results in terms of
both the areal density ρ and the body’s mass mc . Denote the mass of each half circular piece by
m = 21 mc
GIVEN: The body’s dimensions are given in the figure.
DRAW:

SOLVE: To determine the moments and products of inertia for the composite body, we will require
some inertia values for a half-circular body. Consider a circle with normal axis z 0 through its mass
center. Since the moment of inertia of a full circle with mass mc about an axis through its mass
center and in the plane of the circle is 14 mc r2 , that of a half circle through the same point is:
1 1 1 1 1
   
Iparallel = mc r2 = (2m) r2 = mr2
2 4 2 4 4
The moment of inertia of a full circle about an axis through its mass center and normal to the
plane of the circle is 12 mc r2 , thus for a half circle through the same point:
1 1 1 1 1
   
Inormal = mc r2 = (2m) r2 = mr2
2 2 2 2 2
By symmetry, Ixx = Izz for the composite body. Thus we will only calculate Izz . Using the parallel
axis expressions of inertia for half circle A we obtain:
 
A
Izz = I¯A0 0 + m r2 + d2 = I A00 + mr2
zz z z 00
1 2
= mr + mr2
2
3 2
= mr
2
For half circle B:
B
Izz = I¯B0 0 + mr2
zz
1 2
= mr + mr2
4
5 2
= mr
4

146
Thus, for the composite body,
A B 11 2
Izz = Izz + Izz = mr
4

Ixx = Izz = 11 2 11
8 mc r = 8 ρπr
4

The moment of inertia about the y-axis is the same for both half circles. Iyy is therefore quite
straightforward to compute,

A B 1 1 1
Iyy = Iyy + Iyy = mr2 + mr2 = mr2
4 4 2

Iyy = 14 mc r2 = 14 ρπr4
B , will be zero since the y−z plane is a plane of symmetry for half circle
The product of inertia, Ixy
B (with normal x-axis). The product of inertia is then Ixy = Ixy A . One method of obtaining I A
xy
is to integrate over the half circle A using integration parameters x, y and the bounds x = 0 and
x2 + (y − r)2 = r2 . (⇒ x = 0 and x2 = 2yr − y 2 )

Z Z 2rZ 2yr−y 2
A
Ixy = xy dm = xyρ dx dy
Body 0 0

Z 2r
2yr − y 2
= ρ y dy
2
0
!
8r4
= ρ − 2r4
3
2 4
= ρr
3
Thus,
A = 2 ρr 4 = 2 m r 2
Ixy = Ixy 3 3π c

147
8.6.13
GOAL: Determine the Ixx , Iyy , Izz and Ixy for the illustrated body.
GIVEN: Body’s orientation and shape. a = 0.1 m, b = 0.02 m, c = 0.07 m, d = 0.06 m. The
density of the large block is 1000 kg /m3 and the density of the small block is 800 kg /m3 .
DRAW:

FORMULATE EQUATIONS: We’ll solve this problem by breaking it up into two rectangular
bodies, one with dimensions a, a, b and the other with dimensions e, e, d, where e = a − c and is
introduced for convenience. e = 0.03
! m. From Appendix B! we have: !
2 2
a +b a2 + b2 a2 + a2
Ix0 x0 = m , Iy0 y0 = m , Iz 0 z 0 = m
12 12 12
for the large block and ! ! !
d2 + e2 d2 + e2 e2 + e2
Ix0 x0 =m , Iy0 y0 =m , Iz 0 z 0 =m
12 12 12
for the small block.
To find the mass moments of inertia and products of inertia we use (8.24)-(8.29) with appropriate
values of r1 , r2 and r3 .
SOLVE: Let’s first consider the body with dimensions a, a, b. We’ll put a “1” to the upper left of
the I as an identifier.
For this body r1 = a/2, r2 = a/2 and r3 = b/2.
1I 1 2 2
xx = Ix0 x0 + m1 (r2 + r3 )

     
= m1 a2 + b2 + m1 a2 + b2 = m1 a2 + b2
12 4 3
1I = 1I + m1 (r32 + r12 )
yy y0 y0
     
= m1 a2 + b2 + m1 a2 + b2 = m1 a2 + b2
12 4 3
1I = 1I + m1 (r12 + r22 )
zz z0 z0
     
= m1 a2 + a2 + m1 a2 + a2 = m1 a2 + a2
12 4 3
!
1 1 a2
Ixy = Ix0 y0 + m1 r1 r2 = m1
4
Now we’ll consider the body with dimensions d, e, e. We’ll put a “2” to the upper left of the I as

148
an identifier.
For this body r1 = e/2, r2 = c + e/2 and r3 = b + d/2.
2I 2 2 2
xx = Ix0 x0 + m2 (r2 + r3 )

   2 
d2 + e2
2
c + 2e + b+ d

= m2 12 + m2 2
2I = 2I + m2 (r32 + r12 )
yy y0 y0
    2 
d2 + e2 e 2
+ b+ d

= m2 12 + m2 2 2
2I = 2I + m2 (r12 + r22 )
zz z0 z0
    2 
e2 + e2 e 2
+ c + 2e

= m2 12 + m2 2
e e
  
2 2
Ixy = Ix0 y0 + m2 r1 r2 = m2 c+
2 2
The individual masses are given by
m1 = (1000 kg /m3 )(0.1 m)(0.1 m)(0.02 m) = 0.2 kg

m2 = (800 kg /m3 )(0.03 m)(0.03 m)(0.06 m) = 0.0432 kg


Adding the rotational inertias together and evaluating for the given dimensional values gives us
Ixx = 1
Ixx + 2 Ixx = (1.6̄×10−2 m2 )(0.2 kg) + (1.01×10−2 m2 )(0.0432 kg)

Iyy = 1
Iyy + 2 Iyy = (1.6̄×10−2 m2 )(0.2 kg) + (3.1×10−3 m2 )(0.0432 kg)

Izz = 1
Izz + 2 Izz = (6.6̄×10−3 m2 )(0.2 kg) + (7.6×10−3 m2 )(0.0432 kg)

Ixy = 1
Ixy + 2 Ixy = (2.5×10−3 m2 )(0.2 kg) + (1.275×10−3 m2 )(0.0432 kg)

Ixx = 3.77×10−3 kg· m2 Iyy = 3.47×10−3 kg· m2

Izz = 1.66×10−3 kg· m2 Ixy = 5.55×10−4 kg· m2

149
8.7 Angular Momentum

150
8.7.1
GOAL: Determine the angular momentum of the illustrated body about the fixed point O.
GIVEN: The body rotates about the Y -axis with angular speed ω, and has areal density ρ.
DRAW:

FORMULATE EQUATIONS: Let x, y, z be a set of body-fixed axes with origin O. At the


instant illustrated, these axes are aligned with the ground-fixed X, Y, Z axes. The body rotates
about the Y -axis with angular speed ω; thus the z and Z axes remain aligned and the angular
*
velocity may be expressed as: ω = ω*h. The angular momentumi of the body about point O is then:
IXX ω1 − IXY ω2 − IXZ ω3 *
ı+
* h i
HO = IYY ω2 − IYZ ω3 − IYX ω1 *
+
h i*
IZZ ω3 − IZX ω1 − IZY ω2 k
* *
HO = −IXY ω *
ı + IY Y ω *
 − IZY ω k
Once we have determined the relevant inertia values about the point O, we may substitute and
solve for the angular momentum.
SOLVE: We only need to determine IXY , IY Y and IY Z . IY Y is easily found as the mass moment
of inertia about the Y axis is simply that of two thin rods, with masses ρab, ρac and lengths b, c,
respectively, rotated about their ends:
(ρac)c2 (ρab)b2 ρa(b3 + c3 )
IY Y = + =
3 3 3
Using the appropriate equations from (8.21)-(8.29) lets us determine the products of inertia. Each
rectangle has no product of inertia when evaluated about its mass center and thus the only finite
products of inertia come about because we’re shifting our attention from each plate’s mass center
to the point O.
a −b ρa2 b2
 
IXY = ρ(ab) =−
2 2 4
ac ρa2 c2
IY Z = ρ(ac) =
22 4
 
*
HO = ab2 *
ı + b3 + c3 *
 − ac2 k* ρaω
4 3 4

151
8.7.2
GOAL: Determine the angular momentum of the illustrated rectangular body about the fixed
point O.
GIVEN: The body rotates about the Z-axis with angular speed ω, and has mass m.
DRAW:

FORMULATE EQUATIONS: Let x, y, z be a set of body-fixed axes with origin O. At the


instant illustrated, these axes are aligned with the ground-fixed X, Y, Z axes. The body rotates
about the Z-axis with angular speed ω; thus the z and Z axes remain aligned and the angular
* *
velocity may be expressed as: ω = ω k .hThe angular momentumi
of the body about point O is then:
Ixx ω1 − Ixy ω2 − Ixz ω3 *
ı+
* h i
HO = Iyy ω2 − Iyz ω3 − Iyx ω1 *
+
h i*
Izz ω3 − Izx ω1 − Izy ω2 k
* *
HO = −Ixz ω *
ı − Iyz ω *
 + Izz ω k (1)
Once we have determined the relevant inertia values about the point O, we may substitute and
solve for the angular momentum.
SOLVE: The products of inertia about parallel axes through the center of mass G are all zero, due
to the symmetry of the body. The products of inertia about the point O may be computed from
the parallel axis expressions for inertia.
a b 1
  
Ixz : ¯
Ixz = Ix0 z 0 + mr1 r3 = 0 + m − = − mab
2 2 4
c b 1
  
Iyz : Iyz = I¯y0 z 0 + mr2 r3 = 0 + m − = − mcb
2 2 4
The moment of inertia about the z 0 -axis through the mass center is: I¯z 0 z 0 = 1
a2 + c2 . Using

12 m
the parallel axis expressions for inertia again, we obtain:
"   2 #
2
  1  2  a c
Izz = I¯z 0 z 0 + m r12 + r22 = m a + c2 + m +
Izz : 12 2 2
1  2 
= m a + c2
3
Substituting into (1),
*
HO = mω 14 ab *
ı + 14 cb *
 + 13 a2 + c2 k
 * 

152
8.7.3
GOAL: Determine the angular momentum of the illustrated ring.
GIVEN: The outer radius of the washer is r2 , the inner radius is r1 m, the mass is m and the
*
angular velocity is ω k .
DRAW:

FORMULATE EQUATIONS: From Appendix B we have the following inertias, taken about
the principal axes x0 , y 0 , z 0 of a solid cylinder, oriented as the ring is for this problem.
m(3r2 + h2 ) mr2
Ix0 x0 = Iz 0 z 0 = , Iy0 y0 =
12 2
All products of inertia are zero.
SOLVE: For our problem we’ll find the moments of inertia for two solid cylinders, one of radius
r1 and one of radius r2 . By subtracting we’ll obtain the inertias for the ring. The mass of the ring
is given by
m = πhρ(R22 − R12 )
Outer radius cylinder:
o o
πR22 hρ(3R22 + h2 )
Ix0 x0 = Iz 0 z 0 =
12

o
πR22 hρR22 πhρR24
Iy0 y0 = =
2 2
Inner radius cylinder:
i
πR12 hρ(3R12 + h2 )
Ix0 x0 = i Iz 0 z 0 =
12

i
πR12 hρR12 πhρR14
Iy0 y0 = =
2 2
We now subtract the inertias of the smaller hcylinder
 from the inertias
 of i
the larger one.
π hρ 3 R24 − R14 + h2 R22 − R12
Ix0 x0 = Iz 0 z 0 =   h 12  i
πhρ R22 − R12 h2 + 3 R12 + R22
= h  12 i
m h2 + 3 R12 + R22
= 12

153
  
π hρ R22 − R12 R12 + R22
Iy0 y0 = IY Y =
 2
2 2
m R 1 + R2
= 2
Using the parallel axis theorem to find IXX and IZZ we obtain
h  i
m h2 + 3 R12 + R22  2
h
2 2
mh2 m(R1 + R2 )
IXX = IZZ = +m = +
12 2 3 4
The products of inertia about X, Y , Z are zero from symmetry.
* *
The general expression for angular momentum (using ω = ω k ) is therefore
m(R12 + R22 )
" #
* 2
HO = mh
*

3 + 4 ωk

154
8.7.4
GOAL: Calculate the angular momentum of the system about point O.
*
GIVEN:
 The
 system consists of four thin bars of length a, and has an angular velocity ω =
*
10 *
 + 18k rad/s.
DRAW:

FORMULATE EQUATIONS: The framework is symmetric in the x−y, x−z, and y−z planes,
thus all products of inertia about the mass center G are zero. The body-fixed x, y, z axes are
therefore a set of principle axes for the system, and the angular momentum may be expressed as:
* * * *
HG = I¯xx ω1 b 1 + I¯yy ω2 b 2 + I¯zz ω3 b 3 (1)

To determine the angular momentum about the point O, we’ll use the expression,
* *
HO = HG + r*G/ ×mv*G (2)
O

SOLVE: We need to find the moments of inertia I¯xx , I¯yy , and I¯zz about the mass center. Assume
the bars have linear density ρ, and consider a differential mass unit dm = ρ dL, where dL is a
differential unit of length
√ along a bar. For a unit change in length dL along the bar, there is a
corresponding change 2 dz along the z-axis. dm can thus be expressed as:

dm = ρ dL = ρ 2 dz

Since the bars are one-dimensional, a single variable of integration (chosen to be z) is all that’s
needed to calculate the inertias. We’ll integrate over one of the bars, bar A in the diagram, and
then use our results to find the moments of inertia of the entire system. Note that the limits on z
are 0 and √a2 . For Iyy
A
:

√a
Z 2  √ 1 3
Z  
A
Iyy = x2 + z 2 dm = 0 + z 2 ρ 2 dz = ρa
6
BodyA 0

A A
By symmetry, we expect Izz = Iyy . Although we could perform this integration over z with the
a
substitution y = 2 − z along bar A, it will be easier to just apply the same arguments as above

to the y-axis and integrate over y:


√a
Z 2  √ 1 3
Z  
A
Izz = x2 + y 2 dm = 0 + y 2 ρ 2 dy = ρa
6
BodyA 0

155
As x2 = 0 in the above equations, for Ixx
A
we have:

1 3
Z  
A
Ixx = y 2 + z 2 dm = Iyy
A A
+ Izz = ρa
3
BodyA

The moments of inertia of all four bars about the center of mass of the framework will be the same,
due to the symmetry of the system. Thus,
4
I¯xx = Ixx
A B
+ Ixx C
+ Ixx D
+ Ixx A
= 4Ixx = ρa3
3
Similarly,
2
I¯yy = I¯zz = ρa3
3
The total mass of the system is M = ρ(4a), so rewriting these expressions in terms of M we obtain:
1 1
I¯xx = M a2 I¯yy = I¯zz = M a2 (3)
3 6
There are, of course, a number of ways to determine the above moments of inertia. A second
method worth mentioning involves the use of symmetry arguments and the parallel axis expressions.
Through the symmetry of the framework, we know that I¯yy = I¯zz , and since x2 = 0, we have
I¯xx = I¯yy + I¯zz = 2I¯yy = 2I¯zz . Thus, we only need to compute one of the inertia values, I¯xx . This
1
may be computed from the known moment of inertia of a thin bar about a transverse axis, 12 mL2 ,
the fact that there are 4 bars, and the parallal axis expressions:
!  2 !
1 M M a 1
   
I¯xx = 4 Ix0 x0 + m y + z 2 2
= 4 2
a + = M a2
12 4 4 2 3
a
where 2 is the distance from G to the center of any bar.
We are now ready to compute the angular momentum of the system. Note that at the given
* * * *
instant, the body-fixed b 1 , b 2 , b 3 unit vectors are aligned with the ground-fixed *
ı,*
 , k unit vectors.
Returning to equation (1), with substitutions from (3),
* * * *
HG = I¯xx ω1 b 1 + I¯yy ω2 b 2 + I¯zz ω3 b 3
1 1 1 *
= M a2 (0) *ı + M a2 (10) *  + M a2 (18) k
3 6 6
5 *
= M a2 * + 3M a2 k
3
Now substituting into equation (2), with v*G = ω×
*
r*G/
O

* *
HO = HG + r*G/ ×mv*G
O
5 a * a *
  
2* 2* *
= M a  + 3M a k + √ k ×M 10  + 18k × √ k
*
3 2 2
20 *
= M a2 *
 + 3M a2 k
3
*  *

20 *
HO = M a2 3  + 3k

156
8.7.5
GOAL: Calculate the angular momentum of the object about point O.
*
GIVEN: The object has an angular velocity ω = (−6 *
 + 10 *
 ) rad/s.
DRAW:

FORMULATE EQUATIONS: Theh angular momentum ofi the body about point O is given by:
Ixx ω1 − Ixy ω2 − Ixz ω3 *
ı+
* h i
HO = Iyy ω2 − Iyz ω3 − Iyx ω1 *
+ (1)
h i*
Izz ω3 − Izx ω1 − Izy ω2 k
Once we have determined the relevant inertia values about the point O, we can substitute and solve
for the angular momentum.
SOLVE: We’ll break the body into two pieces: AB and OC and identify the horizontal bar AB as
“H” and the vertical bar OC as “V .”
2m m
mH = , mV =
3 3

H H
mH L2 2mL2
IXX = IZZ = =
3 9
H H H H
IY Y = IXY = IY Z = IZX = 0

V V
mH L2 mL2
IXX = IY Y = =
3 9
V V V V
IZZ = IXY = IY Z = IZX = 0
Our overall rotational inertias are given by the sum of the two components:
2mL2 mL2 mL2
IXX = + =
9 9 3
mL2 2mL2
IY Y = , IZZ =
9 9
IXY = IY Z = IZX = 0
Using these values in (1), along with ω1 = −6 rad/s, ω2 = 10 rad/s, gives us
* *
HO = Ixx ω1 *
ı + Iyy ω2 *
 + Izz ω3 k
2
= mL * mL2 *
3 (−6 rad/s) ı + 9 (10 rad/s) 
*
ı + 10
 
HO = mL2 −2 * *
9  rad/s

157
8.7.6
GOAL: Calculate the angular momentum of the disk D about G and O.
*
GIVEN: The object has an angular velocity ω = (76 *
ı − 24 *
 ) rad/s. a = 0.3 m, r = 0.25 m,
m = 10 kg.
DRAW:

Note that the ω1 and ω2 of the original figure don’t correspond to angular rotation rates about the
* *
ı ,  axes, respectively.
FORMULATE EQUATIONS: The angular momentum expressions for the body when rotating
about the points O and G are given by:h i
Ixx ω1 − Ixy ω2 − Ixz ω3 *ı+
* h i
HG = Iyy ω2 − Iyz ω3 − Iyx ω1 *
+ (1)
h i*
Izz ω3 − Izx ω1 − Izy ω2 k
h i
Ixx ω1 − Ixy ω2 − Ixz ω3 *
ı+
* h i
HO = Iyy ω2 − Iyz ω3 − Iyx ω1 *
+ (2)
h i*
Izz ω3 − Izx ω1 − Izy ω2 k
* * * * * *
where ω D = ω1 ı + ω2  + ω3 k = (76 ı − 24  ) rad/s.
Once we have determined the relevant inertia values about the point O, we can substitute and solve
for the angular momentum.
SOLVE:
mr2 mr2
Iy0 y0 = Iz 0 z 0 = , Ix0 x0 = (3)
4 2
All products of inertia are zero.
We can use the parallel axis theorem to find the corresponding rotational inertias about O:
mr2 mr2
IY Y = IZZ = + ma2 , IXX = (4)
4 2
Due to symmetry the products of inertia are still zero.
2
*
HG = mr * mr2 *
(1), (3) ⇒ 2 ω1 ı + 4 2 ω2 
(10 kg)(0.25 m)
= 2 (76 *
ı − 12 *
 ) rad/s
= (23.75 *  ) kg· m2 / s
ı − 3.75 *

158
 
2
*
HO = mr ω *
ı + m r2 + a2 ω * 
(2), (4) ⇒ 2 1 4 2
(0.25 m)2 (0.25 m)2
   
= (10 kg) * *
76 ı − 24  + (0.3 m)2 rad/s
2 4
= (23.75 *  ) kg· m2 / s
ı − 25.35 *

159
8.7.7
GOAL: Calculate the angular momentum of the system about O.
GIVEN: The framework has a mass of 0.005 kg and each arm has a length of h = 0.12 m. The
wheels have radii r = 0.007 m and negligible thickness. The framework rotates about the Y axis
at 2.2 rad/s and the wheels rotate without slip. Each wheel has a mass of 0.001 kg.
DRAW:

FORMULATE EQUATIONS: h i
Ixx ω1 − Ixy ω2 − Ixz ω3 *
ı+
* h i
HO = Iyy ω2 − Iyz ω3 − Iyx ω1 *
+ (1)
h i*
Izz ω3 − Izx ω1 − Izy ω2 k
Once we have determined the relevant inertia values about the point O, we can substitute and solve
for the angular momentum.
SOLVE:
Although the system contains several pieces, its symmetry allows us to simplify the analysis substan-
tially. Each individual wheel will generate an angular momentum contribution due to its rotation
about the axis along the corresponding arm (the X axis for wheel A in the figure, for instance).
Because an identical contribution will be generated by each wheel, their sum will go to zero. Thus
we’re left with the same angular momentum as we’d have for the 3-arm assemble and attached
wheels rotating about the Y axis as a single rigid body.
Each arm of the 3-arm body has mass moment of inertia about the Y axis of
m3−a h2
!
m3−a h2
IY Y = =
3 3 9
For each wheel we have
mw r2
IY Y = + mw h 2
4
The complete rotational inertia is therefore
" !#
(0.005 kg)(0.12 m)2 (0.007 m)2
IY Y =3 + (0.001 kg) + (0.12 m)2 = 6.72×10−5 kg· m2
9 4

Thus we have
*
HO = 6.72×10−5 kg· m2 (2.2 *
 rad/s) = 1.48×10−4 *
 kg· m2 / s


160
8.7.8
GOAL: Determine the angular momentum of the illustrated rectangular body about the fixed
point O.
GIVEN: The body rotates about the Z-axis with angular speed ω3 , and has a linear density ρ.
DRAW:

FORMULATE EQUATIONS: Let x, y, z be a set of body-fixed axes with associated unit vectors
* * *
b 1 , b 2 , b 3 . The general expression forh the angular momentum
i
of the body about point O is:
Ixx ω1 − Ixy ω2 − Ixz ω3 *
ı+
* h i
HO = Iyy ω2 − Iyz ω3 − Iyx ω1 *
+
h i*
Izz ω3 − Izx ω1 − Izy ω2 k
Letting ω1 = ω2 = 0 gives us
* *
HO = (−Ixz *
ı + −Iyz *
 + Izz k )ω3
Once we have determined the relevant inertia values about the point O, we can substitute and solve
for the angular momentum.
SOLVE: Denote the mass of the segment OA by m1 and the mass of the segment AB by m2 .
m L2
Izz = 1 1 + m2 L21
3
−L2 m L L
 
Ixz = m2 L1 =− 2 1 2
2 2

Iyz = 0

m1 L21
" ! #
* m2 L1 L2 * *
HO = ı + + m2 L21 k ω3
2 3
Re-expressing m1 and m2 in terms of ρ, L1 , L2 gives us
L22 * L21
" ! #
* *
HO = ρL1 ω3 2 ı + 3 + L1 L2 k

161
8.7.9
GOAL: Find the angular momentum of the illustrated structure about the point O.
* *
GIVEN: The angular velocity of the structure is ω = ω b 1 and its areal density is ρ.
DRAW:

FORMULATE EQUATIONS: The equation of interest is:


* h i*
HO = Ixx ω1 − Ixy ω2 − Ixz ω3 b 1 +
h i*
Iyy ω2 − Iyz ω3 − Iyx ω1 b 2 + (1)
h i*
Izz ω3 − Izx ω1 − Izy ω2 b 3

* *
SOLVE: Since ω = ω b 1 , equation (1) gives us:
* * * *
HO = Ixx ω b 1 − Iyx ω b 2 − Izx ω b 3

We need to calculate Ixx , Iyx , and Izx .


The moment of inertia of a thin, rectangular plate about an axis perpendicular to its surface and
1
through the center of mass is 12 m(l2 + w2 ), where l is the length of the plate and w its width. The
moment of inertia about a transverse axis lying in the plane of the plate, through the center of
1
mass and parallel to an edge of length w is 12 ml2 . Using the parallel axis expressions, the moment
of inertia about the x axis for each plate is:
"  2 #
2
1 b c
    
2 2
B
Ixx = I B
+ mB r2 + r 3 = mB b2 + c2 + mB + −
x00 x00 12 2 2
1  
= mB b2 + c2
3
2
1 c 1
  
A
Ixx = I A0 + mA r22 + r32 = mA c2 + mA − = m c2
x x0 12 2 3 A

For the composite structure,

1   1 1  3 
B
Ixx = Ixx A
+ Ixx = ρbc c2 + b2 + ρacc2 = ρ ac + b3 c + bc3
3 3 3

162
A
The plane of symmetry for plate A is the x−z plane, so Iyx = 0. The plane of symmetry for plate B
B B
is the y−z plane, so Iyx = Izx = 0. The only remaining moment of inertia needed for substitution
A
into equation (1) is Izx . Notice that all products of inertia about the center of mass of each plate
are zero, thus by the parallel axis expressions we have:
c a 1
  
A
Izx = Izx = I A
+ mA r3 r1 = 0 + mA − = − ρa2 c2
z 0 x0 2 2 4

Substition into (1) yields


*
HO = 13 ρ ac3 + b3 c + bc3 ω b 1 + 14 ρa2 c2 ω b 3
 * *

163
8.8 Equations of Motion for a Three-Dimensional Body

164
8.8.1
GOAL: Determine the bearing forces for a rotating shaft.
GIVEN: System configuration. Each lumped mass is 2 kg. a = 14 cm and b = 11 cm. ω =
100 rad/s.
DRAW:

FORMULATE EQUATIONS: We’ll use the a force balance


*
F = ma*G
and three moment balances aboutX
the shaft’s mass center
M1 = Ḣ1 + (ω2 H3 − ω3 H2 )
X
M2 = Ḣ2 + (ω3 H1 − ω1 H3 )
X
M3 = Ḣ3 + (ω1 H2 − ω2 H1 )
* * * *
where HG = H1 b 1 + H2 b 2 + H3 b 3
SOLVE: From (8.32) we have:
* * * *
HG = −I x0 z 0 ω b 1 − I y0 z 0 ω b 2 + I z 0 z 0 ω b 3

3a a a −3a
       
I x0 z 0 = m1 (0) + m2 (−b) + m3 − (0) + m4 (b)
2 2 2 2
ab
I x0 z 0 = − (m + 3m4 ) = −2mab
2 2
3a a a −3a
     
I y0 z0 = m1 (−b) + m2 (0) + m3 − (b) + m4 (0)
2 2 2 2
ab
I y0 z0 = − (3m1 + m3 ) = −2mab
2
I z0 z0 = b2 (m1 + m2 + m3 + m4 ) = 4mb2
* * * *
HG = 2mabω b 1 + 2mabω b 2 + 4mb2 ω b 3

165
H1 = 2mabω
H2 = 2mabω
H3 = 4mb2 ω

5a
M1 = 0 − 2mabω 2 = (R4 − R2 ) (1)
2
2 5a
M2 = 0 + 2mabω = (R1 − R3 ) (2)
2
M3 = 0 (3)
R1 + R3 = 0 (4)
R2 + R4 = 0 (5)

2mbω 2
(5) → (1) ⇒ 5aR4 = −2mabω 2 ⇒ R4 = − (6)
5
2mbω 2
(6) → (5) ⇒ R2 =
5
2mbω 2
(4) → (2) ⇒ 5aR1 = 2mabω 2 ⇒ R1 = (7)
5
2mbω 2
(7) → (4) ⇒ R3 = −
5
Using the given values:
2(2 kg)(0.11 m)(100 rad/s)2
R1 = 5 = 880 N
R3 = −880 N R2 = 880 N R4 = −880 N

166
8.8.2
GOAL: Find the moment the shaft must exert on the disk for it to rotate with constant angular
speed ω.
GIVEN: The radius of the disk is R and its areal density is ρ. The disk is inclined an angle φ
from the plane normal to the axis of the shaft, as shown.
DRAW: Let x, y, z be a set of body-fixed axes attached to the disk, with the origin at its center
of mass. The z-axis is chosen to be normal to the plane of the disk.

FORMULATE EQUATIONS: The angular momentum equation of motion has the form:

X * *
˙
MG = HG

*
One method of solution is therefore to form HG and take its derivative. A second method of solution
would be to use Euler’s equations, since the x, y, z axes are a set of principle axes for the disk. We
will use both methods and check that the solutions are the same.
SOLVE: The moments of inertia of the disk about the x, y, z axes are I¯xx = I¯yy = 14 mR2 , and
I¯zz = 1 mR2 . The angular velocity of the disk is:
2

*
 * *

*
ω = ωk = ω cos φ b 3 + sin φ b 1

*
Since the angular velocity is constant, the angular acceleration α is zero.
By symmetry, all products of inertia in the x, y, z coordinate system are zero (and thus x, y, z are
principle axes). The angular momentum then becomes:

* * * *
HG = I¯xx ω1 b 1 + I¯yy ω2 b 2 + I¯zz ω3 b 3
1 * 1 *
= mR2 ω sin φ b 1 + 0 + mR2 ω cos φ b 3 (1)
4 2

167
Taking the derivative:

X * *
˙
d * *
*
MG = HG = HG + ω×HG
N
dt S

 * *
 1 * 1 *

= 0 + ω cos φ b 3 + sin φ b 1 × mR ω sin φ b 1 + mR2 ω cos φ b 3
2
4 2
1 *
= − mR2 ω 2 sin φ cos φ b 2
4
*
Note that the unit vector b 2 is always in the direction perpendicular to the axis of the shaft. With
m = ρπR2 , we have:
*
M = − 14 ρπR4 ω 2 sin φ cos φ b 2
*

*
Since α and ω2 are zero, the only non-zero Euler’s equation is

1 1
 
= I¯yy α2 + ω1 ω3 I¯xx − I¯zz mR2 − mR2

M2 = 0 + (ω sin φ) (ω cos φ)
4 2

M2 = − 14 ρπR4 ω 2 sin φ cos φ

which yields an identical result as above.

168
8.8.3
GOAL: Determine the bearing forces at A.
GIVEN: System configuration. The shaft has a mass m1 and the half-disk has a mass m2 .
DRAW:

* *
The primed axes are attached to the disk and rotate along with it, as do the unit vectors b 1 , b 2 ,
*
b 3 . The radius of the half-disk is given by r = d/2.
FORMULATE EQUATIONS: We’ll use the a force balance
*
F = ma*G
and three moment balances aboutX
the shaft’s mass center
M1 = Ḣ1 + (ω2 H3 − ω3 H2 )
X
M2 = Ḣ2 + (ω3 H1 − ω1 H3 )
X
M3 = Ḣ3 + (ω1 H2 − ω2 H1 )
* * * *
where HG = H1 b 1 + H2 b 2 + H3 b 3
SOLVE: From (8.32) we have:
* * * *
HG = −I z 0 x0 ω b 1 − I y0 z 0 ω b 2 + I z 0 z 0 ω b 3

I z 0 x0 = I y 0 z 0 = 0

m2 r 2
I zz =
4
* m2 r 2 ω *
HG = b3
4
m2 r 2 ω
H1 = 0, H2 = 0, H3 =
4
We can now consider our force and moment balances.

169
3d
M1 = 0 = (R4 − R2 ) (1)
2
3d
M2 = 0 = (R1 − R3 ) (2)
2
4m rω 2
R2 + R4 − (m1 + m2 )g cos θ = −m2 rω 2 = − 2 (3)

R1 + R3 − (m1 + m2 )g sin θ = 0 (4)

(2), (4) ⇒ 2R1 = g(m1 + m2 ) sin θ

g(m1 + m2 ) sin θ
R1 = R3 = 2
d
 
4m2 ω2
(1), (3) ⇒ 2
2R2 = g(m1 + m2 ) cos θ −

g(m1 + m2 ) cos θ m2 dω 2
R 2 = R4 = 2 − 3π

170
8.8.4
GOALS:
(0.1) Find the full equations of motion for the illustrated system under the condition ψ̈ = 0.
(0.2) Find the equilibrium conditions for which ψ̇ = ψ̇0 (constant), θ̇ = θ̈ = 0 and θ = θ0 6= 0.
GIVEN: The rod is attached by a horizontal pivot to a vertical shaft, as shown in the figure. The
linear density of the rod is ρ.
DRAW: Let x, y, z be a body-fixed coordinate system attached to the rod, with corresponding
* * *
unit vectors *c 1, *
c 2, *
c 3 . Let b 1 , b 2 , b 3 be a set of unit vectors that rotate with the vertical shaft,
and, at the instant depicted, are aligned with the ground-fixed X, Y, Z axes.

*
The coordinate transformation matrix between b i and *
c i is:
* * *
b1 b2 b3
*
c1 1 0 0
*
c2 0 cos θ − sin θ
*
c3 0 sin θ cos θ

GOVERNING EQUATIONS: The full equations of motion will consist of three force equations
and three moment equations. Newton’s law provides the force equations, and since x, y, z are
principle axes for the rod (two planes of symmetry, x−z and y−z), Euler’s equations will provide
the moment equations:
*
F = ma*G (1)

X  
M1 = Ixx α1 + ω2 ω3 Izz − Iyy (2)
X
M2 = Iyy α2 + ω1 ω3 (Ixx − Izz ) (3)
X  
M3 = Izz α3 + ω1 ω2 Iyy − Ixx (4)

*
where MO = M1 * c 1 + M2 *
c 2 + M3 *
c 3 is the net moment acting on the rod about the point O.
SOLVE: We’ll begin with the left side of equations (1-4). The forces acting on the rod are the
force of gravity and the reaction force between the rod and shaft at point O. Denote the reaction

171
force acting on the rod as having components Ri in the *
c i frame. The net force acting on the rod
is then:
* *
F = R1 *
c 1 + R2 *
c 2 + R3 *
c 3 − mg b 3
= R1 c 1 + R2 c 2 + R3 c 3 − ρLg (− sin θ *
* * *
c 2 + cos θ *
c 3) (5)

The rod pivots freely about the *


c 1 direction, so we may write the reaction torque at O as having
*
only two components, T2 and T3 , in the * c 2 and *
c 3 directions. The net moment MO acting on the
rod is then:
*  *

MO = T2 *
c 2 + T3 *
c 3 + r*G/ × −mg b 3
O
 L   
= T2 *
c 2 + T3 *
c3 + − c 3 ×−ρLg − sin θ *
*
c 2 + cos θ *
c3
2
1
= T2 * c 3 + ρgL2 sin θ *
c 2 + T3 * c1 (6)
2

That takes care of the left side of our equations. We will now address the kinematic and inertial
terms appearing on the right side of equations (1-4). We begin with the angular velocity of the
rod, which is:
* * *
ω = ψ̇ b 3 − θ̇ b 1 = ψ̇ (cos θ *
c 3 − sin θ *
c 2 ) − θ̇ *
c1 (7)

Taking a time derivative, the angular acceleration is


* *
* db3 * db
*
α = ψ̈ b 3 + ψ̇ − θ̈ b 1 − θ̇ 1
dt dt
* * *
= (0) b 3 + ψ̇(0) − θ̈ b 1 − θ̇ψ̇ b 2
= −θ̈ *
c 1 − θ̇ψ̇ cos θ *
c 2 − θ̇ψ̇ sin θ *
c3 (8)

Since O is fixed in space, the velocity and acceleration of the rod’s center of mass may be computed
as

v*G *
= ω× r*G/
O
  L*
= ψ̇ cos θ *
c 3 − ψ̇ sin θ *
c 2 − θ̇ *
c 1 ×− c
2 3
1 1
= ψ̇L sin θ *
c 1 − θ̇L *
c2 (9)
2 2

a*G *
= α× r*G/ + ω×
*
v*G
O
  L
= −θ̈ *
c 1 − θ̇ψ̇ cos θ *
c 2 − θ̇ψ̇ sin θ *
c 3 ×− * c +
2 3
  1 1 
+ ψ̇ cos θ * c 3 − ψ̇ sin θ *
c 2 − θ̇ *
c 1 × ψ̇L sin θ *
c 1 − θ̇L * c2
2 2
1   1  
c 1 + L ψ̇ 2 sin θ cos θ − θ̈ *
= ψ̇ θ̇L cos θ * c 2 + L ψ̇ 2 sin2 θ + θ̇2 *c3 (10)
2 2

The moments of inertia of the thin rod about the x, y, z axes are Ixx = Iyy = 13 mL2 = 13 ρL3 and
Izz = 0. Substitution of (6),(7),(8) into our moment balance equations (2-4), with m = ρL, now
yields:

172
1 1 3   1
X     
M1 = mgL sin θ = ρL −θ̈ + −ψ̇ sin θ ψ̇ cos θ 0 − ρL3
(2) → 2 3 3
1 1 3 
ρgL2 sin θ = ρL −θ̈ + ψ̇ 2 sin θ cos θ
2 3

⇒ 3g sin θ − ψ̇ 2 sin θ cos θ = 0


θ̈ + 2L (11)

This is the equation of motion for θ, integration of which will yield θ(t).

1 3  1
hX i     
(3) → M2 = T 2 = ρL −θ̇ψ̇ cos θ + −θ̇ ψ̇ cos θ ρL3 − 0
3 3

⇒ T2 = − 23 ρL3 θ̇ψ̇ cos θ (12)

hX i      1 1

(4) → M3 = T 3 = 0 −θ̇ψ̇ sin θ + −θ̇ −ψ̇ sin θ ρL3 − ρL3
3 3

⇒ T3 = 0 (13)

Note that the constraint torque T3 has been included in our analysis since rotation of the rod about
*
c 3 is constrained (unlike the rotation about * c 1 , for example). However, our asssumption that the
rod is one-dimensional requires that this constraint torque is always zero.
*
Let Fi = F · *c i and ai = a* · *
c i . A force balance (1) then takes the form (with m = ρL):
*
F = ma*G ⇒ Fi = ρLai (i = 1, 2, 3)

*
(F = ma*G ) · *
c1 ⇒ R1 = ρψ̇ θ̇L2 cos θ (14)
*
 
1 2
(F = ma*G ) · *
c2 ⇒ R2 + ρLg sin θ = 2 ρL ψ̇ 2 sin θ cos θ − θ̈ (15)
*
 
1
(F = ma*G ) · *
c3 ⇒ R3 − ρLg cos θ = 2 ρL
2 ψ̇ 2 sin2 θ + θ̇2 (16)

You might notice that force equation (15) and motion equation (11) share some common terms.
Although unnecessary for our purposes, this leads to an interesting simplification for R2 , in that
both the second and first order time-derivatives may be eliminated:
1
(11) → (15) ⇒ R2 = − ρLg sin θ
4
(0.2) Substitution of the given equilibrium conditions into the equation of motion (11) yields:
3g
0+ sin θ0 − ψ̇02 sin θ0 cos θ0 = 0
2L
3g
⇒ − ψ˙02 cos θ0 = 0
2L
!
⇒ θ0 = cos−1 3g
2Lψ̇02

173
which is the equilibrium value of θ for a given ψ̇0 .
In order for the above inverse cosine function to exist, the operand must lie in the range [-1,1].
Thus,
3g
−1 ≤ ≤ 1
2Lψ̇02
Since all quantities appearing in the operand are positive, this is equivalent to:

3g
ψ̇02 ≥ 2L

If ψ̇0 is too small, no such equilibria (θ0 6= 0) exists. Also note that the same argument implies
cos θ ≥ 0, assuring −π/2 ≤ θ0 ≤ π/2.

174
8.8.5
GOAL: Find the rotational speed that allows θ to remain constant at 45◦ . Is this the same result
as if all the mass were concentrated at C?
GIVEN: System configuration. L = 0.5 m, m = 2 kg.
DRAW:

FORMULATE EQUATIONS:
We’ll use the a force balance
*
F = ma*G
and three moment balances aboutX
the shaft’s mass center
M1 = Ḣ1 + (ω2 H3 − ω3 H2 )
X
M2 = Ḣ2 + (ω3 H1 − ω1 H3 )
X
M3 = Ḣ3 + (ω1 H2 − ω2 H1 )
* * * *
where HG = H1 b 1 + H2 b 2 + H3 b 3
SOLVE: *
*
(a) Using (8.33) and ω = ω k gives us
* * * *
HO = −Izx ω b 1 − Iyz ω b 2 + Izz ω b 3

mL2 1
 
Izz = + mL2 sin2 θ = mL2 + sin2 θ
12 12
Iyz = mL2 cos θ sin θ
Izx = 0
 
1
H1 = 0, H2 = −mL2 cos θ sin θω, H3 = mL2 12 + sin2 θ .
Ḣ1 = 0, Ḣ2 = 0 and Ḣ3 = 0.
The first of our moment balance equations yields
X
M1 = −ωH2 = ω 2 mL2 cos θ sin θ = mgL sin θ

g 9.81 m/s2
ω2 = = √ = 27.75 rad/s2
L cos θ (0.5 m)(1/ 2)

175
ω = 5.27 rad/s
(b) If all the mass were at C then we’d have:

Izz = mL2 sin2 θ


Iyz = mL2 cos θ sin θ
Izx = 0

Iyz is the same as in (a). Therefore the results for this case will not differ from those of (a).

176
8.8.6
GOAL: Describe the effect of lateral perturbations on the front wheel of a bicycle, when one is
riding with no hands.
DRAW:

ASSUME: Initially, the bike travels in a perfectly straight line and is perfectly upright.
FORMULATE EQUATIONS: The sum of the moments about the mass center of the wheel
must equal the time rate of change of its angular momentum.
X * d*
MG = H
dt G
*
SOLVE: Let * ı be the forward vector, *
 the right-to-left vector, and k the vertical up direction.
Initially, the front wheel has an angular momentum about its mass center with only one component
*
in the *  direction, HG = H2*  . Assume that instead of riding with no hands, the handlebars are
* *
held fixed by the rider. If the bike leans to the left (ω · ı 6= 0), the angular momentum vector
*
will tilt towards the ground, in the −k direction. Thus, the net moment on the wheel must also
*
have component in the −k direction (provided by the rider’s grip on the handlebars). We conclude,
*
therefore, that with no hands the wheel will have a tendency to turn in the positive k direction,
or to the left.
A similar argument shows that when the rider leans right, the wheel will turn to the right as well.

Leaning causes the bike to turn in the direction of the lean.


* * *
We may obtain the same result with an examination of a moment balance. Let b 1 , b 2 , b 3 be a set
*
of body-fixed unit vectors initially aligned with *
ı,*
 k . If the bike is traveling forward and leaning
* * *
to the right then its angular velocity may be represented as ω = ω1 b 1 + ω2 b 2 and the angular
* * * *
momentum as HG = H1 b 1 + H2 b 2 . Our third moment balance equation, corresponding to the b 3
component, is
M3 = I¯zz α3 + ω1 ω2 (I¯yy − I¯xx )
X

For a wheel, I¯yy > I¯xx . With no hands, and assuming a negligible
P resistance to turning at the
contact point between the wheel and ground, the net moment M3 will be zero and thus

ω1 ω2 (I¯yy − I¯xx )
α3 = − < 0
I¯zz
Thus the wheel turns to the right.

177
The resulting motion of the wheel will tend to stabilize the bicycle. When the wheel turns, the
centripetal acceleration of the bicycle requires a frictional force between the wheels and ground
toward the center of curvature of the rider’s path. The moment this force exerts on the bicycle will
tend to counter the moment due to gravity.

178
8.8.7 *
*
GOAL: Find the total force F and moment MG acting on the dynamical potato.
GIVEN: Linear velocity and acceleration; angular velocity and acceleration; mass of potato; and
all moments and products of inertia.
DRAW:

GOVERNING EQUATIONS:
*
F = ma* (1)


X * *
˙ d * *
MG = HG = HG + ω×HG (2)
N dt S

SOLVE: Determing the net force acting on the dynamical


 
potato is a simple matter.
*
F = 25 kg 4 ı m/s2
*

(1) → *
F = 100 *
ı N

To apply (2), we require the angular momentum about G, which may be computed from the
expression
h i*
Ixx ω1 − Ixy ω2 − Ixz ω3 b 1 +
* h i*
HG = Iyy ω2 − Iyz ω3 − Iyx ω1 b 2 + (3)
h i*
Izz ω3 − Izx ω1 − Izy ω2 b 3

At the given instant,

[(0.05)(8) − (0.02)(−12) − (0.04)(3)] *


ı +
*
HG = [(0.03)(−12) − (0.02)(8) − (0.07)(3)] *
 +
*
[(0.10)(3) − (0.04)(8) − (0.07)(−12)]k
*
= (0.52 *  + 0.82k ) kg·m2 /s
ı − 0.73 *

179
Now, with the substitution of (3) into (2),

X * d * *
MG = HG + ω×HG
dt S

h i*
= Ixx α1 − Ixy α2 − Ixz α3 b 1 +
h i*
Iyy α2 − Iyz α3 − Iyx α1 b 2 +
h i* *
Izz α3 − Izx α1 − Izy α2 b 3 + ω×HG
= [(0.05)(85) − (0.02)(−100) − (0.04)(30)] *ı +
*
[(0.03)(−100) − (0.02)(85) − (0.07)(30)]  +
*
[(0.10)(30) − (0.04)(85) − (0.07)(−100)]k +
* *
(8 *
ı − 12 *
 + 3k )×(0.52 *
ı − 0.73 *
 + 0.82k )
*  *

MG = −2.60 *
ı − 11.8 *
 + 7.00k N·m

Note that the linear velocity does not appear in any of our computations.

180
8.8.8
GOAL: Determine the angular acceleration of the bar and the support force FO at the instant the
bar is released. Consider two cases: a) θ = 0 and θ = 30◦ .
GIVEN: Mass and dimensions of bar, initial positions before release.
DRAW:

FORMULATE EQUATIONS: The equations of motion for a rigid body are


*
F = ma* (1)

X * *˙ d * *
*
MO = HO = HO + ω×HO (2)
N dtS

*
Since we are only concerned with the instant of release, ω = 0, and equation (2) becomes:
X * h i*
MO = Ixx α1 − Ixy α2 − Ixz α3 b 1 +
h i*
Iyy α2 − Iyz α3 − Iyx α1 b 2 + (3)
h i*
Izz α3 − Izx α1 − Izy α2 b 3

SOLVE: The moment of inertia for a slim rod about a transverse axis through its mass center
1
is 12 mL2 . With a little help from the superposition principle and the parallel axis theorem, the
inertias of our rigid body about point O are:
1 3 1 2
 
Ixx = m b2 = mb = 0.0386 kg·m2
3 7 7
1 4 3 13
 
Iyy = m a2 + ma2 = ma2 = 0.297 kg·m2
3 7 7 21
3 b 2 13 1
   
Izz = Ixx + Iyy = m a2 + = ma2 + mb2 = 0.336 kg·m2
7 2 21 7
3 b 3
 
Ixy = ma = mab = 0.0771 kg·m2
7 2 14
Ixz = 0
Iyz = 0

181
where m = 3 kg, a = 0.40 m, and b = 0.30 m.
The center of mass of the bar is located at
4 a* 3 * b* 5 * 3 *
   
* * *
xG = b1 + ab1 + b2 = a b1 + b b 2 = 0.286 b 1 + 0.0643 b 2
7 2 7 2 7 14
* *
(a) When θ = 0, the space-fixed *ı,* , k and body-fixed b i unit vectors are aligned. The only
moment about O is that due to gravity, which at the given instant is
X
* * 5mga * 3mgb *
MO = xG ×(−mg k ) =  − ı
7 14
Using this result in (3) yields three equations
3mgb
− = Ixx α1 − Ixy α2
14
5mga
= Iyy α2 − Ixy α1
7
0 = Izz α3

which may be solved for the angular accelerations

α1 = 15.7 rad/s2 ; α2 = 32.4 rad/s2 ; α3 = 0


*
α = (15.7 *  ) rad/s2
ı + 32.4 *

The acceleration of the mass center is


2 *
a* = α×
* *
x * * * *
G = (15.7 ı + 32.4  )×(0.286 ı + 0.0643  ) = −8.24k rad/s

Equation (1) may now be solved for the reaction force,


* * * * *
FO − mg k = ma* ⇒ FO = (3 kg)(−8.24k rad/s2 ) + (3 kg)(9.81 m/s2 )k
* *
FO = 4.71 Nk
* *
(b) When θ = 30◦ , the gravity vector becomes −g sin 30◦ b 2 − g cos 30◦ b 3 . The moment about O is
now:
5 * 3 *
 
* *
b b 2 ×(−mg sin 30◦ b 2 − mg cos 30◦ b 3 )
X
MO = a b1 +
7 14
√ √
5amg * 5 3amg * 3 3bmg *
= − b + b2 − b1
14 3 14 28
Using this result in (3) and solving for the angular acceleration components yields

α1 = 13.6 rad/s2 ; α2 = 28.0 rad/s2 ; α3 = −12.5 rad/s2


* * * *
*
α = (13.6 b 1 + 28.0 b 2 − 12.5 b 3 ) rad/s2 = (13.6 *  + 3.17k ) rad/s2
ı + 30.5 *

The acceleration of the mass center is


* * * * *
a* = α×
* *
xG = (13.6 b 1 + 28.0 b 2 − 12.5 b 3 )×(0.286 b 1 + 0.0643 b 2 )
* * *
= (0.805 b 1 − 3.58 b 2 − 7.14 b 3 ) rad/s2

182
*
and the force FO is
* * *
FO = ma* + mg(sin 30◦ b 2 + cos 30◦ b 3 )
* * * * *
= (3 kg)(0.805 b 1 − 3.58 b 2 − 7.14 b 3 ) + (3 kg)(9.81 m/s2 )(sin 30◦ b 2 + cos 30◦ b 3 )
* * * * *
FO = (2.42 b 1 + 3.98 b 2 + 4.08 b 3 ) N = (2.42 *
ı + 1.41 *
 + 5.52k ) N

183
8.8.9 *
*
GOAL: Determine the angular acceleration α of the disk and the reaction force FO at the given
instant.
GIVEN: We’re given the mass and dimensions of the disk as well as its initial position and angular
velocity.
DRAW:

GOVERNING EQUATIONS: The equations of motion for a rigid body are


*
F = ma*G (1)

X * *˙ d * *
*
MO = HO = HO + ω×HO (2)
N dt
S
SOLVE: The center of mass of the disk is located at
4R * 4 *
 
r*G/ = −R * ı −  = −*
ı −  m (3)
O 3π 3π
The mass moments of inertia of the disk about point O are
1 1
Ixx = mR2 = (12 kg)(1.0 m)2 = 3 kg·m2
4 4
1 5 5
Iyy = mR2 + mR2 = mR2 = (12 kg)(1.0 m)2 = 15 kg·m2
4 4 4
1 3 3
Izz = mR2 + mR2 = mR2 = (12 kg)(1.0 m)2 = 18 kg·m2
2 2 2
4R 4 4 16
 
Ixy = m (−R) − = mR2 = (12 kg)(1.0 m)2 = kg·m2
3π 3π 3π π

Ixz = Iyz = 0
To apply (2) we require the angular momentum of the disk about O, which may be computed from
h i*
Ixx ω1 − Ixy ω2 − Ixz ω3 b 1 +
* h i*
HO = Iyy ω2 − Iyz ω3 − Iyx ω1 b 2 + (4)
h i*
Izz ω3 − Izx ω1 − Izy ω2 b 3

184
At the given instant,
* *
HO = [0 − (16/π)(2) − 0] *
ı + [(15)(2) − 0 − 0] *
 + [(18)(5) − 0 − 0]k
*
= (−32/π *  + 90k ) kg·m2 /s
ı + 30 * (5)
* *
MO = r*G ×m(−g k ), and then taking the dot products
P
Substituting (4) and (5) into (2), with
*
with *
ı,* , k yields three equations:

(2) · *ı ⇒ 16g/π = [3α1 − (16/π)α2 ] + (2)(90) − (5)(30)


*
(2) ·  ⇒ −12g = [15α2 − (16/π)α1 ] + (5)(−32/π)
*
(2) · k ⇒ 0 = 18α3 − (2)(−32/π)

Solving the above for α1 , α2 , and α3 ,

α1 = −2.14 rad/s2 α2 = −5.18 rad/s2 α3 = −1.13 rad/s2

*
*
α = (−2.14 *  − 1.13k ) rad/s2
ı − 5.18 *
The acceleration of the mass center is
*
a*G = α×
*
r*G = (−0.480 *  − 4.27k ) m/s2
ı + 1.13 *
*
Equation (1) may now be solved for the force FO ,
* *
FO − mg k = ma*G
* * *
⇒ FO = (12 kg)(9.81 m/s2 )k + (12 kg)(−0.480 *  − 4.27k ) m/s2
ı + 1.13 *
* *
FO = (−5.76 *
ı + 13.6 *
 + 66.5k ) N

185
8.8.10
GOAL: Find the acceleration a*G of the mass center, the angular acceleration α,
*
and the acceler-
*
ation aP of the point P at the instant given.
*
GIVEN: Weight and dimensions of body, its angular velocity; and the force F Q applied at point
Q.
DRAW:

FORMULATE EQUATIONS: The equations of motion for a rigid body are


*
F = ma*G (1)

X * *˙ d * *
*
MG = HG = HG + ω×HG (2)
N dt
S
SOLVE: From (1), the acceleration of the mass center is
* *
F [(20 *
ı − 8*
 + 10k ) − 40 *
 ] lbs *
*
aG = = = (16.1 *  + 8.05k ) ft/s2
ı − 38.64 * (3)
40 lbs
 
m
32.2 ft/s2
The rigid body can be viewed as a composite body with three sections: a lower-left cube, an upper-
right cube, and a rectangular parallelpiped in the middle. The moments of inertia for the body are
found by superimposing the inertias of all three sections, and applying the parallel-axis expressions
as needed. Let x0 , y 0 , z 0 be a new set of coordinates axes parallel to x, y, z, and with an origin at
the mass center. By symmetry, I¯x0 z 0 = I¯y0 z 0 = 0.
 2 
1 m 2 m h 1 m 2 7
       
I¯x0 x0 = 2 h + + h = mh2
6 4 4 2 6 2 24
2 
1 m 2 m 3h 1 m 17
        
I¯y0 y0 = 2 h + + 2
h + (2h) 2
= mh2
6 4 4 2 12 2 12
1 m 2 m 3h 2 h 2 1 m 37
            
I¯z 0 z 0 = 2 h + + + 2
h + (2h) 2
= mh2
6 4 4 2 2 12 2 24
m 3 h m 3 h 3
     
I¯x0 y0 = h + − h − = mh2
4 2 2 4 2 2 8
The moment exerted on the body about its mass center is
X * * * * 8 20 * 32 *
MG = r*Q/ ×FQ = (−2h *
ı − 0.5hk )×(20 *
ı − 8*
 + 10k ) = − *ı +  + k ft·lbs
G 3 3 3

186
*
To apply (2), we also need the angular momentum HG ,
h *
i
I¯x0 x0 ω1 − I¯x0 y0 ω2 − I¯x0 z 0 ω3 b 1 +
* h *
i
HG = I¯y0 y0 ω2 − I¯y0 z 0 ω3 − I¯y0 x0 ω1 b 2 + (4)
h i*
I¯z 0 z 0 ω3 − I¯z 0 x0 ω1 − I¯z 0 y0 ω2 b 3
2 
40 lbs 8 in 7 3 17 3 37
       
*
= (−5) − (4) *
ı + (4) − (−5) *
 + (−1) k
32.2 ft/s2 12 in/ft 24 8 12 8 24
*
= (−1.63 *  − 0.851k ) slug·ft2 /s
ı + 4.16 * (5)

Equation (2) may be rewritten as


*      *
 *
I¯x0 x0 α1 − I¯x0 y0 α2 *
ı + I¯y0 y0 α2 − I¯y0 x0 α1 *
 + I¯z 0 z 0 α3 k + ω×
X
*
MG = HG (6)

*
*
P
Substitution of (4), (5), and MG into (6) yields three equations that may be solved for α,
* *
*
α = α1 *
ı + α2*
 + α3k = (−9.10 *  + 29.3k ) rad/s2
ı + 9.47*

The acceleration of point P will be


 
a*P = a*G + α×
*
r*P/ + ω×
* *
ω× r*P/
G G
* * *
= (16.1 ı − 38.64  + 8.05k ) +
4* 2* 1*
 
* * *
(−9.10 ı + 9.47 + 29.3k )× ı +  + k +
3 3 3
4* 2* 1*
  
* *
(−5 *
ı + 4*
 − k )× (−5 * ı + 4*
 − k )× ı +  + k
3 3 3
*
a*P = (−34.6 *  − 20.3k ) ft/s2
ı − 41.9 *

187
8.8.11
*
GOAL: Find the angular acceleration α of the body, the acceleration a*G of the mass center, and
* *
the velocity and acceleration vQ and aQ of the point mass Q, all at time t = 0.
GIVEN: Geometry and mass of system components; the initial linear and angular velocities of the
system; the force applied to the system and its point of application.
DRAW:

ASSUME: The thickness of the cylindrical shell is negligible.


FORMULATE EQUATIONS: The equations of motion for a rigid body are

*
F = ma*G (1)


X * *
˙ d * *
*
MG = HG = HG + ω×HG (2)
N dt S

SOLVE: From (1), the acceleration of the mass center is

* *
F (−60 *
ı + 40 *
 − 50k ) N *
a*G = = = (−5.45 *  − 4.55k ) m/s2
ı + 3.64 * (3)
M + 2m 5 kg + 2(3 kg)

To compute the moments of inertia for the system we may superimpose the individual inertia
values for the two point masses and the cylindrical shell. Note that the origin of the given x, y, z

188
coordinate system is the current center of mass. By symmetry, I¯xz = I¯yz = 0.

L 2
 
I¯xx = 2
M R + 2m R + = (5 kg)(0.05 m)2 + 2(3 kg) (0.05 m + 0.20 m)2 =
4
= 0.3875 kg·m2
 2
1 1 L
I¯yy = 2
M R + M L + 2m2
=
2 12 4
1 1
= (5 kg)(0.05 m)2 + (5 kg)(0.80 m)2 + 2(3 kg)(0.20 m)2 = 0.5129 kg·m2
2 12 "
 2   #
1 1 L L 2 1
I¯zz = 2
M R + M L + 2m2
+ R+ = (5 kg)(0.05 m)2 +
2 12 4 4 2
1 h i
+ (5 kg)(0.80 m)2 + 2(3 kg) (0.20 m)2 + (0.25 m)2 = 0.8879 kg·m2
12
L L L L
     
I¯xy = m − R+ +m −R − = −2(3 kg)(0.20 m)(0.25 m) =
4 4 4 4
= −0.30 kg·m2
The moment exerted on the body about its mass center is
X * * * *
*
MG = r×F = (−0.20 *
ı + 0.25*
 )×(−60 *
ı + 40 *
 − 50k ) = −12.5 *
ı − 10 *
 + 7k N·m
*
To apply (2), we also require the angular momentum HG ,
h *
i
I¯xx ω1 − I¯xy ω2 − I¯xz ω3 b 1 +
* h *
i
HG = I¯yy ω2 − I¯yz ω3 − I¯yx ω1 b 2 + (4)
h i*
I¯zz ω3 − I¯zx ω1 − I¯zy ω2 b 3
*
= [(0.3875)(5) − (−0.3)(2)] *
ı + [(0.5129)(2) − (−0.3)(5)]*
 + [(0.8879)(−3)]k
*
= (2.54 *  − 2.66k ) kg·m2 /s
ı + 2.53 * (5)
Equation (2) may be rewritten as
*     *
 *
I¯xx α1 − I¯xy α2 *  + I¯zz α3 k + ω×
ı + I¯yy α2 − I¯yx α1 *
X
*
MG = HG (6)
*
*
P
Substitution of (4), (5), and MG into (6) yields three equations that may be solved for α,
* *
*
α = α1 *
ı + α2*
 + α3k = (−26.2 *  − 0.624k ) rad/s2
ı − 15.3*

The velocity and acceleration of the point mass Q will be


*
v*Q = v*G + ω×
*
r*Q/ = 0 + (5 *
ı + 2*
 − 3k )×(0.20 *
ı − 0.25*
)
G

*
v*Q = (−0.75 *  − 1.65k ) m/s2
ı − 0.60 *
 
a*Q = a*G + α×
*
r*Q/ + ω×
* *
ω× r*Q/
G G
* *
= (−5.45 ı + 3.64  − 4.55k ) + (−26.2 *
* *
ı − 15.3*
 − 0.624k )×(0.20 *
ı − 0.25*
)+
 
* *
+(5 *
ı + 2*
 − 3k )× (5 *
ı + 2*
 − 3k )×(0.20 *
ı − 0.25 *
)

189
*
a*Q = (−10.7 *  + 3.57k ) m/s2
ı + 14.0 *

190
8.8.12
*
GOAL: Determine the angular acceleration α of the T-bar, and the linear accelerations a*G and a*P
of the center of mass and point P , respectively, at the moment a given force F is applied.
GIVEN: We’re given the mass m and dimension h for the body, and the applied force F with its
location. The body is initially at rest.
DRAW:

ASSUME: The thickness of the T-bar is negligible and may be modeled as a slender rod.
FORMULATE EQUATIONS: The center of mass of the T-bar is located at

1 2 1 h h
  
*
xG = m(0) + m − * = − * = −0.05*
 m (1)
m 3 3 2 6

Let x0 , y 0 , z 0 be a set of axes that pass through the mass center and are parallel with the given x, y,
z. The products of inertia about the x0 , y 0 , z 0 axes will all equal zero, due to the symmetry of the
body (and thus Euler’s equations are generally applicable). In addition, since the body is initially
*
at rest, its angular velocity ω = 0 at the moment of interest. These conditions lead to a simplified
version of the angular equations of motion.

*
F = ma*G (2)


* *
˙ d * * *
HG = I¯x0 x0 α1 *
ı + I¯y0 y0 α2 *
 + I¯z 0 z 0 α3 k
X *
MG = HG = HG + ω× (3)
N dt S

SOLVE: The moments of inertia about the x0 , y 0 , z 0 axes are

2 h 2 1 m 2 m h 2 1
       
I¯x0 x0 = m + h + = mh2 = 0.0225 kg·m2
3 6 12 3 3 3 12
1 2 2
 
I¯y0 y0 = m (2h)2 = mh2 = 0.06 kg·m2
12 3 9
11
I¯z 0 z 0 = I¯x0 x0 + I¯y0 y0 = mh2 = 0.0825 kg·m2
36

The moment about the mass center is


X * * * *
MG = r*×F = (−0.15 *
ı + 0.05 *
 )×(−50 *
ı + 40*
 + 20k ) = ( *
ı + 3*
 − 3.5k ) N·m

191
Solving (3) for the angular acceleration components yields:

1 N·m
α1 = = 44.4 rad/s2
0.0225 kg·m2
3 N·m
α2 = 2
= 50 rad/s2
0.0600 kg·m
−3.5 N·m2
α3 = = −42.4 rad/s2
0.0825 kg·m2
*
*
α = (44.4 *  − 42.4k ) rad/s2
ı + 50*

The center of mass acceleration is computed from (2),


*
(−50 *
ı + 40*
 + 20k ) N *
a*
G = = (−16.7 *  + 6.67k ) m/s2
ı + 13.3*
3 kg

The acceleration of point P is then


 
a*P = a*G + α×
*
r*P/ + ω×
* *
ω× r*P/
G G
* *
= (−16.7 ı + 13.3 + 6.67k ) + (44.4 *
* *
ı + 50*
 − 42.4k )×(0.3 *
ı + 0.05*
)+0
*
a*P = (−14.5 *  − 6.11k ) m/s2
ı + 0.606*

192
8.8.13
GOAL: Determine the ratio (ω1 − ω2 )/(L1 − L2 ) that will result in a steady precession rate Ω
about the vertical, with shaft CD remaining in the horizontal plane.
GIVEN: The mass and dimensions of the system components are assumed to be known. The
angular velocities of the disks are assumed to be ω1 and ω2 , in the directions indicated by the
diagram.
* * * *
DRAW: Let b 1 , b 2 , b 3 be a set of unit vectors fixed to the shaft CD. b 3 will thus always be the
vertical upward direction.

ASSUME: The mass of the shaft is negligible.


FORMULATE EQUATIONS: We’ll choose to solve this problem by using the angular equations
of motion about the fixed point O.
X * *
˙
MO = HO (1)
SOLVE: The moments of inertia of the two disks about points O will be very similar. Note the
use of the parallel axis expressions in Iyy and Izz below.

1
A
Ixx = mR2 B
Ixx = 1
2 mR
2
2
1 1
A
Iyy = 2 2 B 2 + mL22
4 mR + mL1 Iyy = 4 mR
1 2 2 1 2
A
Izz = 4 mR + mL1
B
Izz = 4 mR + mL22

The products of inerta are all zero, by symmetry.


If the shaft remains horizontal, the angular velocity of each disk will be
*A * * *B * *
ω = ω1 b 1 + Ω b 3 ω = −ω2 b 1 + Ω b 3 (2)

Since we are using a set of principal axes, the angular momentum about O becomes
* *A *B * * * *
A A B B
HO = HO + HO = Ixx ω1 b 1 + Izz Ω b 3 − Ixx ω2 b 1 + Izz Ωb3 (3)
*
˙ * *
˙ * *
˙
A A B B
⇒ HO = Ixx ω̇1 b 1 + Ixx ω1b 1 + 0 − Ixx ω̇2 b 1 − Ixx ω2b 1 + 0
1 * 1 *
= mR2 (ω̇1 − ω̇2 ) b 1 + mR2 (ω1 − ω2 )Ω b 2 (4)
2 2

193
The only moment about O is that due to gravity
X * * * * * *
MO = L1 b 1 ×(−mg b 3 ) + (−L2 b 1 )×(−mg b 3 ) = mg(L1 − L2 ) b 2 (5)

Substitution of (4) and (5) into (1) yields

* 1 * 1 *
mg(L1 − L2 ) b 2 = mR2 (ω̇1 − ω̇2 ) b 1 + mR2 (ω1 − ω2 )Ω b 2 (6)
2 2
*
Equating the b 2 components gives the answer we’re looking for

ω1 − ω2 2g
=
L1 − L2 R2 Ω

*
Note that since disks A and B spin freely about the shaft, ω̇1 = 0 and ω̇2 = 0, satisfying the b 1
component of the above equation. Writing the equations of motion for each disk about its mass
center would quickly reveal that this is true. In fact, as an alternative solution we could have
employed these equations instead, along with a moment balance for the massless shaft.

194
8.8.14
GOAL: Examine the angular momentum and angular velocity of a discus after its release, and
how they change over time. Also examine the trajectory of the discus mass center and the distance
it is thrown.
GIVEN: Mass and dimensions of discus; initial position, orientation, linear velocity and angular
velocity.
DRAW:

ASSUME: The discus may be approximated as a circular disk with given mass and dimensions.
* * * *
SOLVE: Let b 1 , b 2 , b 3 be a set of body-fixed unit vectors with b 2 being the “spin” axis, or the
direction normal to the disk surface. Using our cicular disk approximation for the discus gives
moments of inertia equal to:
 2
1 d 1 1 1
I¯x0 x0 = m + mt2 = (2 kg)(0.11 m)2 + (2 kg)(0.045 m)2 = 0.00639 kg·m2
4 2 12 4 12
1 d 2 1
 
I¯y0 y0 = m = (2 kg)(0.11 m)2 = 0.0121 kg·m2
2 2 2
I¯z 0 z 0 = I¯x0 x0 = 0.00639 kg·m2
All products of inertia are zero, by symmetry.
(a) The initial angular momentum of the discus is
* * * *
HG = I¯x0 x0 ω1 b 1 + I¯y0 y0 ω2 b 2 + I¯z 0 z 0 ω3 b 3
* * *
= [(0.00639)(−1.3) b 1 + (0.0121)(20) b 2 + (0.00639)(0.8) b 3 ] kg·m2 /s
* * *
HG = (−8.30×10−3 b 1 + 0.242 *b 2 + 5.11×10−3 b 3 ) kg·m2 /s
Since there are no moments acting on the discus about its mass center, the rate of change of angular
P * *
˙ *
momentum is zero, MG = HG = 0. Thus HG is constant.
(b) From Euler’s equations, the motion of the discus is described by
I¯ 0 0 ω̇ + ω ω (I¯ 0 0 − I¯ 0 0 ) = 0
xx 1 2 3 zz yy
I¯y0 y0 ω̇2 + ω3 ω1 (I¯x0 x0 − I¯z 0 z 0 ) = 0 (1)
I¯z 0 z 0 ω̇3 + ω1 ω2 (I¯y0 y0 − I¯x0 x0 ) = 0

195
We can rearrange the above equations to yield (observe I¯x0 x0 − I¯z 0 z 0 = 0)

I¯y0 y0 − I¯z 0 z 0 I¯x0 x0 − I¯y0 y0


ω̇1 = ω2 ω3 ω̇2 = 0 ω̇3 = ω1 ω2 (2)
I¯ 0 0
xx
I¯ 0 0
zz

and note that ω̇1 = Cω3 and ω̇1 = −Cω3 , where C is a constant.
* * * * * * *
In general, if ω = ω1 b 1 + ω2 b 2 + ω3 b 3 , where b 1 , b 2 , b 3 are body-fixed unit vectors, the rate of
*
change of ω may be expressed as
*˙ * * *
ω = ω̇1 b 1 + ω̇2 b 2 + ω̇3 b 3 (3)

For the present case,

I¯y0 y0 − I¯z 0 z 0 *
I¯x0 x0 − I¯y0 y0 *
*
˙
ω = ¯ ω2 ω3 b 1 + ¯ ω1 ω2 b 3
Ix0 x0 Iz 0 z 0

*
Examining (2) and (3), we see that there are three release conditions for which ω is constant.

• ω1 = ω3 = 0; Only spin imparted to discus, ω2 6= 0


• ω2 = 0; No spin imparted. Rotation is about a fixed transverse axis.
• ω1 = ω2 = ω3 = 0; No rotation whatsoever, a special case of the above

(c) Neglecting air resistance, the discus will follow a typical parabolic trajectory for a projectile
under the influence of gravity. We can compute the distance of the throw by considering the vertical
and horizontal components of the motion equations. The time that will elapse before the discus
strikes the ground may be computed from the vertical component,
q
2 + 2gy
voy
voy o
t = +
g g
q
(25 m/s) sin 25◦ (25 m/s)2 (sin 25◦ )2 + 2(9.81 m/s2 )(1.5 m)
= + = 2.29 s
9.81 m/s2 9.81 m/s2

The throw distance is now computed from the horizontal component,

x = vox t = (25 m/s)(cos 25◦ )(2.29 s) = 51.8 m

Our computation has come up short compared to an actual Olympic throw with the same release
conditions. The reason for this is because we have neglected the lift provided by the 35◦ “attack”
angle. While the frictional resistance of air will generally decrease a projectile’s speed, the lift
provided under these particular conditions is large enough to actually increase the maximum height
and flight time for the discus. This increase in flight time is apparently the more significant effect,
allowing a properly-trained athlete to throw the discus farther than our prediction.

196
8.8.15
GOAL: (a) Find expressions for the vertical components of the reaction forces at the sphere’s
contact point C and at the support O; (b) Write an expression for the angle θ at which the contact
force goes to zero, and note any restrictions on m, R, and L for which this is impossible.
GIVEN: The mass of the sphere, length of the rod, radius of the sphere, and angle θ are all
assumed to be known. ω is constant.
DRAW:

* * *
ı  k
*
b1 cos θ 0 − sin θ
*
b2 0 1 0
*
b3 sin θ 0 cos θ
ASSUME: The mass of the rod may be neglected. We assume that when the vertical reaction force
goes to zero the sphere continues a “rolling” motion over the surface, although there is essentially no
contact. We will also assume that the horizontal component of the contact force in the * ı direction
is zero. This will be a valid assumption at the moment contact is lost, and without this assumption
the *ı -components of the reactions at C and O would be redundant. Of course, this assumption
would also hold for a very smooth surface (we would just have to initiate “rolling” somehow).
FORMULATE EQUATIONS: We will choose to write our moment equation about the center
of mass. The point O would work just as well, and we could even choose C, with a little more
work. X*
F = ma*G (1)

X * *
˙ d * *
*
MG = HG = HG + ω×HG (2)
N dt S

We are given the rotation rate ω about the vertical axis. Since the sphere is rolling its motion also
consists of a rotation about the shaft, which we will denote as a rate η. The angular velocity of
* * *
the sphere is therefore ω = ωk + η b 3 . Since the point C is stationary, we may write a constraint
condition relating ω and η:
* * * * * *
v*C = ω×
*
r*C/ = (ωk + η b 3 )×(−L b 3 − Rk ) = −ωL sin θ b 2 + ηR sin θ b 2 = 0
O

⇒ ωL = ηR (3)

The locus of contact points on the sphere is a circle of radius R sin θ.

197
SOLVE: The moments of inertia for a sphere about its mass center are identical, and all products
of inertia about axes through G are zero.
2
I¯xx = I¯yy = I¯zz = mR2
5
* * * * *
With ω = ωk + η b 3 = −ω sin θ b 1 + (ω cos θ + η) b 3 , the angular momentum of the sphere becomes:
* 2 * 2 *
HG = − mR2 ω sin θ b 1 + mR2 (ω cos θ + η) b 3 (4)
5 5
*
˙ 2 * 2 *
⇒ HG = − mR2 ω sin θ(ω cos θ) b 2 + mR2 (ω cos θ + η)(ω sin θ) b 2
5 5
2 2 *
= mR ωη sin θ b 2 (5)
5
* * * *
Let the reaction forces at O and C be FC = FC x *
ı + FC y *
 + FC zk , and FO = FOx *
ı + FO y *
 + FOzk .
* * *
The moment at O will consist of a single component in the b 1 -direction, MO = MO1 b 1 . The other
*
two components are zero because the system is pinned horizontally about b 2 , and the massless
*
shaft allows free rotation of the sphere about b 3 .
X * * * * * *
MG = (L b 3 )×(FOx *
ı + FO y *
 + FOzk ) + (−Rk )×(FC x *
ı + FC y *
 + FC zk ) + MO1 b 1
* *
= (RFC y cos θ − FOy L + MO1 ) b 1 + (−RFC x + FOx L cos θ − FOz L sin θ) b 2 +
*
+RFC y sin θ b 3 (6)
Equating components of (5) and (6) gives
0 = RFC y cos θ − FOy L + MO1 (7)
2
mR2 ωη sin θ = −RFC x + FOx L cos θ − FOz L sin θ (8)
5
0 = RFC y sin θ (9)
The sphere’s center travels in a circle of radius L sin θ, with angular speed ω. It’s acceleration is
a*G = mLω 2 sin θ *
ı and our force balance (1) yields the component equations
mLω 2 sin θ = FC x + FOx (10)
0 = FC y + FO y (11)
0 = FC z + FOz − mg (12)
From (9), (11), and (7) we see that FC y = 0, FOy = 0, and MO1 = 0. Interesting, but not what we
need. The relevant equations for our purposes are (8), (10), and (12). With our assumption that
FC x = 0 (as it must when the sphere loses contact, or on a very smooth surface), these equations
may be solved for the vertical reactions at C and O,
2
(10),(3)→(8) ⇒ mRLω 2 sin θ = mL2 ω 2 sin θ cos θ − FOz L sin θ
5

2
 
⇒ FOz = mω 2 L cos θ − R (13)
5

2
 
2
(13)→(12) ⇒ FC z = mg − mω L cos θ − R (14)
5

198
(b) Letting FC z go to zero in equation (14) and solving for θ yields

1 g 2
 
cos θ = 2
+ R
L ω 5

Since cos θ ≤ 1, we have


g 2
+ R<L
ω2 5
Thus no matter how large ω may be, it will be impossible for the vertical reaction to equal zero
unless
2
R<L
5
p
There is also a lower limit on ω, namely, ω > g/L.

199
8.8.16
GOAL: Determine the angular speed ω, and its direction, that is required if the angle θ between
the vertical and shafts AC and BD is to remain constant at θ = 30◦ , for a given rotational rate Ω.
GIVEN: Each disk has mass m = 5 kg and radius R = 0.08 m; shafts AC and BD have length
L1 = 0.20 m with spring attachment points at a distance L2 = 0.15 m from points C and D;
the length of link CD is L3 = 0.30 m, which is the same as the unstretched length of the spring
(k = 320 N/m); Ω = 3 rad/s in the direction shown.
DRAW:

* * *
ı  k
*
b1 cos θ 0 − sin θ
*
b2 0 1 0
*
b3 sin θ 0 cos θ
ASSUME: The mass of the spring and linkages is negligible.
FORMULATE EQUATIONS: Since the system is symmetric, we can confine our work to only
one disk and linkage. The linear and angular equations of motion for rigid bodies are
X*
F = ma*G (1)

X * *˙ d * *
*
MG = HG = HG + ω×HG (2)
N dtS

200
SOLVE: The shaft is massless, so the only moments of inertia to consider are that of the disk.
* * *
Since the disks are axisymmetric, we may use the shaft-fixed b 1 , b 2 , b 3 directions as “body-fixed”
unit vectors without loss of generality. All products of inertia are zero.
1 1 1
I¯xx = mR2 I¯yy = mR2 I¯zz = mR2
4 4 2
With θ constant, the angular velocity of the disk is
* * * * *
ω = Ωk + ω b 3 = −Ω sin θ b 1 + (Ω cos θ + ω) b 3 (3)
The angular momentum of the disk is then
* * *
HG = −I¯xx Ω sin θ b 1 + I¯zz (Ω cos θ + ω) b 3
1 * 1 *
= − mR2 Ω sin θ b 1 + mR2 (Ω cos θ + ω) b 3 (4)
4 2
*
˙ 1 * 1 *
HG = − mR Ω sin θ(Ω cos θ b 2 ) + mR2 (Ω cos θ + ω)(Ω sin θ b 2 )
2
4 2
1 2
= mR Ω sin θ(Ω cos θ + 2ω) (5)
4
*
Let the reaction force at point D be Fx *ı + Fy *
 + Fzk . The link is pinned at D so that there is no
*
moment in the b 2 -direction; the negligible mass of the shaft and the fact that the disk spins freely
*
about is axis indicates that there can be no moment about b 3 either. Thus the reaction moment is
*
just M1 b 1 .
X * X * *
MG = ( r*×F) + M1 b 1
* * * *
= −L1 b 3 ×(Fx *
ı + Fy *
 + Fzk ) + −(L1 − L2 ) b 3 ×(−2kL2 sin θ) *
ı + M1 b 1
* *
= (L1 Fy + M1 ) b 1 + [L1 Fz sin θ − L1 Fx cos θ + 2k(L1 − L2 )L2 sin θ cos θ] b 2 (6)
* *
Substituting (5) and (6) into (2), and then equating b 1 and b 2 components gives
0 = L1 Fy + M1 (7)
1
mR2 Ω sin θ(Ω cos θ + 2ω) = L1 Fz sin θ − L1 Fx cos θ + 2k(L1 − L2 )L2 sin θ cos θ (8)
4
The mass center travels a circular path of radius L3 /2+L1 sin θ with constant speed. Its acceleration
is * * *
a*G = Ωk ×[Ωk ×(L3 /2 * ı + L1 b 3 )] = −Ω2 (L3 /2 + L1 sin θ) *
ı
The component equations of our force balance (1) are
−mΩ2 (L3 /2 + L1 sin θ) = Fx − 2kL2 sin θ (9)
0 = Fy (10)
0 = Fz − mg (11)
By (10) and (7), Fy = 0 and M1 = 0. Solving (9) and (11) for Fx and Fz and substituting into (8)
yields:
1
mR2 Ω sin θ(Ω cos θ + 2ω) = L1 mg sin θ + 2k(L1 − L2 )L2 sin θ cos θ
4
−L1 [2kL2 sin θ − mΩ2 (L3 /2 + L1 sin θ)] cos θ (12)
Using the given parameter values in (12) allows us to solve for ω,
*
ω = 24.5 rad/s, in the positive b 3 direction

201
8.8.17
*
GOAL: (a) Determine ω̇1 , ω̇2 , and the angular acceleration α at the instant a given force is applied
to the cylinder. (b) Find the reaction forces and moments at the support A at this instant.
GIVEN: Mass and dimensions of system; initial rotation rates ω1 , ω2 ; and the applied force F
and its location.
DRAW:

* * * *
The b 1 , b 2 , b 3 axes are attached to the shaft, with b 2 remaining vertical.
ASSUME: The mass of the 90◦ -bend shaft is negligible.
FORMULATE EQUATIONS: We employ the rigid body equations of motion. We will choose
to sum the applied moments about the fixed point O lying at the center of the 90◦ -bend.
X*
F = ma*G (1)

X * *
˙
d * *
*
MO = HO = HO + ω×HO (2)
N dt S
SOLVE: The moments of inertia for the cylinder about its mass center are I¯xx = 12 mR2 and
I¯yy = I¯zz = 14 mR2 + 12
1
mt2 . Since we have chosen the fixed point O we’ll use the parallel axis
expressions to obtain the inertia values about this point.

1
Ixx = mR2
2
1 1 t 21 1
 
Iyy = mR2 + mt2 + m L + mR2 + mt2 + mL2 + mLt
4 12 2 4 3
2
1 1 t 1 1

Izz = mR2 + mt2 + m L + = mR2 + mt2 + mL2 + mLt
4 12 2 4 3

The angular velocity of the cylinder is


* * *
ω = ω1 b 1 + ω2 b 2 (3)

The angular momentum of the system about O is

1 1 1
 
* * *
HO = mR2 ω1 b 1 + mR2 + mt2 + mL2 + mLt ω2 b 2 (4)
2 4 3

202
Taking the derivative,
1 1 1 1
 
*
˙ * * *
HO = mR2 ω̇1 b 1 + mR2 ω1 (−ω2 b 3 ) + mR2 + mt2 + mL2 + mLt ω̇2 b 2 (5)
2 2 4 3
* * *
The forces acting on the system are gravity −mg b 2 , the applied force F = 40 b 3 N, and the reaction
forces. Since the shaft is assumed to be massless, the net forces and moments acting on any portion
of it must be zero (otherwise it would have infinite acceleration terms). It will be easier to visualize
the reactions if we initially consider only those existing at point O of the shaft. Doing so also
removes them from the moment equation. We’ll wait until part (b) below to consider those existing
at point A.
* * * *
The reaction force at O may be written as FO = FO1 b 1 + FO2 b 2 + FO3 b 3 . The reaction moment at
* * *
O consists of only one component, MO = MO3 b 3 . Since the cylinder spins freely about b 1 there can
*
be no reaction torque in this direction; and since the shaft revolves freely about b 2 there can be no
reaction torque in this direction either. The sum of the moments about O is now
t *
 
X * * * * * *
MO = L+ b 1 ×(−mg) b 2 + (L b 1 + R b 2 )×F b 3 + MO3 b 3
2
t *
  
* *
= RF b 1 − LF b 2 + MO3 − mg L + b3 (6)
2
Substitution of (5) and (6) into (2) and equating like components gives
1
mR2 ω̇1 = RF (7)
2
1 1
 
mR2 + mt2 + mL2 + mLt ω̇2 = −LF (8)
4 3
1 t
 
− mR2 ω1 ω2 = MO3 − mg L + (9)
2 2
The acceleration of the cylinder’s mass center is
t * t * t 
        
* * * * *
a*G = ω̇2 b 2 × L + b 1 + ω2 b 2 × ω2 b 2 × L + b1 = L + −ω̇2 b 3 − ω22 b 1
2 2 2
The component equations of the force balance (1) are now
t
 
FO1 = −mω22 L + (10)
2
FO2 − mg = 0 (11)
t
 
FO3 + F = −mω̇2 L + (12)
2
Equations (7)-(12) are our six rigid body equations of motion. Equations (7) and (8) may be solved
directly for ω̇1 and ω̇2 :
2F 2(40 N)
ω̇1 = = = 320 rad/s2 (13)
mR (5 kg)(0.05 m)

ω̇2 =−1 LF
1 2
mR + mt + mL2 + mLt
2
4 3
(0.15 m)(40 N)
=− 
1 1

2 2 2
(5 kg) (0.05 m) + (0.10 m) + (0.15 m) + (0.15 m)(0.10 m)
4 3

203
ω̇2 = −28.9 rad/s2 (14)
*
We can now solve for the angular acceleration α. Note ω1 = 120 rev/min = 4π rad/s, and ω2 =
4
40 rev/min = 3 π rad/s

4 *
 
* *˙ * * * * *
α =ω = ω̇1 b 1 + ω̇2 b 2 − ω1 ω2 b 3 = 320 b 1 − 28.9 b 2 − (4π) π b3
3
* * *
*
α = (320 b 1 − 28.9 b 2 − 52.6 b 3 ) rad/s2 (15)

(b) Equations (9)-(12) may be solved for the force and moment reactions at O. To find the reactions
at A, we just have to balance the forces and moments acting on the AO segment of the shaft.
Again, since the shaft is assumed massless these forces and moments must cancel. We’ll let the
* * * * * * *
force reaction at A be FA = FA1 b 1 + FA2 b 2 + FA3 b 3 , and the moment reaction MA = MA1 b 1 + MA3 b 3 .

t
 
FA1 = FO1 = −mω22 L+ = −17.5 N
2
FA2 = FO2 = mg = 49.1 N
t
 
FA3 = FO3 = −F − mω̇2 L+ = −11.06 N
2

* * * *
FA = (−17.5 b 1 + 49.1 b 2 − 11.06 b 3 ) N

Summing the moments for massless segment AO about O, which must equal zero,
X * * * * * * * *
M = (−L) b 2 ×(FA1 b 1 + FA2 b 2 + FA3 b 3 ) + MA1 b 1 + MA3 b 3 − MO3 b 3 = 0

⇒ MA1 = LFA3 = −1.659 N·m


 
t
MA3 = MO3 − LFA1 = mg L + 2 − 12 mR2 ω1 ω2 − LFA1 = 12.1 N·m

* * *
MA = (−1.659 b 1 + 12.1 b 3 ) N·m

204
8.8.18
GOAL: Find the tension in the string, and the reaction forces at each ball-and-socket joint, when
mass m3 has descended a distance s = 29 πr2 from the given release position.
GIVEN: m1 = 3 kg, m2 = 0.5 kg, m3 = 2 kg, r1 = 10 cm, r2 = 4 cm, d = 6 cm, and
L1 = L2 = L3 = 15 cm.
* * *
DRAW: Let b 1 , b 2 , b 3 be a set of body-fixed unit vectors attached to the rotating shaft. Initially,
* * * *
b 1 , b 2 , b 3 and *
ı,*
 , k coincide.

* * * * s
The rotational transformation from *
ı,*
 , k to b 1 , b 2 , b 3 , with θ = r2 , is

* * *
ı  k
*
b1 cos θ 0 − sin θ
*
b2 0 1 0
*
b3 sin θ 0 cos θ

FORMULATE EQUATIONS: The system is viewed as two rigid bodies, the first being the
rod, disk, and pulley; the second being the hanging mass m3 . The governing equations are the
moment and force balances for the shaft, and a force balance for the hanging mass. There are eight
unknowns: six reaction force components at A and B, the tension T in the string, and the motion

205
of the system. The moment and force balances will provide 7 equations, but we will see that two
force components merely cancel, leaving 6 equations in 6 unknowns.
It is convenient to write the moment balance about a fixed point along the shaft axis. We will
choose the point B, since this choice eliminates the reactions at B from the moment equations.
Our governing equations are then:
X * *
˙
shaft: MB = HB (1)
X*
F shaft = (m1 + m2 )a*G−shaft (2)
hanging mass: T − m3 g = m3 a (3)
As an alternative approach, the force equation for the shaft (2) could easily be replaced by a second
moment balance, perhaps about A, to solve for the reactions at B.
* *
SOLVE: The angular velocity of the shaft may be written ω = ω* = ω b 2 . With only one rotational
component, the expression for the angular momentum of the shaft becomes:
* * * *
HB = Iyy ω b 2 − Ixy ω b 1 − Izy ω b 3 (4)
The required moments of inertia may be computed using the known values for a disk, the parallel
axis theorem, and the additive property for composite bodies:

1 1
   
Iyy = 1
Iyy + 2
Iyy = I¯10 + m1 d 2
+ I¯200 + m2 (0) = m1 r12 + m1 d2 + m2 r22
y y0 y y 00 2 2
   
Ixy = 1
Ixy + 2
Ixy = I¯10 + m1 (0)(2L) + I¯200 + m2 (0)(L) = 0+0
x y0 x y 00
   
Izy 1
= Izy 2
+ Izy = I¯10 + m1 (−d)(2L) + I¯200 + m2 (0)(L) = 0 − 2m1 dL + 0
z y0 z y 00

The right side of equation (1) is now


*
˙ * * *
HB = Iyy α b 2 − Izy α b 3 − Izy ω 2 b 1
1 1
   
* * *
= m r2 + m1 d2 + m2 r22 α b 2 + 2m1 dL α b 3 + ω 2 b 1 (5)
2 1 1 2
* * * *
Let the reactions at A and B be written as FA = FAx *
ı + FAy *
 + FAzk and FB = FBx * ı + FBy *
 + FBzk .
There are no constraint torques since the ball-and-socket joints rotate freely in all directions. Also
denote L = L1 = L2 = L3 . The moment about B is then
X * * * * *
MB = r*A/ ×FA + r*G1/ × −m1 g k + r*G2/ × −m2 g k + r*C/ ×−Tk
B B B B
 *  *  * 
= −3L *
 × FAx *
ı + FAy *
 + FAzk + −2L *
 + d b 3 × −m1 g k +
 *  * 
−L *
 × −m2 g k + −L *
 + r2 *
ı × −T k
    *
ı + m1 gd sin rs + T r2 *
= −3LFAz + 2Lm1 g + Lm2 g + T L *  + 3LFAxk (6)
2

*
Substitution of (5) and (6) into (1), and taking dot products with *
ı,*
 , k , yields the first three
equations of motion:
 
(1) · *
ı : −3LFAz + 2Lm1 g + Lm2 g + T L = 2m1 dL α sin rs + ω 2 cos rs (7)
2 2

1 1
 
(1) · *
 : m1 gd sin rs + T r2 = m1 r12 + m1 d2 + m2 r22 α (8)
2 2 2
*
 
(1) ·k : 3LFAx = 2m1 dL α cos rs − ω 2 sin rs (9)
2 2

206
The force balance for the shaft, as written in Eqn. (2), requires the acceleration of the center of
mass of the disk and pulley. However, since the pulley’s center of mass remains stationary, we
can balance the external forces acting on the system using the motion of the disk center alone.
Equation (2) then becomes: X*
F shaft = m1 a*G1
where
* * *
 * *

a*G1 = α×
*
r* + ω×(
* *
ω× r*) = α b 2 ×d b 3 + ω b 2 × ω b 2 ×d b 3
 * *

= m1 d α b 1 − ω 2 b 3

Using the above force balance, the three force equations for the shaft are then
 
*
ı : FAx + FBx = m1 d α cos rs − ω 2 sin rs (10)
2 2
*
 : FAy + FBy = 0 (11)
*
   
k : FAz + FBz − m1 + m2 g − T = m1 d −α sin rs − ω 2 cos rs (12)
2 2

* *
The acceleration of the hanging mass is a* = ak = −αr2 k . Using this result in (3) gives

T − m3 g = −m3 r2 α (13)

We now have all equations necessary for the solution. Specifically, these are equations (7)-(13).
Equations (8) and (13) govern the motion of the system and may be solved for α:
m1 gd sin rs + m3 gr2
2
α= 1 (14)
2
2 m1 r 1 + m1 d2 + 21 m2 r22 + m3 r22

We also need to determine ω, which may be found by integrating our expression for α. To simplify
this task we will first apply the chain rule to obtain the relation α = ṡ dw dw α
ds = ωr2 ds ⇒ r2 ds = ωdω.
Substitution into this relation, with dummy variables s0 and ω 0 , yields:
Zω Z s
1 m1 gd s0

0 0
ω dω = 1 2 2 1 2 2
sin + m3 g ds0
2 m1 r1 + m1 d + 2 m2 r2 + m3 r2
r2 r2
0 0
 
m1 gd 1 − cos rs + m3 gs
⇒ ω2 = 1
2
(15)
2
4 m1 r1 + 12 m1 d2 + 14 m2 r22 + 21 m3 r22
With the given mass and length parameters, equations (14) and (15) will give us the angular velocity
and acceleration at the instant s = 29 πr2 :

α = 86.8 rad/s2
ω = 29.6 rad/s

Using these values in equations (7),(9), and (13), with s = 92 πr2 , we can solve for the tension T
and the reactions FAx and FAz :

T = 12.7 N; FAx = −105.0 N; FAz = 15.1 N

Inserting these values in equations (10) and (12) we find FBx and FBz :

FBx = −52.5 N; FBz = 16.3 N

207
The remaining equation of motion, (11), only tells us that

FAy = −FBy

That is, the y-components of the reaction forces at A and B must cancel. With no externally
applied forces in this direction, and no preexisting stresses in the shaft along the y-axis, these
forces should remain zero.

208
8.8.19
GOAL: Determine the precession rate and rotational kinetic energy of a football during flight.
GIVEN: Weight and geometry of football; the spin rate is 15 rad/s about the long axis of the
football and the long axis itself wobbles through a total angle of 15◦ .
DRAW:

FORMULATE EQUATIONS: Since there are no external moments acting on the football its
angular momentum is conserved. We can determine the precession rate by setting
X * *
˙
MG = HG = 0 (1)

The rotational kinetic enegy may be found using


1* *
KE rot = ω · HG (2)
2
SOLVE: Modeling the football as a solid prolate spheroid, its has two equal semi-minor axes
a = b = (19 in)/(2π) = 3.024 in. and a semi-major axis c = 5 in. First let’s get some unit
conversions out of the way,
19 in 5 in
a = b = = 0.252 ft c = = 0.416̄ ft
2π(12 in/ft) 12 in/ft
9 oz
m = = 0.0175 slug
(16 oz/lb)(32.2 ft/s2 )

The moments of inertia of an ellipsoid about principal axes through its mass center are
1 2
I¯xx = m(a2 + b2 ) = ma2 = 4.44×10−4 slug·ft2
5 5
1
I¯yy = I¯zz = m(a2 + c2 ) = 8.28×10−4 slug·ft2
5
The angular velocity of the football may be expressed as
* * * *
ω = ω2 b 1 + ω1 *
ı = (ω2 + ω1 cos θ) b 1 − ω1 sin θ b 2
*
We may now write an expression for the angular momentum HG ,
* * *
HG = I¯xx (ω2 + ω1 cos θ) b 1 + I¯yy (−ω1 sin θ) b 2

209
Taking the derivative with respect to time, and setting it equal to zero
*
˙ *˙ *˙
⇒ HG = I¯xx (ω2 + ω1 cos θ)b 1 − I¯yy ω1 sin θb 2
* *
= I¯xx (ω2 + ω1 cos θ)ω1 sin θ b 3 − I¯yy ω12 sin θ cos θ b 3 = 0 (3)

Note that in the above we have assumed ω̇1 = 0 and ω̇2 = 0. Had these terms been included we
* *
would have seen some b 1 and b 2 components in the above equation, but with no external moments
we would quickly find that they are zero. Solving (3) for ω1 yields

I¯xx
!
ω2 2a2 ω2 2(3.024 in)2 (15 rad/s)
ω1 = = =
I¯yy − I¯xx cos θ (c2 − a2 ) cos θ [(5 in)2 − (3.024 in)2 ] cos 15◦

ω1 = 17.9 rad/s

The rotational kinetic energy is computed from (2).


1* *
KE rot = ω · HG
2
1h * *
i h * *
i
= (ω2 + ω1 cos θ) b 1 − ω1 sin θ b 2 · I¯xx (ω2 + ω1 cos θ) b 1 − I¯yy ω1 sin θ b 2
2
1h¯ i
= Ixx (ω2 + ω1 cos θ)2 + I¯yy (ω1 sin θ)2
2
1h
= (4.44×10−4 slug·ft2 )[15 rad/s + (17.9 rad/s) cos 15◦ ]2 +
2 i
+(8.28×10−4 slug·ft2 )[(17.9 rad/s) sin 15◦ ]2

KE rot = 0.240 ft·lb

210
8.8.20
GOAL: Determine the direction of rotation that the student experiences after rotating the axle of
the wheel as described.
GIVEN: The wheel is given an intial spin ω about its axle in the direction indicated in the figure.
There is no initial rotation of the swiveling platform. The student then rotates the wheel axle so
his right hand is directly above his left, with the axle of the wheel oriented vertically in space.
DRAW:

SOLVE: Consider the mechanical “system” consisting of the student, wheel, and platform. The
only external forces acting on the system are gravity and the reactions at the platform support.
Since the platform swivels freely about the z-axis, no external moments may be applied to the
system about this axis (or any point lying on the z-axis). The resultant of any external forces
acting on the platform must have a line of action that passes through the z-axis, or else be purely
vertical, parallel to this axis. Gravitational forces are assumed to be purely vertical as well.
Since no external moments may be applied to our system about the z-axis, the angular momentum
*
of the system about any point lying on this axis is conserved in the k -direction. This is clear from
the angular momentum equation of motion:
X * *
˙
MO = HO
*
˙ *
⇒ 0 = HO · k

Initially, the wheel axle is oriented horizontally, and the student is at rest. The system’s initial
*
angular momentum component in the k -direction is therefore zero, and it must remain zero for all

211
time. As the student tilts the axle of the wheel, bringing his right hand above the left, the wheel
*
itself begins to acquire an angular momentum component in the positive k direction. To counter
this positive gain, the student and platform must begin rotating in the opposite direction, at an
angular speed which maintains the system’s overall angular momentum component in this direction
at zero. Thus,
Student rotates in -z direction, or Ω is negative
In the solution above, we have only addressed the vertical direction, and have made no mention of
the horizontal components. So the question might be asked, “What about the change in angular
momentum in the horizontal direction? Initially the wheel has a large component in this direction,
but in the final orientation it has none.” The answer is that while the horizontal angular momen-
tum components do change, the platform support provides the external moments allowing for this
change, and the student’s muscles provide the torques required for him to remain upright.

212
8.8.21
GOAL: (a) Determine the angle φ through which the student and platform rotate before coming
to rest, after the described action is perfomed. (b) Find the initial and final kinetic energy of the
system, as well as the work done by friction, and explain any differences.
GIVEN: Mass and moments of inertia of the wheel; initial rotation rate ω of wheel and its direction;
magnitude of the torsional friction; distance d between vertical axis and wheel center; approximate
moment of inertia I for student/platform combination about vertical z-axis.
DRAW:

ASSUME: The described action is performed instantaneously. The student does not shift the
position of his mass center or that of the wheel. Neglect any friction in the wheel axle.
*
FORMULATE EQUATIONS: The wheel itself spins freely about the body-fixed b 1 axis, and
*
thus its angular rotation rate ω is constant. This is evident from an examination of the b 1 -
component of Euler’s equation
* disk
= I¯xx α1 + ω2 ω3 (I¯zz − I¯yy )
X
M1
⇒ 0 = I¯xx ω̇ + ω2 ω3 (0) ⇒ ω̇ = 0

The angular velocity of the wheel after the rotation of its axle will therefore be
* * *
ωwheel = ω b 1 = ωk (1)

To find the angle through which the platform rotates we will have to determine the intial rotation
*
rate Ω of the platform, and apply our angular equation of motion in the k direction
X * * *
˙ *
MP · k = HP · k (2)

to find Ω̇, and then integrate to get φ.


SOLVE: Since it is assumed that the student performs the 90◦ rotation of the wheel axis instan-
taneously, the torsional friction does no work and the angular momentum of the system about the
vertical z-axis is conserved during this action. Initially, the student is at rest and the wheel axle is

213
* * *
horizontal, so the angular momentum about the k direction is zero (HP · k = 0). We can use this
fact to determine the rotational rate Ω of the student immediately after the axle rotation:
* * * * *
HP · k = IΩ + HG ,wheel · k + ( r*G−G/ ×mv*G−wheel ) · k = 0
P (3)
= IΩ + Ixx ω + md2 Ω = 0

Ixx ω (0.2 kg·m2 )(18 rad/s)


⇒ Ω0 = − = − = −2.39 rad/s (4)
I + md2 0.9 kg·m2 + (2 kg)(0.55 m)2
*
We now apply equation (2), where the only moment about point P in the k -direction is the torsional
friction Tf , in the direction opposite the rotation Ω,
X * * *
˙ *
MP · k = HP · k
Tf = (I + md2 )Ω̇
Tf
⇒ Ω̇ = = 0.133 rad/s2 (5)
I + md2
Integrating,
Z Z
Ω dΩ = Ω̇ dφ
1 2
⇒ (Ω − Ω20 ) = Ω̇(φ − φ0 )
2
Set Ω = 0 to determine the angle through which the platform rotates before coming to rest.
Ω20 (Ixx ω)2 (−2.39 rad/s)2
φ − φ0 = − = − = −
2Ω̇ 2Tf (I + md2 ) 2(0.133 rad/s2 )
Letting φ0 = 0,
φ = −21.5 rad
(b) The kinetic energy of the system before the handlebars are rotated and after the platform comes
to rest will be the same since the only motion in each case is the rotation ω of the wheel, which
has constant amplitude.
1* * 1 * * 1
KE initial = KE f inal = ω · HG = ω b 1 · Ixx ω b 1 = I ω2
2 2 2 xx
1
KE initial = KE f inal = (0.2 kg·m2 )(18 rad/s)2 = 32.4 J
2
The work done by the friction torque is
* * (Ixx ω)2
Wf = Tfk · φk = −
2(I + md2 )

Wf = (0.2 N·m)(−21.5 rad) = −4.31 J

The initial kinetic energy of the system is equal to the final kinetic energy, although there has
been negative work done by friction. The explanation for this is that the internal muscular forces
generated by the student when tilting the wheel added energy to the system. This exact amount of
energy was then dissipated by friction to bring the platform to rest. In fact, we could have logically
computed the angle φ simply by determining the amount of rotation required to absorb the enery
added by the student when he tilted the wheel axle.

214
8.8.22
GOAL: Determine the ground reaction moment at point O.
GIVEN: The given parameters are the mass and geometry of system. The system revolves about
* * *
the vertical with a constant angular velocity ω = 30k rev/min = πk rad/s.
* * *
DRAW: The body-fixed b 1 , b 2 , b 3 unit vectors are attached to the rotating support beam. They
*
coincide with the absolute *ı,*
 , k unit vectors at the instant depicted in the diagram.

ASSUME: Neglect the mass of the support beam; consider only the hemispherical shells.
FORMULATE EQUATIONS: The angular momentum equation of motion for rigid bodies will
allow us to compute the reaction moment at O.

X * *
˙ d * *
*
MO = HO = HO + ω×HO (1)
N dt S

SOLVE: The angular momentum of the structure about point O is


h i*
Ixx ω1 − Ixy ω2 − Ixz ω3 b 1 +
* h i* * * *
HO = Iyy ω2 − Iyz ω3 − Iyx ω1 b 2 + = −Ixz ω3 b 1 − Iyz ω3 b 2 + Izz ω3 b 3 (2)
h i*
Izz ω3 − Izx ω1 − Izy ω2 b 3

where ω1 = ω2 = 0 and ω3 = π rad/s.


*
Since the angular velocity is constant, α = 0, and (1) becomes
X * * * * * *
*
MO = ω×HO = ω3 b 3 ×(−Ixz ω3 b 1 − Iyz ω3 b 2 + Izz ω3 b 3 )
* *
= −Ixz ω32 b 2 + Iyz ω32 b 1 (3)

215
We therefore only need to compute the two products of inertia Ixz and Iyz . Since the x-z plane
is a plane of symmetry for the structure, Iyz = 0, leaving us with only Ixz to compute. The mass
* *
center of the upper sphere is located at 2r b 1 + L2 b 3 while that of the lower sphere is located at
* *
− 2r b 1 + L1 b 3 . Using the parallel axis expressions and the superposition principle for composite
bodies allows us to compute Ixz . By symmetry, I¯ 0 0 = I¯ 00 00 = 0 we
xz x z

r r 1
  
I¯y00 y00 + m L22 + Ixz = I¯x00 z 00 + mL1 − + I¯x0 z 0 + mL2 = mr(L2 − L1 )
2 2 2

The reaction moment at O may now be computed from (3):


X * * 1 *
MO = −Ixz ω32 b 2 = − mr(L2 − L1 )ω32 b 2
2
!
1 250 lb *
= − 2 (4 ft)(24 ft − 16 ft)(π rad/s)2 b 2
2 32.2 ft/s

*
MO = −1.23×103*
 ft·lb

216
8.8.23
GOAL: Determine the angular speed ω1 required for the support force between brace E and arm
BC to go to zero, for a given ω2 .
* *
GIVEN: Geometry and dimensions of system; the angular velocity ω 2k of the vertical shaft AB.
* * *
DRAW: Let b 1 , b 2 , b 3 be a set of unit vectors fixed to arm BC.

* * *
ı  k
*
b1 cos θ 0 sin θ
*
b2 0 1 0
*
b3 − sin θ 0 cos θ
ASSUME: Neglect the mass of arm BC.
FORMULATE EQUATIONS: We will apply the angular equations of motion for a rigid body
about point B. We choose point B, instead of the mass center G, simply because it will eliminate
the reaction forces at B from our equations and avoid a consideration of the force equations.

X * *
˙
d * *
*
MB = HB = HB + ω×HB (1)
N dt S
SOLVE: The angular velocity of the disk is
* * * * *
ω = ω2k + ω1 b 3 = ω2 sin θ b 1 + (ω2 cos θ + ω1 ) b 3
*
To apply (1), we need to determine the angular momentum HB . The moments of inertia of the disk
*
about the b i axes through its mass center are I¯xx = I¯yy = 41 mR2 and I¯zz = 12 mR2 . We utilize the
parallel axis expressions to determine the inertia values about point B:
1 1 1
Ixx = mR2 + mL2 Iyy = mR2 + mL2 Izz = mR2
4 4 2
Ixy = Ixz = Iyz = 0 (by symmetry)

217
The angular momentum is now
* * *
HB = Ixx ω2 sin θ b 1 + Izz (ω2 cos θ + ω1 ) b 3
1 1
   
* *
= mR2 + mL2 ω2 sin θ b 1 + mR2 (ω2 cos θ + ω1 ) b 3 (2)
4 2
*
The system is subject to the condition that ω1 , ω2 , and θ are constant. The time derivative of HB
is therefore
*
˙ * * * * *
HB = ω2k ×HB = ω2 (sin θ b 1 + cos θ b 3 )×HB
1 1
   
* *
= mR2 + mL2 ω22 sin θ cos θ b 2 − mR2 (ω2 cos θ + ω1 )ω2 sin θ b 2 (3)
4 2
* * *
With arm BC pinned about b 2 , the moment at B may be represented as M1 b 1 + M3 b 3 (although
intuitively we know these are both zero due to the steady motion of the system and the fact that
the disk spins freely about its axis). When the contact force at the brace E goes to zero it will
exert no moment on BC, and so the only external moment is that due to gravity. The sum of the
moments about B is thus
X * * * * * *
MB = M1 b 1 + M3 b 3 + (−L b 3 )×[−mg(sin θ b 1 + cos θ b 3 )]
* * *
= M1 b 1 + M3 b 3 + mgL sin θ b 2 (4)
*
Applying (1), and equating the b 2 -components of (3) and (4) gives

1 1
   
mgL sin θ = mR2 + mL2 ω22 sin θ cos θ − mR2 (ω2 cos θ + ω1 )ω2 sin θ (5)
4 2

Solving (5) for ω1 yields


!
2L2 1 2gL
ω1 = 2
− ω2 cos θ − 2
R 2 R ω2

2(9.81 m/s2 )(0.3 m)


!
2(0.3 m)2 1 ◦
= − (2 rad/s) cos 30 −
(0.1 m)2 2 (0.1 m)2 (2 rad/s)
= −264 rad/s

* *
ω1 = −264 b 3 rad/s

218
8.8.24
GOAL: Determine the bending moment at the weld O in terms of m, ω, r, and g. Use two methods:
(i) Equations of motion for the mass center G, and (ii) Equations of motion about the fixed point
O.
GIVEN: The hemispherical shell rotates with constant angular velocity ω about the shaft axis, in
the direction indicated.
* * *
The shell has mass m and radius r.
DRAW: Let b 1 , b 2 , b 3 be a set of body-fixed unit vectors that, at the instant illustrated, are
aligned with the x, y, z axes, respectively.

GOVERNING EQUATIONS:
X * *
˙
MG = HG (1)
(i) X*
F = ma*G (2)

X * *
˙
(ii) MO = HO (3)

SOLVE: (i) The moments of inertia of the hemispherical shell about its mass center are I¯xx = 32 mr2
and I¯yy = I¯zz = 125
mr2 . All products of inertia are zero. The angular velocity of the shell is
* *
ω = ω b 3 . The expression for angular momentum is thus:

* * 5 *
HG = I¯zz ω b 3 = mr2 ω b 3 (4)
12
* * * *
Let the contraint forces and moments at the weld be written as F = F1 b 1 + F2 b 2 + F3 b 3 and
* * * *
M = M1 b 1 + M2 b 2 + M3 b 3 . For a hemispherical shell, the mass center G is located a distance 2r
along the x-axis, from either end of the shell. Substitution of the constraint forces and moments
into equation (1) yields:
X * *
˙
MG = HG

1 1
 
* * * * * *
⇒ F2 r b 1 + −F1 r + F3 r b 2 − F2 r b 3 + M1 b 1 + M2 b 2 + M3 b 3 = 0 (5)
2 2

219
 *
 *
r 1 2
The acceleration of the mass center is a*G = ω×
* *
ω× 2 b 1 = − 2 rω b 1 . The force balance, equation
(2), now yields:
* * * * 1 *
F1 b 1 + F2 b 2 + F3 b 3 − mg b 3 = − mrω 2 b 1 (6)
2
Equation (6) may be solved for the constraint forces, F1 = − 12 mrω 2 ; F2 = 0; F3 = mg. Using these
results in equation (5) gives us the moments:
1  
M1 = 0; M2 = − mr g + rω 2 ; M3 = 0
2
*
M = − 12 mr g + rω 2 b 2
*

(ii) If we center our work about the fixed point O we’ll see that we only need one equation to solve
for the bending moment, at a cost of slightly more complex inertia terms and a non-zero angular
*
momentum time-derivative. With only one angular velocity component, in the b 3 direction, the
angular momentum expression is
* * * *
O O O
HO = Izz ω b 3 − Ixz ω b 1 − Iyz ω b2 (7)

Using the parallel axis expressions, we find the moments and products of inertia appearing in the
above equation to be:

5 r2 2 2
   
O
Izz = I¯zz + m r12 + r22 = mr2 + m +0 = mr
12  4 3
r   1 2
O
Ixz = I¯xz + mr1 r3 = 0+m r = mr
2  2
O
Iyz = I¯yz + mr2 r3 = 0+m 0 r = 0

And from equation (7) we now have:


* 2 2 * 1 *
HO = mr ω b 3 − mr2 ω b 1
3 2
*
˙ 1 2 2*
⇒ HO = − mr ω b 2
2
Since the reaction forces at O exert no moment about O, equation (3) becomes;

* * * 1 * 1 *
M1 b 1 + M2 b 2 + M3 b 3 + mgr b 2 = − mr2 ω 2 b 2
2 2
*
M = − 12 mr g + rω 2 b 2
*

220
8.8.25
GOAL: (a) Determine the angular acceleration of the spherical shell when its angular velocity is
* *
ω = 10k rad/s. (b) Find the reaction moment at the weld O at this same instant. Report in the
x, y, z coordinate frame.
GIVEN: We are given the external motor torque that is driving the shaft, and the areal density
and radius of the shell.
* * * *
DRAW: Let b 1 , b 2 , b 3 be body-fixed axes aligned with the principle axes of the shell, while *
ı ,*
 ,k
are body-fixed axes attached to the rotating shaft.

* * *
ı  k
*
b1 sin θ 0 − cos θ
*
b2 0 1 0
*
b3 cos θ 0 sin θ

ASSUME: The mass of the shaft is negligible.


FORMULATE EQUATIONS: We will apply the rigid body equations of motion about point
O:
*
F = ma*G (1)


X * *
˙ d * *
*
MO = HO = HO + ω×HO (2)
N dt S

SOLVE: The location of the mass center with respect to point O is

r* 1
 
* * *
r*G/ = r*A/ + r*G/ = rk + b = r − cos θ b 1 + r sin θ b 3
O O A 2 1 2

The moments and products of inertia of the shell about its mass center are

2 5
I¯x0 x0 = mr2 I¯y0 y0 = I¯z 0 z 0 = mr2 I¯x0 y0 = I¯x0 z 0 = I¯y0 z 0 = 0
3 12

The moments and products of inertia about point O may be found using the parallel axis expres-

221
sions:
2 2 2
 
Ix00 x00 = mr + mr2 sin2 θ = 2ρπr4 + sin2 θ
3 3
"  2 #
5 2 1 5
 
2
Iy00 y00 = mr + m r − cos θ + r sin θ = 2ρπr4
2 2
− cos θ
12 2 3
2 2 #
"
5 1 5 1
  
Iz 00 z 00 = mr2 + mr2 − cos θ = 2ρπr 4
+ − cos θ
12 2 12 2
1 1
   
4
Ix00 z 00 = 0 + mr − cos θ (r sin θ) = 2ρπr sin θ − cos θ
2 2
Ix00 y00 = Iy00 z 00 = 0 + 0 = 0
Substituting the given geometric and mass properties gives
Ix00 x00 = 7.03×10−4 kg·m2 Iy00 y00 = 5.87×10−4 kg·m2
Iz 00 z 00 = 3.18×10−4 kg·m2 Ix00 z 00 = −1.11×10−4 kg·m2
The angular velocity of the system may be expressed as
* * * *
ω = ωk = ω(− cos θ b 1 + sin θ b 3 )
The angular momentum about point O is
h i*
Ix00 x00 ω1 − Ix00 y00 ω2 − Ix00 z 00 ω3 b 1 +
* h i*
HO = Iy00 y00 ω2 − Iy00 z 00 ω3 − Iy00 x00 ω1 b 2 +
h i*
Iz 00 z 00 ω3 − Iz 00 x00 ω1 − Iz 00 y00 ω2 b 3
 *  *
= −Ix00 x00 ω cos θ − Ix00 z 00 ω sin θ b 1 + Iz 00 z 00 ω sin θ + Ix00 z 00 ω cos θ b 3 (3)
Summing the moments about point O,
r*
 
X * * * *
MO = rk + b 1 ×(−mgk ) + MO1 *
ı + MO 2 *
 +T
2
1 * * * * * *
mgr sin θ b 2 + MO1 (sin θ b 1 + cos θ b 3 ) + MO2 b 2 − T cos θ b 1 + T sin θ b 3
= (4)
2
where T = −0.05 N·m.
Equation (2) may now be expressed in component form as
X * *
MO · b 1 = Ḣ1 + (ω2 H3 − ω3 H2 )
X * *
MO · b 2 = Ḣ2 + (ω3 H1 − ω1 H3 ) (5)
X * *
MO · b 3 = Ḣ3 + (ω1 H2 − ω2 H1 )
yielding the three equations
MO1 sin θ − T cos θ = −Ix00 x00 ω̇ cos θ − Ix00 z 00 ω̇ sin θ + 0 (6)
MO1 cos θ + T sin θ = Iz 00 z 00 ω̇ sin θ + Ix00 z 00 ω̇ cos θ + 0 (7)
1  
mgr sin θ + MO2 = 0 + (ω sin θ) −Ix00 x00 ω cos θ − Ix00 z 00 ω sin θ −
2  
(−ω cos θ) Iz 00 z 00 ω sin θ + Ix00 z 00 ω cos θ
   
= ω 2 Iz 00 z 00 − Ix00 x00 sin θ cos θ + ω 2 Ix00 z 00 cos2 θ − sin2 θ (8)

222
Equations (6) and (7) may be solved for ω̇ and MO1 , and with the substitutions θ = 40◦ , T =
−0.05 N·m and the computed inertia values we have
T
ω̇ = = −115 rad/s2
Ix00 x00 cos2 θ + Iz 00 z 00 sin2 θ + 2Ix00 z 00 sin θ cos θ
h i
MO 1 = ω̇ (Iz 00 z 00 − Ix00 x00 ) sin θ cos θ + Ix00 z 00 (cos2 θ − sin2 θ) = 0.0241 N·m

Equation (8) may be solved for MO2 ,


    1
MO 2 = ω 2 Iz 00 z 00 − Ix00 x00 sin θ cos θ + ω 2 Ix00 z 00 cos2 θ − sin2 θ − mgr sin θ =
2
= −2.81 N·m
* *
*
α *
= ω˙
= ω̇k = −115k rad/s2
*
MO = (0.0241 *
ı − 2.81*
 ) N·m

223
8.9 Energy of Three-Dimensional Bodies

224
8.9.1
GOAL: Determine the kinetic energy of the illustrated system and compare how the energy changes
for finite vs infinitesimal width.
*
GIVEN: Body’s orientation. ω = (20 *
ı − 10 *
 ) rad/s, a = 0.5 m, b = 0.4 m, h = 0.02 m or 0.
DRAW:

FORMULATE EQUATIONS:!From Appendix B we have: ! !


b 2 + h2 a2 + b2 a2 + h2
Ix0 x0 = m , Iy0 y0 = m , Iz 0 z 0 = m
12 12 12
To find the mass moments of inertia and products of inertia we use (8.24)-(8.29) with r1 = a/2,
r2 = h/2 and r3 = b/2.
Ixx = Ix0 x0 + m(r22 + r32 )
     
2
+ h2
= m b 12
2
+m b + h2 2
=m b + h2
4 3

Iyy = Iy0 y0 + m(r32 + r12 )


     
2
+ b2
= m a 12
2
+m a + b2 2
=m a + b2
4 3

Izz = Iz 0 z 0 + m(r12 + r22 )


     
2
+ h2
= m a 12
2
+m a + h2 2
=m a + h2
4 3
ah
 
Ixy = Ix0 y0 + mr1 r2 = m
4
bh
 
Iyz = Iy0 z 0 + mr2 r3 = m
4
ab
 
Izx = Iz 0 x0 + mr3 r1 = m
4
SOLVE:
Having all the relevant rotational inertias and knowing that
1* *
KE = ω · HO
2

225
we can form the body’s kinetic energy using (8.33) with ω1 = 20 rad/s, ω2 = −10 rad/s, ω3 = 0:
    

 b2 + h2 (20 rad/s) − ah (−10 rad/s) *ı+




 3  4 


m    
KE = [(20 *
ı − 10 *
 ) rad/s] · a2 + b2 (−10 rad/s) − ah (20 rad/s) *
+
2 

 h 3 4 


 − ab (20 rad/s) − bh (−10 rad/s) k
 i* 
 

4 4

m 20 rad/s  2  10 rad/s 10 rad/s  2


     
KE = (20 rad/s) b + h2 + ah − (10 rad/s) − a + b2 − (5 rad/s)ah
2 3 4 3

50 250 200
       
KE = m a2 + b2 + h2 + 50ah ( rad/s)2
3 3 3
Using a = 0.5 m, b = h0.4 m and
 h  = 0.02 mgives us
50 2 250 + (0.02)2 200 + 50(0.5)(0.02) m/s2
  i
KE = m (0.5) 2 + (0.4)
3 3 3
= m(18.03 m/s2 )
Neglecting h gives us
KE = m(17.5 m/s2 )
Thus the reduction in kinetic energy
 is equal to 
18.03 − 17.5
100 ⇒ 2.92%
18.03

226
8.9.2
GOAL: Determine the kinetic energy of the illustrated system.
* * * * *
GIVEN: Body’s orientation. v*G = (1.5 b 1 + 1.5 b 2 + 10 b 3 ) m/s, ω
*
= (5 b 1 + 15 b 3 ) rad/s.
DRAW:

FORMULATE EQUATIONS: ! ! !
a2 + c2 b2 + a2 b2 + c2
Ixx = m , Iyy = m , Izz = m
12 12 12
The x,y,z axes are centered on the body’s mass center and parallel to the principal axes and thus
there are no products of inertia to consider.
SOLVE:
Having all the relevant rotational inertias and knowing that
1* * 1 *
KE = ω · HO + v*G · L
2 2
we can form the body’s kinetic energy. With no products of inertia the kinetic energy simplifies to
1 1
KE = (Ixx ω12 + Iyy ω22 + Izz ω32 ) + m(v12 + v22 + v32 )
2 2
      
KE =m a2 + c2 ω 2 + b2 + a2 ω 2 + b2 + c2 ω 2
2 12 1 12 2 12 3

+m
 
2 2 2
2 v1 + v2 + v3
    
=m a2 + c2 25 + b2 + c2 225 ( rad/s)2
2 12 12

+m
2 (2.25 + 2.25 + 100) ( m/s)
2

  
KE = m 25a2 + 225b2 + 250c2 ( rad/s)2 + 104.5( m/s)2
2 12

227
8.9.3
GOAL: Determine the kinetic energy of the illustrated system.
*
GIVEN: Body’s orientation. ω = (8 *
ı + 6*
 ) rad/s, r = 0.2 m and h = 0.6 m.
DRAW:

FORMULATE EQUATIONS:
From Appendix B we have ! !
3m(4r2 + h2 ) 3mr2
Ix0 x0 = Iy0 y0 = , Iz 0 z 0 = IZZ =
80 10
To find the mass moments of inertia and products of inertia we use (8.24)-(8.29) with r1 = r2 = 0
and r3 = 3h
4. !
9mh2 3m(r2 + 4h2 )
IXX = IY Y = Ix0 x0 + =
16 20

Ixy = Iyz = Izx = 0


SOLVE:
Having all the relevant rotational inertias and knowing that
1* *
KE = ω · HO
2
we can form the body’s kinetic energy using (8.33) with ω1 = 8 rad/s, ω2 = 6 rad/s, ω3 = 0:
3m(r2 + 4h2 )
 
* 
(8 rad/s) ı 


1 * *

 20 

KE = [(8 ı + 6  ) rad/s] ·
2  3m(r 2 + 4h2 )

 

* 


20 (6 rad/s) 

(0.2 m)2 + 4(0.6 m)2


 
(8 rad/s) *
ı

 

3m *

 20 

KE = [(8 ı + 6 *
 ) rad/s] ·
2  2 2 
 (0.2 m) + 4(0.6 m) (6 rad/s) *

 

20  

KE = m(7.104 + 3.996)( m/s)2 = 11.1m( m/s)2

228
8.9.4
GOAL: Determine the kinetic energy of the illustrated system.
*
GIVEN: Body’s orientation. ω = (20 *
ı − 10 *
 ) rad/s, a = 0.5 m, b = 0.4 m, h = 0.02 m or 0.
DRAW:

FORMULATE EQUATIONS:
! ! !
b2 a2 a2 + b2 ab
 
Ixx =m , Iyy = m , Izz = m , Ixy = −m , Iyz = Izx = 0
18 18 18 36

To find the mass moments of inertia and products of inertia we use (8.24)-(8.29) with r1 = a/3,
r2 = b/2 and r3 = 0.
IXX = Ixx + mr22
     
b2
= m 18
2
+ m b9
2
= m b6

IY Y = Iyy + mr12
     
a2
= m 18 + m a9
2
= m a6
2

IZZ = Izz + m(r12 + r22 )


     
2
+ b2
= m a 18
2
+m a + b2 2
=m a + b2
9 6
ab ab ab
     
IXY = Ixy + mr1 r2 = −m +m =m
36 9 12

IY Z = IZX = 0
SOLVE:
Having all the relevant rotational inertias and knowing that
1* *
KE = ω · HO
2
we can form the body’s kinetic energy using (8.33) with ω1 = 6 rad/s, ω2 = 6 rad/s, ω3 = 0:
    
2
b ab *
6  (6 rad/s) − 12 (6 rad/s)  ı +

 

m * *
 
KE = [(6 ı + 6  ) rad/s] · 
2 
 a2 (6 rad/s) − ab (6 rad/s) * 


 6 12 

KE = 3m a2 + b2 − ab ( rad/s)2


229
8.9.5
GOAL: Determine the kinetic energy of the arm AB.
GIVEN: Body’s orientation.
DRAW:

FORMULATE EQUATIONS:
Let L1 = |OA| and L2 = |AB|.
* * *
From the given information of the original problem we see that ω AB = ψ̇ b 3 − θ̇ b 2 .
Using the general formula for the mass moment of inertia of a thin rod along with the parallel axis
theorem gives us
mL22 mL22
IXX = mL21 , IY Y = , IZZ = + mL21
3 3
The only non-zero product of inertia is IXY :
L
Z2 ρL1 L22 mL1 L2
IXY = ρ L1 x dx = =
2 2
0

SOLVE:
Having all the relevant rotational inertias and knowing that
1* *
KE = ω · HO
2
we can form the body’s kinetic energy using (8.33) with ω1 = 0, ω2 = −θ̇, ω3 = ψ̇:
 mL1 L2 *
 


 2 b 1 (θ̇)+ 



   

 

 mL2

 

1 * * 2 *

KE = [(−θ̇ b 2 + ψ̇ b 3 )] ·
2  3 (−θ̇) b 2 + 

 

 
mL22

 ! 

 *

 2 
3 + mL1 ψ̇ b 3

 

 

L2 L22 L21
" ! #
KE = m 62 θ̇2 + 6 + 2 ψ̇ 2

230
8.9.6
GOAL: Determine the kinetic energy of the rotating disk.
GIVEN: Body’s orientation. r = 0.23 m, h = 0.01 m, a = 0.3 m, ω1 = 10 rad/s, ω2 = 70 rad/s,
m = 1.2 kg.
DRAW:

FORMULATE EQUATIONS: Let the disk’s radius be denoted by r. From the given informa-
* * *
tion in the original problem we see that ω AB = −ω1  + ω2 ı .
We’ll determine the kinetic energy associated with the speed of the mass center ( 21 mvG 2 ) and that

associated with rotation about the mass center.


From Appendix B we have  
m 3r2 + h2 mr2
Iyy = Izz = , Ixx =
12 2
All products of inertia are zero.
SOLVE:
Having all the relevant rotational inertias and knowing that
1 2 1* *
KE = mvG + ω ·H
2 2 D G
we can form the body’s kinetic energy using (8.33) with ωD = 70 * ı rad/s − 10 *
 rad/s. Note that
from the definition in the original problem ω1 indicates rotational velocity about the (negative) Y
axis and ω2 indicates rotational velocity about the x axis.
The component of kinetic energy due to motion of the mass center is given by
1 2 1 1
KE = mvG = m(aω1 )2 = (1.2 kg)(0.3 m)2 (10 rad/s)2 = 5.4 J

trans 2 2 2
The rotational kinetic energy is found from  
2

 mr *
(ω2 ) ı + 

1

 2 

* *
KE = [(ω2 ı − ω1  )] ·

rot 2  m(3r 2 + h2 )

 

* 
(−ω ) 
 
12 1
2 2
(3r2 + h2 )ω12
" #
m r ω2
KE = KE + KE = 5.4 J + +

trans rot 2 2 12

" #
1.2 kg (0.23 m)2 (70 rad/s)2 (3(0.23 m)2 + (0.01 m)2 )(10 rad/s)2
KE = 5.4 J + + = 84.0 J
2 2 12

231
8.9.7
GOAL: Determine the kinetic energy of the rotating system.
GIVEN: Body’s orientation. r = 0.004 m, h = 0.1 m.
DRAW:

FORMULATE EQUATIONS: From the given information in the original problem we see that
* * * * *
ωA = ω1 b 1 + ω2 b 2 = (−50 b 1 + 2 b 2 ) rad/s.
We’ll consider three components of the kinetic energy of the system: the rotational energy of the
3-arm body about O, the translational kinetic energy of the mass center of each wheel and the
rotational energy of each wheel about its mass center.
Approximating the arms as thin rods lets us calculate the mass moment of inertia of the 3-arm
body about the Y axis:
m3−a h2
!
I3−a = 3 = m3−a h2
3
Approximating the wheels as thin disks we have, from Appendix B:
mr2 mr2
Izz = Iyy = , Ixx =
4 2
All products of inertia are zero.
SOLVE:
Having all the relevant rotational inertias and knowing that
1 1 1* *
KE = I3−a ω22 + mvG 2
+ ω ·H
2 2 2 w G
* *
we can form the body’s kinetic energy using (8.33) with ωw = −50 b 1 rad/s + 2 b 2 rad/s.
The kinetic energy due to rotation of the 3-arm body is given by
1 1
KE = I3−a ω22 = (0.005 kg)(0.1 m)2 (2 rad/s)2 = 1.0×10−4 J

3−a 2 2
The component of kinetic energy due to motion of the mass center of the wheels is given by
1
KE = 3 mw h2 ω22 = 1.5(0.002 kg)(0.1 m)2 (2 rad/s)2 = 1.2×10−4 J

trans 2
Finally, the rotational kinetic energy of the wheels is found from
mr2 ω *
 

 2
 b
1 1
+ 

1 * *

KE = 3 [(ω1 b 1 + ω2 b 2 )] ·

rot 2  mr 2 *

 

ω b

4 2 2

232
r2 ω12 r2 ω22
" #

KE = 3m 2 + 4

rot 2

3(0.002) kg (0.004 m)2 (−50 rad/s)2 (0.004 m)2 (2 rad/s)2


 
= 2 2 + 4

= 6.00×10−5 J
Summing the three components gives us
KE = 1.0×10−4 J + 1.2×10−4 J + 6.00×10−5 J = 2.8×10−4 J

233
8.9.8
GOAL: Determine the kinetic energy of a rolling cone.
GIVEN: Body’s orientation and dimensions. m = 22 g.
DRAW:

FORMULATE EQUATIONS: First we need to determine the cone’s angular velocity. Example
8.4 shows how to determine the angular velocities of a rotating disk on the end of a bent shaft. Our
rolling cone can be thought of as precisely the same problem. The motion of OA will be the same
−1 ◦
√ 8.4’s shaft. From geometry we have β = sin (0.25) = 14.48 . r = |AB| = 1 in
as that of Example
2 2
and h = |OA| = 4 − 1 in = 3.873 in
The cone takes 2 s to complete one full rotation around the floor and thus we have

ω = = π rad/s
OA 2s
The correspondence between our problem and that of Example 8.4 is
ωcone → ωW
ω → ωS
AO
3.873 in → L1
1 in → L2
Substituting these values into the expression for ωW gives us
 
ωcone = (π rad/s) sin β − 3.873 cos β + sin β *
c 1 + (π rad/s) cos β *
c3
1
* *
= (−11.78 c 1 + 3.042 c 3 ) rad/s
From Appendix B we have

(Note that these axes do not directly correspond to the ones in our problem). For this figure we
have
3m(4r2 + h2 ) 3mr2
Ix0 x0 = Iy0 y0 = , Iz 0 z 0 = IZZ =
80 10
For our problem m = 22 g, r = 1 in = 0.0254 m and h = 3.87 in = 0.0984 m.
To find the mass moments of inertia and products of inertia we use (8.24)-(8.29) with r2 = r3 = 0
and r1 = 3h
4.

234
9mh2 3m(r2 + 4h2 )
IY Y = IZZ = Iy0 y0 + =
16 20
IXY = IY Z = IZX = 0
SOLVE:
Having all the relevant rotational inertias and knowing that
1* *
KE = ω · HO
2
we can form the body’s kinetic energy using (8.33) with ω1 = 8 rad/s, ω2 = 6 rad/s, ω3 = 0:
 
2

 3mr (−11.78 rad/s) *
c 

 10 1 
1 * *
 
KE = [(−11.78 c 1 + 3.042 c 3 ) rad/s] ·
2 2 2
 3m(r + 4h ) (3.042 rad/s) *

 

c
 

20 3

(0.0254 m)2
 
(−11.78 rad/s) *
c1

 

3(0.022 kg)

 10 

KE = [(−11.78 *
c 1 + 3.042 *
c 3 ) rad/s] ·
2  2 2 
 (0.0254 m) + 4(0.0984 m) (3.042 rad/s) *

 

20 3c 

KE = 8.96×10−4 J

235
8.9.9
GOAL: Determine the angular momentum of the illustrated body about the fixed point O.
GIVEN: The body rotates about the Y -axis with angular speed ω, and has areal density ρ.
DRAW:

FORMULATE EQUATIONS: Let x, y, z be a set of body-fixed axes with origin O. At the


instant illustrated, these axes are aligned with the ground-fixed X, Y, Z axes. The body rotates
about the Z-axis with angular speed ω; thus the z and Z axes remain aligned and the angular
*
velocity may be expressed as: ω = ω*  . The angular momentum of the body about point O is
then:
  
*
IXX −IXY −IXZ 0
* *
HO = IO ω =  −IY X IY Y −IY Z   ω  = −IXY ω *
ı + IY Y ω *
 − IY Z ω k (1)
  
−IZX −IZY IZZ 0

Once we have determined the relevant inertia values about the point O, we may substitute and
solve for the angular momentum.
SOLVE: We only need to determine IXY , IY Y and IY Z . IY Y is easily found as the mass moment
of inertia about the Y axis is simply that of two thin rods, with masses ρab, ρac and lengths b, c,
respectively, rotated about their ends:
(ρac)c2 (ρab)b2 ρa(b3 + c3 )
IY Y = + =
3 3 3
Using the appropriate equations from (8.21)-(8.29) lets us determine the products of inertia. Each
rectangle has no product of inertia when evaluated about its mass center and thus the only finite
products of inertia come about because we’re shifting our attention from each plate’s mass center
to the point O.
a −b ρa2 b2
 
IXY = ρ(ab) =−
2 2 4
ac ρa2 c2
IY Z = ρ(ac) =
22 4
Using these inertia values gives us !
* ab2 * b3 + c3 * ac2
HO = ı +  − ρaω
4 3 4
The kinetic energy is found from !
1* * 1 b3 + c3
 
KE = ω · HO = ω *
 · *
ρaω  = b3 + c3 ρaω 2
2 2 3 6

236
8.9.10
GOAL: Determine the kinetic energy of the illustrated rectangular body about the fixed point O.
GIVEN: The body rotates about the Z-axis with angular speed ω, and has mass m.
DRAW:

FORMULATE EQUATIONS: Let x, y, z be a set of body-fixed axes with origin O. At the


instant illustrated, these axes are aligned with the ground-fixed X, Y, Z axes. The body rotates
about the Z-axis with angular speed ω; thus the z and Z axes remain aligned and the angular
* *
velocity may be expressed as: ω = ω k . The angular momentum of the body about point O is
then:   
*
Ixx −I xy −I xz 0
* *
HO = IO ω =  −Iyx Iyy −Iyz   0  = −Ixz ω * ı − Iyz ω *
 + Izz ω k (1)
  
−Izx −Izy Izz ω
Once we have determined the relevant inertia values about the point O, we may substitute and
solve for the angular momentum.
SOLVE: The products of inertia about parallel axes through the center of mass G are all zero, due
to the symmetry of the body. The products of inertia about the point O may be computed from
the parallel axis expressions for inertia.
a b 1
  
Ixz : ¯
Ixz = Ix0 z 0 + mr1 r3 = 0 + m − = − mab
2 2 4
c b 1
  
Iyz : Iyz = I¯y0 z 0 + mr2 r3 = 0 + m − = − mcb
2 2 4
The moment of inertia about the z 0 -axis through the mass center is: I¯z 0 z 0 = 1
a2 + c2 . Using

12 m
the parallel axis expressions for inertia again, we obtain:
"   2 #
2
  1  2  a c
Izz = I¯z 0 z 0 + m r12 + r22 = m a + c2 + m +
Izz : 12 2 2
1  2 
= m a + c2
3
Substituting into (1),
1 * 1 * 1 2
 
* *
HO = mω ab ı + cb  + a + c2 k
4 4 3
The kinetic energy is found from
1 * 1 * a2 + c2 2
c2 mω 2
mω k = a +
*
KE = ω · HO = ω k · 6
2 2 3

237
8.9.11
GOAL: Determine the kinetic energy of the illustrated system.
* *
GIVEN: Body’s orientation. ω = ωk .
DRAW:

FORMULATE EQUATIONS:
For our problem we’ll find the moments of inertia for two solid cylinders, one of radius r1 and one
of radius r2 . By subtracting we’ll obtain the inertias for the ring. The mass of the ring is given by
m = πhρ(R22 − R12 )
Outer radius cylinder:
o o
πR22 hρ(3R22 + h2 )
Ix0 x0 = Iz 0 z 0 =
12

o
πR22 hρR22 πhρR24
Iy0 y0 = =
2 2
Inner radius cylinder:
i
πR12 hρ(3R12 + h2 )
Ix0 x0 = i Iz 0 z 0 =
12

i
πR12 hρR12 πhρR14
Iy0 y0 = =
2 2
We now subtract the inertias of the
h  smaller cylinder
 from the
i inertias
h of the larger one.
i
4 4 2 2 2
π hρ 3 R2 − R1 + h R2 − R1 m h2 + 3 R12 + R22
Ix0 x0 = Iz 0 z 0 = =
12 12
    
π hρ R22 − R12 R12 + R22 m R12 + R22
Iy0 y0 = IY Y = =
2 2
Using the parallel axis theorem to find IXX and IZZ we obtain
h  i
m h2 + 3 R12 + R22  2
h
2 2
mh2 m(R1 + R2 )
IXX = IZZ = +m = +
12 2 3 4
The products of inertia about X, Y , Z are zero from symmetry.
* *
The general expression for angular momentum (using ω = ω k ) is therefore
" 2 2 #
* mh2 m(R1 + R2 ) *
HO = + ωk
3 4
SOLVE:
Having the relevant rotational inertia and knowing that

238
1* *
KE = ω · HO
2
we can form the body’s kinetic energy using (8.33):
" 2 2 #
1 * mh2 m(R1 + R2 ) *
KE = [ω k ] · + ωk
2 3 4

m(R12 + R22 ) 2
" #
2
KE = mh
6 + 8 ω

239

You might also like